Comprehensive coverage

The most distant supermassive black hole to date has been discovered

A fascinating discovery from the young universe: researchers have discovered the most distant supermassive black hole observed so far, only 690 million years after the Big Bang. The researchers were surprised by its enormous mass, which is 800 million times the mass of the Sun, and still do not understand how it could have collected so much mass in such an early period.

Imaging of the quasar J1342+0928, the active galactic halo that contains at its center a supermassive black hole, with a mass 800 million times that of our Sun, that is gobbling up a lot of material from a hot disk surrounding it. Source: Robin Dienel/Carnegie Institution for Science.
A visualization of the quasar J1342+0928, the active galactic nucleus that contains a supermassive black hole at its center, with a mass 800 million times that of our Sun, that is gobbling up a lot of matter from a hot disk surrounding it. source: Robin Dienel/Carnegie Institution for Science.

An international team of researchers announced yesterday the discovery of the most distant supermassive black hole known to date, whose light took more than 13 billion light years to reach us, from a time when the universe was only 690 million years old, or 5% of its current age (13.8 billion years) . The research findings were published yesterday in the journal Nature.

The black hole, named J1342+0928 (a code name indicating its location in the sky), has a massive mass of 800 million times the mass of our Sun. The supermassive black hole absorbs a lot of material from a hot disk swirling around it at the center of a galaxy - a phenomenon known as Quasar.

"Quasars are among the brightest and most distant sky objects, and are critical for understanding the young universe," said Researcher Bram Vanmans, from the Max Planck Center for Astronomy in Germany. The particular quasar in which the supermassive black hole resides has a luminosity equal to 40 trillion times the luminosity of our Sun.

The enormous distance of the quasar and the black hole at its center was determined by measurement The redshift of his light in the so-called phenomenon Doppler effect, a wavelength of light from a source moving away from the viewer, "melts" and changes to a longer wavelength. Due to the acceleration in the expansion of the universe, the farther the object is from us, the farther it also moves away from us, and its light is redshifted more. The redshift of the distant quasar is about 7.5, meaning that its wavelength as seen from Earth is According to 8.5 (7.5+1) than it was when it was originally ejected from it more than 13 billion years ago.

The mystery of the mass of the primordial black hole

The enormous mass of the distant and ancient black hole surprised researchers. Existing theories explaining how supermassive black holes form and develop, absorbing matter from gas and stars at the center of galaxies, fail to explain how the black hole grew so fast, so short a time, in cosmological terms, after the Big Bang.

"Collecting all this mass in less than 690 million years is a huge challenge for theories of supermassive black hole growth," said Eduardo Benidos, from the Carnegie Center in the USA, who led the research.

An illustration trying to illustrate the vast distance of the supermassive black hole, whose light took more than 13 billion light years to reach the light of two Magellan telescopes in Chile, one of which is shown in the bottom of the illustration. Source: Robin Dienel, courtesy of the Carnegie Institution for Science.
An illustration illustrating the enormous distance of the supermassive black hole, whose light took more than 13 billion light years to reach the Magellan telescopes in Chile, one of which is shown at the bottom of the illustration. source: Robin Dienel, courtesy of the Carnegie Institution for Science.

"If you start with a seed like a big star, and let it grow at the maximum possible rate, and start from the moment of the big bang, you are not able to create something with 800 million solar masses - it is not realistic", He explained Robert Simcoe, a physics professor from MIT, who participated in the study. "Therefore there must be another way in which it was created. And how exactly it happened, no one knows."

According to the researchers, in order to explain the formation of such a massive black hole in such an early period of the universe, models for the formation of supermassive black holes would have to show that "seeds" of black holes with a mass of up to about 10,000 solar masses could have existed in the young universe , in a period of only 65 million years after the Big Bang, or, alternatively, to explain how the earliest black holes grew at a greater rate than is currently accepted.

Evidence from the young universe

The uniqueness of the black hole also stems from the period in which the researchers speculate that it existed. Light from the quasar took more than 13 billion years to reach Earth, and it was ejected from it when the age of the universe was only 690 million years old, during the period known as the "reionization phase" (Reionization), or "re-ionization" in Hebrew.

Until about 380,000 years after the Big Bang, the universe was completely dark, because photons were trapped within the dense, hot plasma of particles and electrons. But even after the universe cooled enough to allow the appearance of hydrogen atoms with a neutral electrical charge, and became transparent to light, it remained dark until the first stars shone. Then the period known as the "reionization phase" began. that took place Between 13.5 and 12.5 billion years ago, when the energy emitted by the first stars and galaxies caused most of the hydrogen atoms in the young universe to ionize - to lose their electron.

"The reionization phase was the last significant change in the universe, and it is considered one of the frontiers of astrophysical research," Benaidos said.

By analyzing the spectrum of light emitted from the environment near the quasar, the researchers were able to determine that between 38 and 77 percent of the hydrogen in its environment is in the form of neutral atoms, rather than ions. This means that the quasar's environment has not yet had time to become largely ionized, and that it existed during the reionization period - the evidence The unit which was discovered from this ancient period of the universe.

In a comparison chart between the most massive and the most distant quasars known to date. While J1342+0928 (the yellow star) is not the most massive quasar, it is the most distant one discovered to date. Source: Jinyi Yang/UA; Reidar Hahn/Fermilab; M. Newhouse/NOAO/AURA/NSF.
The chart compares the supermassive black holes with the largest mass and distance known to date. While J1342+0928 (the yellow star) is not the most massive, it is the most distant discovered to date. source: Jinyi Yang/UA; Reidar Hahn/Fermilab; M. Newhouse/NOAO/AURA/NSF.

There are now a number of competing theories as to when exactly the reionization phase of the universe occurred. According to the researchers, the new evidence indicates that reionization occurred relatively late in the cosmological chronology.

"This adds to our overall understanding of the universe, because we have identified the point in time where the universe is halfway through the very rapid transition from a neutral state to an ionized state," said Simco "We now have the most precise measurement of the date when the first stars lit up."

The search for the most distant quasar

Distant quasars are difficult to detect. In recent years, especially since 2010, astronomers have been able to double the number of known quasars that date from about 850 to a billion years after the Big Bang.

In order to "break the record" and locate a distant quasar from an even earlier period, the team of researchers led by Eduardo Benido used a complex and systematic method to map millions of candidates.

To identify candidates for quasars, the researchers used complex data from a number of extensive sky surveys, with the aim of locating objects that, on the one hand, are bright in the infrared range, and on the other hand, undetectable in the visible light range - this is because, due to the Doppler effect, the redshift of the light from distant quasars makes them For brighter ones in infrared.

The sky surveys used by the researchers included those they produced WISE space telescope, "The Infrared Telescope of the United Kingdom" (UKIRT) at Mauna Kaa in Hawaii, and the "dark energy camera" (DECam) in Chile.

Out of the hundreds of millions of targets observed in these sky surveys, the researchers were able to reach several hundred quasar candidates. These candidates were given more in-depth observations using several telescopes, especially two Magellan telescope in Chile.

This Sisyphean effort eventually yielded the discovery of a single quasar from this early period of the universe, however, given that the researchers Viewers That there are only between 20 and 100 distant and bright quasars like J1342+0928 in the entire sky, this is a very significant discovery.

The two small photographs above show the difference in quasar brightness in visible and infrared light. While in infrared (right) the quasar is clearly visible, in visible light it cannot be seen at all, because the Doppler effect has caused its visible light to be distorted into longer wavelengths - infrared. At the bottom of the image is the spectrum of the quasar, as recorded by spectrometers at two Magellan telescopes in Bazila. Using the spectrum of the object, the researchers were able to measure the enormous mass of the supermassive black hole that resides at the center of the quasar. Source: Eduardo Bañados/Carnegie Observatories and Xiaohui Fan/UA.
The two small photographs at the top of the image show the difference in quasar brightness in visible and infrared light. While the quasar is clearly visible in the Magellan telescope's infra-red image (right), in visible light it cannot be seen at all, because the Doppler effect caused its visible light to be diverted to longer wavelengths - such as infra-red. At the bottom of the image is the spectrum of the quasar, as recorded by spectrometers at two Magellan telescopes in Chile. By analyzing its spectrum, the researchers were able to measure the enormous mass of the quasar's supermassive black hole. source: Eduardo Bañados/Carnegie Observatories and Xiaohui Fan/UA.

The galaxy in which the black hole resides also surprises the researchers

using the radio telescope array NOEMA in the French Alps and the very large array (VLT) in New Mexico in the USA, a German team of researchers from the Max Planck Institute for Astronomy examined the characteristics of the galaxy in which the supermassive black hole resides. They published the their findings In a companion article to the main article in The Astrophysical Journal Letters.

The researchers discovered to their surprise that the galaxy, which is also of course from that ancient period of 690 million years after the big bang, contains very high amounts of dust and heavy elements. Due to the immense brightness of the quasar at the center of the galaxy, researchers are unable to observe its starlight directly, at least not with existing telescopes. Nevertheless, the fact that it contains a higher amount of heavy elements, which are formed in the centers of stars, indicates that the galaxy probably contains a high amount of stars.

The mass of all the stars of the distant galaxy is comparable, according to the researchers' estimate warn Because it is not completely accurate, about 20 billion times the mass of our sun. If you compare this to the Milky Way galaxy, the mass of all the stars in it is between 40 and 60 billion times the mass of the Sun, this is a very large amount of stars, certainly in such a young period of the universe.

This discovery regarding the galaxy containing the distant quasar will allow, according to the researchers, to refine and refine existing models regarding the formation of galaxies.

"Models of galaxy evolution will have to explain how a galaxy is able to form the stars required to produce the observed amounts of dust and heavy chemical elements in such a relatively short time," said researcher Bram Vannemans of the Max Planck Institute for Astronomy.

What next?

The supermassive black hole, as well as the galaxy in which it resides, will now become a target for follow-up observations. Several observations have already been confirmed, using the Hubble Space Telescope that will observe the quasar in the near-infrared range, the Spitzer Space Telescope in the infrared range and the Chandra Space Telescope that will observe it in X-rays. In addition, observations in the array of radio telescopes have been confirmed ALMA in the Atacama Desert in Chile, in order to continue to study the quasar's host galaxy.

The space observatory, which is considered the future successor of the Hubble Space Telescope, James Webb Space Telescope, which is expected to be launched in 2019 into space, will greatly aid in the future study of both this particular quasar and the young universe. The telescope will observe space in the infrared range in order to allow it to look as far as possible, andto reveal evidence From stars and galaxies from an even earlier period of the universe, up to about 13.5 billion years ago.

See more on the subject on the science website:

319 תגובות

  1. The information that there is such a black hole is really scary to think about its size now because it happened relatively at the beginning of the universe so now when billions of years have passed after which it managed to eat a few more stars (when a black hole eats a star then its mass increases) so now it is a black hole with a huge mass Its size is like that

  2. The information that there is such a black hole is really scary to think about its size now because it happened relatively at the beginning of the universe so now when billions of years have passed after which it managed to eat a few more stars (when a black hole eats a star then its mass increases) so now it is a black hole with a huge mass Its size so that a ball in our country is nothing for him?

  3. We are back from Santa Barbara.

    Discussion summary.

    In the Scientific American article "A quantum threat to relativity", it is claimed that non-locality in quantum entanglement, which includes an immediate effect at a distance, contradicts the special theory of relativity according to which no effect can travel faster than light.

    The article states that although for many years the physicists and philosophers thought that the so-called contradiction did not exist, in recent years there have been more and more voices saying that the contradiction does exist.

    The explanation for why is, in my opinion, another article that describes the Weiler experiment, according to which the past can be influenced from the future.

    According to the theory of special relativity, speed on Orit means a journey into the past. Although this is not a direct contradiction to Kosiliyya, but still a journey into the past is something that is difficult to accept from a physical point of view. Ili Paat - by.

    I brought up the idea that you don't need to distill entanglement and non-locality to see that quanta and relations don't add up. At the root of the thought experiment from which relativity developed as written in Einstein's original relativity essay, is a photon with a definite momentum and position at a definite moment. Although this idea made perfect sense in 1905, it contradicts the uncertainty principle, the fundamental principle of quantum mechanics.

    The problem is ignited in the following paragraph:

    "You can emit a photon from point A and after a suitable time receive it at B.

    But if it is spread in the space before, and with equal probability, then there are two possibilities:

    1. The reality of the photon beyond B is not something physical - it is only the wave function or philosophy or zebra virology, but not the photon itself.

    (A bit strange if we consider that the square of the wave function is the probability of finding the particle at a certain point).

    2. The particle itself is also beyond B.

    In case 1, then there are no realities for a photon that moves faster than c, Einstein is right and Quantum regrets. But to my understanding, this is not the case.

    In case 2, if the photon itself is beyond B in a time less than ct, then there is no escaping the conclusion that it moves faster than c and in fact at any speed.'

    I still haven't received an answer from anyone here about the question expressed in the last line:

    "In case 2, if the photon itself is beyond B in a time less than ct, then there is no escaping the conclusion that it moves faster than c and in fact at any speed".

    So does anyone here volunteer to explain?

    Ariel, when I went through the comments I discovered your response that was pending at the time. Perhaps you will try to answer and prove with your own eyes that there are things that simply do not yet have an unequivocal answer?

    A discussion was held between Nissim and me about the strangeness of relativity and especially the lengthening of length, but we did not continue with the other points:

    If Maxwell's model is wrong and there is no ether - how did he manage to use it to find the speed of light? After all, he does not exist!

    In the original twin paradox, each twin can claim that their time is the real time. But if the bang theory is correct, there is an absolute time that is also expressed in the temperature of the universe according to the Friedman formula. Therefore when the twins meet again, the remaining twin's time is universe time while the traveler's time is artificial time.

    Development of this idea means that if we reverse the formation and the traveling twin is stationary with respect to the radiation, then the time of the remaining twin is higher than the age of the radiation and therefore also the age of the universe.

    So how can his time be higher than the age of the universe he is a part of?

    Finally, a personal point that I'm glad was clarified in this article.

    For years, I have accused Ai Albenzo of various claims, the first of which is that I am a shameless liar.

    I asked him to apologize, but he refused and also provided 3 examples in this article that I am a liar:

    "Yes, you definitely lied in the many discussions we had. It's funny of you to expect me to now know how to quote you exactly where, how much, why and how considering that these are discussions that some of them were 3 or 4 years ago, but I have a few examples in mind. One example is that for a long period of time you would go back and write a quote of mine over and over again, which miraculously would always get cut off in the middle of the sentence. I have explained to you many times that you are citing half a quote and thus completely changing what I said, but it didn't bother you. The first part of the sentence concerned the fact that there are people who do not understand quantum mechanics and therefore claim that it is wrong - and you have the same problem. The part you would have omitted made it clear that I am not accusing you of denying quantum mechanics but of the same kind of mistake - you do not understand something and therefore conclude that it is probably wrong. Another example is that during a long discussion about solving one or another coin puzzle with the help of interlacing, you claimed that I said it had a classical solution even though I explicitly wrote many times that it did not have a classical solution. I was just trying to explain to you that the quantum solution does not include information transfer because it requires the pre-synchronization of a huge number of entangled pairs - which is the quantum equivalent of pre-coordination of code, cheating according to the rules of the game. And yet you repeatedly accused me of claiming that there is a classical solution.'

    It's a bit hard to know what he means because unlike me who always quotes in the source, there are no quotes here and you have to trust the writer that he knows what he is saying. But if someone asks, I'd be happy to bring those "quotes that miraculously always had a stamp cut out for you in the middle of the sentence". I remember them and also know where they are and believe that they will only worsen the situation of Albenzo and his liar's claims.

    "Another example is that during a long discussion about solving one or another coin puzzle with the help of interlacing, you claimed that I said it had a classical solution even though I explicitly wrote many times that it did not have a classical solution. I was just trying to explain to you that the quantum solution does not include information transfer because it requires the pre-synchronization of a huge number of entangled pairs - which is the quantum equivalent of pre-coordination of code, cheating according to the rules of the game. And yet you repeatedly accused me of claiming that there is a classical solution.'

    And here is what Albenzo wrote:

    "There is no difference between this solution and any other solution of a pre-coordinated classical code (as some of the commenters here have tried to suggest), except that this code operates according to quantum laws, and in particular it is found in superposition (as we know, the secret of the magic of entanglement is that it is a state of superposition, and not just a special superposition which is not spherabile to pure subsystems)'.

    I'm willing to bet anyone who wants to for any amount that if we take a random sample of 100 people and read them the last sentence, the vast majority will think that it means that there is no difference between a pre-coordinated classical code and the quantum solution, except for a few marginal details. Certainly not mainly as Albenzo also agrees - that through the quantum solution it is possible to influence from a distance in zero time, which is the subject of the discussion.

    Bad wording by Albenzo, not my lie. If, as he said, "Your reading comprehension is flawed." When they say "there is no difference between Moshe and Danny except that Moshe is blond and Danny is red", does that mean that Moshe and Danny are the same? When you meet them, will you be able to tell them apart or not? " But if you forgot to mention that the Ginghams are known for their ability to influence from a distance in zero time, which is the subject of the discussion, then you understood your so-called Doctor Luca. It's like saying there is no difference between the movie Titanic from 1953 and the movie Titanic from 1997 except that in the movie from 1997 the heroes are blonde and redhead. These heroes - Jack and Rose - are almost the most important thing in the film.

    "Here is an excellent example of a crazy distortion of my words and even an invention: "Are you claiming that in the subtext they actually meant the opposite and that there is no threat and that they are getting along great and that the authors of the article fabricated something imaginary to increase circulation?" You know very well that I never said anything like that. I did not write that they fabricated anything. I didn't write that they meant the opposite.'

    And here is what Albenzo wrote a few days before:

    "First of all, you have to remember that every article in a magazine - even a scientific one, and certainly in Scientific American, which addresses the general public and has a huge interest in selling copies - can fall into a natural tendency for sensational or bombastic headlines."

    Not very similar?

    "You have lied many times, because most of the time your arguments are empty of content, because of your strong tendency to talk about things you have no idea about. Just like what's happening now, when you write more and more things about information when it's clear to everyone - including you - that you simply don't know anything about information theory, including the most basic definitions of what information is or what is beyond information.'

    Did I ever claim to know information theory? Or did I even know that information theory existed before Alessandro mentioned it? I was talking about information like everyone else here is talking about it.

    And since reviewers and biologists and many others also use the word information without giving its exact definition, I don't feel particularly unusual, or maybe we are all actually shameless liars.

    Bottom line, a person can say a sentence that is not true and that does not make them a liar. A liar is one who, under normative conditions, fails a lie machine. Albenzo said a sentence that is not true - "That information does not pass between interwoven particles is not knowledge. There is a mathematical proof.' Does that make him a liar? I don't think so, because that's what he believes, or at least believed at the time.

    And there's also the article in Scientific, which says exactly what I always say, so if I'm a liar, they probably are too.

    In short - neither a liar nor a liar. I'm not looking for fights, and I hope this closes the issue once and for all.

  4. Israel
    This is exactly what I am saying - there is no connection between simplicity and justice. What I'm trying to say is that Newton's Torah is not always simple either.

  5. We had many as if in a kibbutz. Also square chickens and triple lambs.

    The theory of relativity is actually very beautiful and the quants are not, so who is more correct?

  6. Israel
    I have written. Newton's physics did not know how to calculate the orbit of 2 planets.
    I can't understand what's good, the theory is simple, logical and wrong...

  7. Israel
    Should physical theory be simple? We don't know how to calculate the paths of 2 planets and the sun.... What's simpler than that?

  8. I said that there is no direct evidence for the shortening of the length, and this is in contrast to the direct evidence that exists for the lengthening of time.

    Actually, I didn't say that - Vicky says.

    "And as you said, "root of probability" is something strange."

    I said a square and it's not strange, that's how you find probability.

    "The bug I'm trying to solve now is strange - that doesn't mean it doesn't exist."

    It is indeed strange, even your explanation of Alpha Country. I didn't say it doesn't exist, but it is desirable that a physical theory be clear and simple, see Newton's and Maxwell's theories.

    "It does mean that "unuseful information" can move at infinite speed."

    Our role in power is to make it useful. Want - join. No - we will manage on our own.

    Going to Santa Barbara.

  9. Israel
    We started with the fact that you claimed that one of the predictions of special relativity - the contraction of length - is strange, and not experimentally proven.
    I showed you some phenomena that we do not know how to explain, without the length contraction. The saying "it's weird" is not something I know how to deal with. The bug I'm trying to solve now is strange - that doesn't mean it doesn't exist.

    You don't agree with what Einstein claims about special relativity, but you do agree with his opinion when it's convenient for you. It seems to me that his understanding of the meaning of EPR is wrong. Probabilities can change infinitely rapidly. There is a certain probability that my keys are on the table - because I sometimes put them there and sometimes in a drawer. I checked - and here they are in the drawer. At 0 time, the probability of them being on the table dropped to 0 🙂

    I know this is not the whole explanation, and as you said, "probability root" is something strange. But, it does mean that "useless information" can travel at infinite speed.

  10. by me? Where, in the warehouse? Why who am I, Rami Levy?

    But yes, it has an independent existence even without the relativity, and even from before 1905.

    https://en.wikipedia.org/wiki/Olinto_De_Pretto

    I'm not sure we're in sync about our role in the force.

    It is not our job to explain everything. We don't have the tools, the training, the experience, the time, or the talent for that.

    Our problem is much simpler: Einstein writes a paper - EPR - in which he claims that quantum regret is due to the fact that particles affect each other faster than light. He offers a thought experiment to prove his point.

    A variation of the experiment is performed and Quantum wins.

    According to quants, a photon has no location before measurement and is spread with equal probability in the universe.

    Nice asks:

    "Do you accept that I can emit a photon from point A and after a suitable time receive it at B?"

    Israel answers:

    "Without a doubt. You can emit a photon from point A and after a suitable time receive it at B.

    But if it is spread in the space before, and with equal probability, then there are two possibilities:

    1. The reality of the photon beyond B is not something physical - it is only the wave function or philosophy or zebra virology, but not the photon itself.

    (A bit strange if we consider that the square of the wave function is the probability of finding the particle at a certain point).

    2. The particle itself is also beyond B.

    In case 1, then there are no realities for a photon that moves faster than c, Einstein is right and Quantum regrets. But to my understanding, this is not the case.

    In case 2, if the photon itself is beyond B in a time less than ct, then there is no escaping the conclusion that it moves faster than c and in fact at any speed.'

    Our job is relatively simple: to discover the part of the photon that is beyond B.

    Capish?

  11. Israel
    Lover of simplicity? What is the simple explanation for muons? Why when I put a clock on a tall building does it run faster? Why does light bend near the sun? Why does mass increase with speed?

  12. Of course, it doesn't hurt the Kozily, but an effect on the past? Didn't we say that Eli Phat - by?

    Don't you see that it just gets more complicated, more complicated, more complicated and more complicated?

    And as a lover of simplicity, don't you see that a photon moving at all speeds solves most problems including affecting the past?

  13. Israel
    Either it has passed in 0 time or the particle is in two places. What has passed, if it has passed, is not a particle, and not at all within the scope of the theory of relativity.

  14. https://www.hayadan.org.il/quantun-philospy-part-b-07121

    And this something that is common to the particles doesn't pass from one to the other in zero time? After all, he started with the particle we measured and immediately moved to the second one.

    Feynman already said that no one understands quanta, but at least in this matter of immediate influence at a distance it has been proven to be correct and this is in contrast (at least according to Wiki) to relativity.

  15. Israel
    There is something in common between the particles. But, where is the influence of the future on the past?

    I don't understand how something "simple" like the two slots experiment works out for you...

  16. There is also the simple explanation of magnetism that appears in the link I provided.

    And regarding alpha - at a normal zoom at some point most of the stars will disappear from the picture, but this cannot happen here.

    By the way, if you want, I am a subscriber of an online telescope that you can see live every star and galaxy of your choice. I can give you the details, that's pretty nice.

    But I still don't understand: you don't accept that in the interweaving particle A affects particle B in 0 time? Even if we don't call it the transfer of information, even though somehow it arranges particle B in the same quantum state as its own, you don't see how much it challenges the relativity according to which nothing can move faster than light (let's leave tachyons for now).

    And don't you see that it leads to an influence on the past from the future? You don't believe that Eli Phat - by?

  17. Israel
    Here is another explanation:
    https://readingfeynman.org/2015/09/06/magnetism-and-relativity/

    I learned it a long time ago, from Berkeley's books. I don't have the books with me….

    About the camera. I don't understand how your idea gives different results. Let's say I'm approaching the moon at a speed close to c. Suppose there is a transmitter that broadcasts the time every second. That's equivalent to a photon with a clock, right? Also according to your method, the moon and the photons move at close speeds. That is, I will receive the photon of the hour 12:00 very close to the lunar hour 12:00. What is my conclusion? that the moon is very close.
    That is - even in your approach there is a shrinking of distance...

  18. The problem is that according to the video you brought, only from the point of view of the chargers are crowded..

    And I didn't understand how the video camera on my iPhone knows that it is Alpha Country that I want to take a picture of and therefore magnifies only it and not some 1000 other stars in the vicinity..

  19. Israel
    Your point of view does not affect the value of the power. Think of the opposite case - imagine two streams of electrons in a vacuum. Why is there no magnetic attraction between them (unlike the current in wires)?

  20. Israel
    The camera knows nothing. It picks up photons that travel at the speed of light. If they were moving faster than the speed of light, we would get a contradiction.

    Let's put two parallel wires in a plane, in the north direction. Suppose that in one of them a constant current flows towards the north. That is, the electrons move south.

    1) There is no current in the second wire. In this case both the protons and the electrons see a negative charge in the first wire. The sum of the forces - 0.

    2) In the second wire there was also a current in the north direction. In both wires - the electrons see more protons, and this creates an attraction. And, the protons see more electrons, again attraction.

    3) Reverse current in the second wire. The electrons see more protons, but even more electrons, therefore repulsion. The protons see more electrons and this creates a certain attraction. Overall, there is more repulsion, so the wires repel each other.

    It should be emphasized that this does not mean that there are no magnetic fields. Both field types are real, but there is reference system dependency.

  21. To me it is strange.

    How does the camera know that I want this particular star in the picture that is 4 light years away from me? Why not the one next to it?

    And what does it matter to the force from the charges in the conducting wire that I move relative to them? OK, from my point of view they are huddled together, but my point of view is supposed to affect physical strength?

    I did not check, but it is checked that if for example the direction of movement of the charge or the direction of the current are reversed then the force will also be reversed. Does it continue to be rejected?

  22. Israel
    I guess you're right about that. Just note that this means that under "normal" conditions, special relativity will remain true (like Newton's theory is true under certain conditions).

    By the way - I don't understand what is strange about the distortion of the picture. It is very logical to think that if you move close to the speed of light then you will see strange things. It's the same for voice. Get frustrated with a slow ship's waves, which look like a ripple from a thrown stone, compared to a fast ship making a V on the water. The same goes for fighter planes - the eyes see very strange things in supersonic air, such as diamonds in the exhaust gases, or fracture lines in the air near the cones. Supersonic flight creates a strange boom, and particles moving above the speed of light (in the same medium) create strange radiation. Weird doesn't mean something doesn't exist.

  23. There is no claim of non-contradiction here, and as written in Scientific, the quantum threat to relativity arose many years after Dirac.

    Note also that what I am saying is that relativity will probably have to be revised and become a special case of a more general theory that will include non-locality as happened to Newton's theory which is a special case of relativity.

  24. Israel
    I will quote again -

    In fact, QM would make grossly inaccurate predictions if Dirac hadn't shown up and tied QM together with special relativity to create "relativistic QM".

    General relativity, on the other hand

    It's weird, but that doesn't mean it's not true. Think you are approaching a large poster. The content of the image does not change, but it is very distorted. It's not much different from high-speed photography.

  25. "The article claims that there is no contradiction between special relativity and quantum theory, but the opposite."

    Where, where does he claim it?

    So who has a bigger article?'

    to Scientific. About 30 times.

    "You will see all the stars you have seen before, and you will not see a single star you have not seen before."

    But earlier you said that if gamma equals 10 then the moon will occupy 10 times the image..

    So if gamma is worth a million it won't fill the whole picture?

    What is it, all the stars will shrink to the corner of the picture?

    As I said, very strange.

  26. Israel
    I already said - the article emphasizes that the problem is in general relativity and not special relativity. If you insist on reading every sentence assuming that what you believe is true, then there is no point in discussing it.

  27. Israel
    Reporter:
    "And let's say you photograph the moon when gamma is worth a million.
    What will you see in the image with this relative zoom? Just the mouth of Tycho?
    So where did all the other stars in the background go? After all, photons from them reach the camera lens as well, so where did they go?"

    You will see all the stars you have seen before, and you will not see any stars you have not seen before.

  28. Just the title? I read the entire first paragraph:

    Physicist: Quantum Mechanics (QM) and relativity are both 100% accurate, as far as we have been able to measure (and our measurements are really, really good). The incompatibility shows up when both QM effects and relativistic effects are large enough to be detected and then disagree. This condition is strictly theoretical today, but in the next few years our observations of Sagittarius A*, and at CERN should bring the problems between QM and relativity into sharp focus.

    Especially the last sentence:

    should bring the problems between QM and relativity into sharp focus.

    Who is this physicist by the way?

    Read the Scientific article again, it is much less confused than this article. He explains that at the time they really thought that there was no longer a problem between relativity and quanta, but in recent years (ten years after Dirac) the recognition that there is probably a problem is getting stronger among philosophers and physicists.

  29. Israel
    What will? You caught one word in the title of the article, and you hold onto it like a bulldog...

    I should probably translate the passage I brought for you. Well - it is written in the passage that there is no contradiction between special relativity and quantum theory. It says that on the contrary! Paul Dirac showed that special relativity is required to explain quantum phenomena.

    Where is there a problem? if general relativity. One example - loss of information in a black hole (Suskind-Hawking war). A second example is what Albanzo explained - the singularity problems.

    Israel, really, will relate to what is written. Don't get caught up in every word because you think it reinforces your opinion.

  30. Israel
    Of course I read. a quote:
    In fact, QM would make grossly inaccurate predictions if Dirac hadn't shown up and tied QM together with special relativity to create "relativistic QM".

    General relativity, on the other hand,

  31. Well miracles really, you have to pay attention to what I write before you respond, otherwise it sounds a bit challenged..

    "Now are you saying there is a contradiction? I thought we agreed that section 1 was just weird..."

    Section 1 - contraction - strange.

    Section 5 - Quantums and non-locality - a so-called contradiction.

    Capish?

  32. Miracles

    One more time gloog gloog?

    Only about section 1 that you mentioned - the shortening of the length - I said strange, but without an internal contradiction.

    There were several other sections including the section you keep ignoring: the article in Scientific, which for some reason says exactly what I am.

    All I did was show the root of the so-called contradiction, and this time without interweaving and the speed of light.

  33. Israel
    Where did you talk nonsense? Where not? ?

    In conclusion, all you have to say is "that's weird". And no matter what they tell you, including experiments that amazingly confirm the theory of relativity.

    I understand that you are trying to come up with a hypothesis that fits both relativistic and quantum observations. You are not the only one - Shmulik described Penrose's hypothesis, and Albenzo mentioned the subject of strings.

    The problem is - that what you claim against the theory of relativity is, in my opinion, incorrect. You give a delusional thought experiment, and you are surprised that the experiment shows something delusional. You claim that there is no evidence for the length contraction, and ignore a number of evidences for it, one of which is very strong evidence (from ions from space). And there is more.

  34. Miracles, you must get together.

    Go to all comments - where did I say you were wrong? You are the one who said that what I say is wrong but when you understood you agreed that the star would look big and close.

    Surely quantum is strange, he asked Einstein, he fought it all his life - and lost.

  35. Israel
    Yeah, weird. Quantum theory is much weirder in my opinion.

    But, you still haven't explained to me why I'm wrong in my first section.

  36. Who said there is no solution? The simultaneous relativity, the symbolic relativity, epinyism, Ashwedishness, please excuse me - I was just pointing out that it is very strange.

  37. Israel
    Let's assume the moon is 90 degrees to the side. Alpha Centauri will look huge. The moon will appear very narrow (because it will appear contracted only on axis 1).
    As I described before - draw it on a piece of rubber, then stretch/shrink as much as you want.

    You won't come up with anything unreasonable.

  38. And let's say you photograph the moon when gamma is worth a million.

    What will you see in the image with this relative zoom? Just the mouth of Tycho?

    So where did all the other stars in the background go? After all, photons from them reach the camera lens as well, so where did they go?

    ??

  39. But why 10 times? After all, I wrote:

    the star Alpha centauri (4 light years away) fast enough so that gamma factor:

    is equal to 1000,000, and you take a picture of the star from earth. Since length contraction is only in the direction of your movement and not in the perpendicular coordinates, what will your picture show? — a huge star closer than the moon?

    Doesn't it say million?

  40. negative.

    I'm photographing the moon now from the ground, it fills a hundredth of the picture.

    What will be its size if it is photographed with the same camera from a spaceship passing over the earth with gamma equal to 10? How much of the picture will it fill?

  41. Israel
    Both the star and the moon are approaching. Imagine you are drawing a solar eclipse on a rubber pad. You draw the cone that shows the concealment. Now - stretch the rubber evenly. No matter how you do it, straight lines will stay straight.

    Effinity is easy to prove - take 3 points on a straight line and perform the transformation. but ……. Here is a quote from Wikipedia (https://en.wikipedia.org/wiki/Length_contraction)

    In special relativity, Poincaré transformations are a class of affine transformations which can be characterized as the transformations between alternative Cartesian coordinate charts on Minkowski spacetime corresponding to alternative states of inertial motion (and different choices of an origin). Lorentz transformations are Poincaré transformations which are linear transformations (preserve the origin)

    That is - not only affine, but also linear.

    Can you summarize point 1?

  42. So let's say the transformation is affine. Is the star closer or not?

    Because as a general rule when an object is closer then it appears in a larger photograph. Do you agree with that? So why if all Euclidean spaces are affine would it be different in this case?

    So you're claiming that in this case because of the specificity of the situation it won't look bigger? And don't you think it's very strange? And why does Einstein not mention the affinities?

    Can you provide a link between the shortening of the length and epiphytes?

  43. Israel
    So that's it, no. I've already explained why several times, so I'll try again, in a different way. Let's assume that the movement is along the x-axis, at a speed gamma = 2. Here we have a transformation:

    x => x/2
    y => y
    z => z

    This transformation is affine. This means that there are several things going on here. For example - parallel lines remain parallel. For example - planar points remain diagonal. For example - and this is what is important to us - linear points remain linear.

    I gave the story of the spacecraft and the solar eclipse. In the case of an eclipse, if we look from the point of view of the observer of the eclipse, then every point on the circumference of the moon hides a dead spot on the sun. That is - 3 points on a straight line.
    Therefore - when the spacecraft is near the viewer, there will still be an eclipse.

    QED

    So, is your claim that the Lorentz transform is not eigenvalue?

    Israel - pay attention. I am not claiming that there is a shrinking distance. I do claim that what you said about the star being bigger than the moon is wrong.

  44. Miracles

    The madman you say as you mad the Egyptian?

    what is wrong When you move in the direction of the star, isn't the distance to it shorter for you? And if the distance is shorter, isn't the star closer? And isn't a nearby object visible in a larger photograph? Want to try with the building across the street?

  45. Oh miracles one last time, if you understand - you will understand. If not then no.

    There are no contradictions in Proper's relationships - but her conclusions are extremely strange and the shortening of length is perhaps the strangest of them all.

    If you move fast towards a star then the distance to it shortens, the width does not.

    Therefore, optically, what is closer appears larger, and on its surface we see the star larger.

    is it so It is possible. maybe not. What does it matter? It is extremely strange, unlike the Newtonian Maxwell Boltzmann and Bernoulli theories which are not strange. That's the whole point at this particular point. This is not a contradiction like the quanta, just an oddity.

    Capish?

  46. Israel
    I'm really sorry, but I didn't understand. Are you claiming that according to special relativity then the star should look bigger than the moon?

  47. Miracles

    Try to remember from time to time that there is a connection between words and reality..

    "Now you say you didn't write it??"

    Of course I wrote it. Otherwise, where did you copy paste from, not where I wrote?

    And I wrote:

    what will your picture show? — a huge star closer than the moon?”

    This is a question, and it shows what I am claiming - it is very strange.

    If you think the explanation you gave "the transformation is affine. In particular, it means that three points that are on a straight line before the transformation will remain on a straight line after' is correct, so it is not the only explanation. The explanation you gave about the charges that the distance between them shortens which causes a force on a charged body in motion is also not complete, and this also appears in the link I provided.

    Working.

  48. Israel
    You wrote
    "Imagine that you move towards the star Alpha centauri (4 light years away) fast enough so that gamma factor:

    is equal to 1000,000, and you take a picture of the star from earth. Since length contraction is only in the direction of your movement and not in the perpendicular coordinates, what will your picture show? — a huge star closer than the moon?”

    Now you say you didn't write it??

  49. No.

    If that were true, we would get a contradiction in the relationship itself, wouldn't we?

    Do you think Einstein would have missed a contradiction in proper relations?

    And maybe you will already answer the question about the article? How come we both say the same thing?

  50. Israel
    What I am saying is the following:
    Let's assume there is a solar eclipse. A spaceship approaches at close speed and passes by Earth, as it flies towards the eclipse. I claim that the spacecraft will also see the eclipse at the moment of the passage near the Earth. From what I understand - you claim otherwise.

    Did I get you right?

  51. Let's really work in order, and try to see the whole picture.

    So you write:

    1) You write that a distant star should appear larger than the moon. This is simply not true. The transformation is affine. In particular, this means that three points that are on a straight line before the transformation will remain on a straight line after.

    And from this it can be concluded that I claim that there is a problem with the shortening of the length, that's what you meant, right?

    So here is what I did write:

    Even though there are nomathematical inconsistencies within the theory, physically relativity is so strange that many physicists found it very difficult to accept. Theoretical physicists such as Abraham, Lorentz, Poincaré, and Langevin still believed in the existence of an ether.
    For example, Lorenz length contraction is especially difficult to understand and has no direct experimental support; http://math.ucr.edu/home/baez/physics/Relativity/SR/experiments.html#Length_Contraction
    Tests of length contraction
    At this time there are no direct tests of length contraction.

    Translation: Although there are no mathematical contradictions in the theory of relativity itself, it is very strange physically.

    Then I move on to talk about the shortening of the length, mention the fact (from Wikipedia) that it has no direct experimental evidence, and give the example where you move in front of alpha country at speed and ask if it will appear larger than the moon.

    Am I claiming here that the length shortening does not exist? Of course not, after all I said before that there are no contradictions in relationships per se, the problem is only the confrontation with other theories.

    The shortening of length was brought up only to show that relativity leads to conclusions which on the face of it are strange and seem very difficult to digest. See how much we got into trouble with the twin paradox and how much 4 professors got into trouble with it and each one finally came up with an opposite explanation from his friend.

  52. Nice, stop your horses..

    Maybe instead of shouting "You don't understand, you don't understand!" and saying that Israel understands better than Einstein, there is also the possibility that Nissim doesn't fully understand what Israel is saying? 🙂

    Where did you talk nonsense? Where not? 🙁

    From the opening of the lecture:

    At the end of the presentation you will realize I believe, that if Einstein was wrong at all, it was a very reasonable mistake that he made.

    I also must add that chances are that he was not wrong at all, except maybe on the subject of illocality, and I simply don't understand the issue well enough.

    It will help if you answer the question I asked earlier:

    "So maybe you, the national explainer, will explain to me how it is that an article that says exactly what I say is published in Scientific?" Coincidence? A discounted case? magic?"

    I'm not asking if it's true or not, or if you agree or not, just how did it happen that we both say the exact same thing? I didn't read the article until a few years ago and I have the feeling that they also didn't read what I was saying carefully..

    So how did it turn out?

  53. Israel
    1) You write that a distant star should appear larger than the moon. This is simply not true. The transformation is affine. In particular, this means that three points that are on a straight line before the transformation will remain on a straight line after.

    2) You write that there is no evidence for the shortening of the distance. This is also not true. Wikipedia has a description of 5 cases that have evidence for this. Obviously, there is no "direct" measurement - today we have no way to accelerate measuring bodies to relative speeds.

    3) Maxwell's calculations are correct because he uses two seemingly independent constants. In the theory of relativity it is shown that one of them can be calculated from the other.

    4) You write that the assumption that the laws of physics are the same in every inertial system is incorrect because of the cosmic background radiation. I don't understand the contradiction here. You can always choose a third system as a reference - how does this affect the basic assumptions of the theory of relativity? I have a wristwatch - so that's why the whole Ferotian theory of relativity is incorrect? seriously? 🙂

    5) You really fail to understand the twin "paradox". Please - try to understand what I am describing now. I am on the earth and my brother is on the moon - we both have coordinated watches. A spacecraft at a speed of gamma = 10 passes the earth and then the moon. Let's say the distance to the moon is 10 light seconds.
    1) The spacecraft passes me in time and my brother in time t+10.
    2) The spacecraft will see time u when it crosses me, and time u+1 when it reaches the moon.

    In terms of temperature - the cosmic background radiation provides another reference system. We know that our speed relative to this radiation is 370 km per second, towards Leo. The spaceship will find that it is moving relative to the cosmic radiation at an enormous speed in a certain direction. That is, she will see a big deviation to the blue in the direction of her movement.

    6) Regarding the interweaving. The theory of relativity does not prohibit movement above the speed of light. She says that actual mass cannot be accelerated above the speed of light (because it increases to infinity), and of course there must not be contradictions (passing information that affects the past.

    And yes - I think we understand relationships today more than Einstein did. Albenzo gave the example of Darwin. What's the problem with that? You claim that Israel understands better than Einstein...

  54. Israel, what is your field of understanding in physics?
    From what I see, maybe some basic courses in classical physics and maybe one in the introduction to quantum mechanics.
    Are you researching the subject? Have you opened any book that is not a popular science book?
    In my humble opinion (based on threads for several years and also from sites that are not the science that you are also responding to)
    The answer is not unequivocal.
    Like you, I am amazed by the hiddenness of quantum and relativity, and do not understand why my intuition is wrong. The difference between us is that I assume my assumptions are wrong, and ask why my assumptions are wrong. It is much easier to learn something if you assume you are wrong, and try to find why you are wrong. But your perception is that your intuition is the one that is right and not the studies/theories that were developed in the last century and continue to develop today.

    In any case, I strongly advise you not to try to mix topics like relativity and quantum because even physicists who study these fields are still trying to find answers to the questions people like you ask. The difference is that these people do it for a living. They are experienced.

    The last paragraph is not intended to cool your enthusiasm and curiosity about physics, and certainly not to insult you in any way. Your curiosity is welcome, but in order to satisfy it, you will have to do a little more than asking people on the Internet questions about complex topics such as relativity and quantum that require a relatively deep foundation of mathematics and classical physics. Try starting with Berkeley Mechanics volume 1. The last chapters have in-depth coverage of special relativity. Continue from there to volumes 2 and 3. Trust me you won't be disappointed.

    Although I am still a student of physics, I hope that in the coming years this degree will become a bit more. But clearly, the way there is by asking questions about subjects beyond my understanding, but by a long and drawn-out journey.

  55. If you don't understand then it probably doesn't matter..

    The waiting was released. Luckily for you, WordPress has cut out all the formulas, illustrations and links so that it only takes you two hours to read it.

    Mountains, my heart.

  56. Israel
    Which experiment shows a mismatch? Leave me out of articles - ask the authors of the article if something is not clear. Only interesting experiments and calculations here.

    I don't understand why you are confusing the cosmological constant with our poor muons.

  57. The lengthening of time in muons:

    "The special theory of relativity claims that the clock of an observer moving at high speed advances at a relatively low rate compared to an observer at rest. This phenomenon is called relative time dilation, and it can be demonstrated with the help of particles called ions.'

    "The cosmological constant was introduced by Albert Einstein shortly after the formulation of the theory of general relativity in order to enable its solution that describes a static universe."

    Which shows that a wrong explanation is given to justify an existing theory and it does not mean that the theory is wrong. That's the point - and I'm not saying that Aka may not be the correct explanation for the lengthening of time, but I don't need to explain it. My job is just to point out the inconsistency of relativity with the quanta.

    Which reminds me - you still haven't answered the question:

    "So maybe you, the national explainer, will explain to me how it is that an article that says exactly what I say is published in Scientific?" Coincidence? A discounted case? magic?"

    And as for the atmosphere - the equator rotates faster than the poles. Does the air in the upper layers of the atmosphere also rotate faster than the air in the upper layers above the poles?

    Good night.

  58. Israel
    No - this is the explanation from the side of a ground observer. Regarding the muon - the explanation is the shrinking of the distance. For him - his life span is 2 microns, and the thickness of the atmosphere is 600 meters (not 6000).

    What does the cosmological constant belong to? It has nothing to do with special relativity.

    Basically, the atmosphere rotates with the Earth. When you shoot a rocket into space, you see its smoke rising very high. I saw several launches in Florida and California, and in all of them the smoke stayed about the same.
    Just don't forget that at high altitude, there isn't much atmosphere.

    In addition - the air at altitude affects the orbits of low satellites. It seems that at the height of satellites the atmosphere moves very fast because of the height, but... Satellites move much faster. Think of the grief. The telescope is at an altitude of 600 km, which means that the atmosphere moves 10% more than at a low altitude. On the other hand, Hubble circles the Earth in an hour and a half.

  59. Miracles

    The explanation of time slowing down is also the explanation for the long life of the muons which is also extreme in relations.

    Einstein brought the cosmological constant to explain the discrepancy with his model and later complained that it was the biggest mistake of his life. Does this mean that an explanation does not exist? (big bang).

    And since ions and the atmosphere were mentioned: does anyone know if the exosphere (the outer part of the atmosphere) rotates together with the earth? Do the gases in the exosphere above Antarctica rotate more slowly in terms of muons than those above the equator?

  60. Israel
    Your explanation is no better than "why - like this". To me, he is much less good. It does not explain why light bends near mass, it does not explain why mass increases with speed, and it does not explain why gravity affects time.

    He also does not explain why the muons manage to penetrate the atmosphere. What do you mean time slows down? Why won't the milk spoil quickly??

  61. "Earth in motion relative to the background radiation"

    negligible Muons move at almost c, as the sun moves at a few hundreds of kilometers/year relative to radiation.

    "What is background radiation? How many photons per cmXNUMX? What about the atmosphere itself?'

    Irrelevant. You asked for an alternative explanation - you got it. Do you now want another two weeks of irrelevant explanations for the discussion of the supposed contradiction between relativity and quantum?

    'The bigger problem is that it doesn't explain anything. What is background radiation? How many photons per cmXNUMX? What about the atmosphere itself?
    And why would time get longer because of stress?'

    I didn't understand the relevance. We are trying to see what the supposed contradiction between relativity and quantum is. You ask about sightings, and I'm trying to show you that there could be an alternative explanation and point out that it's not necessarily true, so why should I go into the details? And what exactly am I trying to sell? corn?

    Olbers points to the Olbers Paradox to show that the darkness of the night sky contradicts the assumption of an infinite and eternal static universe. He fails to come up with an explanation (the correct explanation: the expansion of the universe). So does the fact that he did not bring the explanation mean that the paradox does not exist?

    I am trying to show that relativity does not work out with the quanta (and a few other things, waiting for my father to release the waiting), does this mean that I have an alternative explanation for every phenomenon?

    So maybe you, the national explainer, will explain to me how it is that an article that says exactly what I say is published in Scientific? Coincidence? A discounted case? magic?

  62. Israel
    In my opinion yes. For several reasons.
    The Earth is in motion relative to the background radiation, so we should have seen a direction dependence. The earth moves at several hundreds of kilometers per second, which is not negligible. Therefore, I would expect a dependence of muon absorption and time.

    The bigger problem is that it explains nothing. What is background radiation? How many photons per cmXNUMX? What about the atmosphere itself?
    And why will time get longer because of stress? Because it fits what you sell? 🙂

  63. What's worse?

    Here is the problem with the muons as I understand it: their short life span does not allow them to travel the distance from the place of their formation to the earth before disintegrating, nevertheless they manage to reach the earth. How does this happen without the lengthening of times/shortening of length. I understand it right?

    And here is the explanation I gave you: because of the acceleration and the movement against the background radiation, the muon's internal clock is in the loop and therefore it slows down, just as the processes of decay are slowed down by refrigeration and therefore the lifespan of an apple in the refrigerator is longer than that of an apple in the refrigerator. An explanation not necessarily correct, but bad?

  64. Israel
    I got a very bad explanation. Muyons are picked up in both Los Angeles and Sydney. how does it work out

    What the theory of relativity says also comes out in the experiment: the lengthening of time, the shortening of distance, the increase of mass, the bending of light, gravitational waves, absorption of muons from cosmic radiation, gravitational repulsion and so on.

  65. Miracles

    "Relativity is simple, both special and general. Quantum theory is terribly complicated. Why prefer the complicated one?'

    We've been there before, remember? 1935, EPR?

    Because that's what comes out in the experiment.

  66. Miracles

    Akka - not aggrieved. Physics - not psychology.

    A muon sees nothing, and has a preferred reference system. You asked for an explanation - you got it.

    Albanzo

    So what do you say, according to the original article of relativity and only according to it - does the contradiction with the quantums that I pointed to exist or not? Remember that you can't talk about approximations of the uncertainty principle because the Lorentz transformations derived from relativity are not approximate.

    Besides, if you want to speak as a teacher to a student, you will have to answer a few questions before you acquire authority.

  67. Well, what will?

    https://www.hayadan.org.il/researchers-discover-the-most-distant-supermassive-black-hole-07121704/comment-page-6/#comment-723358

    Especially the last paragraph.

    In the Hadva course, one of the first things you learn (usually the first) is the definition of the border. Continuity is defined with the help of the limit, and from there to the derivatives. But if you try to learn Hadwa only from Newton's original writings, then you find that the concept of limit does not exist there at all. Newton did all the proofs geometrically and wonder and wonder - most of them are wrong. The conclusions are correct, the path is not and often contains an internal contradiction. Only 150 years after Newton found ways to rigorously prove all the principles and theorems of Hadwa. So what do you say - is there a problem with Hadva or not? If I limit myself only to the original writings (some are in Principia, some were published separately) I find sentences with false proofs. What does this mean about Hadwa?

    You can't learn private relativity just from one of Einstein's papers just like you can't learn evolution just from the origin of species. There is no need to know the path of a photon to formulate the special theory of relativity. The fact that Einstein did this (because he extended classical mechanics and in classical mechanics there is no reason to think that we do not know the trajectory of a photon) does not mean that there is a problem with private relations. It just means that there is a problem with a student who thinks he can learn the entire Torah from one article, and when it is explained to him that he can't - he ignores it.

  68. Israel
    If the muon's clock was slower (for him) then he would see the Earth approaching him at a speed much higher than c.

    The time change of GPS clocks is the result of several factors. One of them, the main one, is the lack of gravity at the height of the satellite. There is no speed issue here. The second element in the change of times is the relative speed, and we also measure that the deceleration is symmetrical, meaning there is no preferred reference system.
    It doesn't match the distress you describe.

  69. In the meantime, let's go.

    Muons are in stress - elk stress - which results from acceleration to high speed and movement against the background radiation. The contraction causes the internal clock of the muon to slow down.

    This is also the reason for slowing down the clocks of any accelerated system, such as GPS satellites.

    This is a possible explanation, but not a necessary one. The ideal solution is the integration of relativity as a special case in a theory that includes non-locality, as Newton's theory is a special case of relativity at low speeds and Maxwell's theory is the reduction of relativity.

  70. Miracles

    Why are you doing this to yourself?

    Hi every body and thank you for coming to my presentation "the problem with relativity".

    Maybe you are waiting for me to complete the sentence "was Einstein wrong?" This is like saying "was Newton wrong" or "was Maxwell wrong?" At the end of the presentation you will realize I believe, that if Einstein was wrong at all, it was a very reasonable mistake that he made.

    I also must add that chances are that he was not wrong at all, except maybe on the subject of illocality, and I simply don't understand the issue well enough. One of the reasons I am making this presentation today is that perhaps one of you will clarify the issues I am about to raise today so I will understand them better. I raised all those subjects in many forums and was in contact with numerous experts, including physics professors, but none of their answers ended my wonderings. I also invest a huge amount of time - and also money - to try to find an answer to those quarries through research and all kinds of experiments.

    I would like to divide the presentation into 4 parts:

    1. the way physics looked until the end of the 19th century.

    2. explaining the reason why Einstein came up with special relativity including a brief explanation of what relativity is and why we have time dilation according to relativity.

    3. The problems I see with relativity.

    4. open discussion for the group.

    The subject is vast, and the time is short, so naturally we won't be able to elaborate too much on each part. never the less, I would still like to tell in chronological order the history of the scientists involved in the road to relativity and hope you'll enjoy the story and also the physics..

    So let's start with part one - A brief history of relativity.

    I'll start with a description of the way physics looked like before Einstein's miracle year of 1905.

    Newton's law of mechanics gave an excellent description of the way physical objects react to forces including all the mathematical formulation. At the end of the 18th century and the beginning of the 19th century, the laws of optics, electricity and magnetism were beginning to emerge. Michael Faraday showed that electricity effects magnetism and vice versa. Faraday btw, had never had a mathematical education, he hardly even knew algebra, but never the less he was a great experimenter, who showed the connection between electricity and magnetism.

    The year is 1861 - civil war is raging in America - and in Scotland a physicist named James Clerk Maxwell publishes a paper named: "on physical line of force".

    Many of you probably heard of Maxwell, let's just say that along with Newton and Einstein, Maxwell is considered one of the greatest physicists of all times.

    Maxwell tried to mathematically and physically formulate Michael Faraday's equations of electromagnetism. he called it:

    The Theory of Molecular Vortices applied to Magnetic Phenomena.
    Probably most of you are familiar with the phenomena that if you spread iron dust (or fillings) next to a magnet, you can see the dust ordering itself on the lines of the magnetic field. also, if we move a magnet next to a cooper wire, it creates an electric current in the wire, and if you run a current in this wire, it can move the needle of a compass.

    Maxwell, an expert on the subject of hydrodynamics, built an hydrodynamic model describing and explaining how those phenomena are occurring in the so-called ether. according to this model, the ether is somewhat like a sea, and in this sea there are currents, undulations, pressures, vortices, and like in any sea, there are also waves.

    In his model, Maxwell showed that those line of forces that we see when we spread the iron dust next to a magnet, are actually currents that flow from the north pole of the magnet to the south pole, and they create another current in the " sea", which is perpendicular to the magnetic current and can be viewed as an electric current. This electric current in its turn creates a 90 degrees magnetic current, which again creates an electric current and so on, until the end of times.

    All of this back and forth force game of an electric field changing to a magnetic field and vice versa propagates in the sea as a transverse wave, a wave that Maxwell called an Electromagnetic wave.

    Maxwell then went to calculate the velocity of this wave.

    He used Newton's formula for sound waves in the air for the speed of the wave: the densityρ determine the speed of soundc (pressure waves), according to the Newton-Laplace formula:
    c=\sqrt{\frac{K}{\rho}}.
    and showed that if we substitute K in the formula (elastic modulus) and the Greek letter row (density) with the known coefficients constants for electricity and magnetism, we get the speed of propagation of the wave in this "ether' medium.

    When he checked to see what the value of this speed was, he found to his surprise that this is the speed of light as measured by Armand Fizeau in 1849 (see equation 136 in Maxwell's ether model):

    Maxwell then concluded that light is also an electromagnetic wave.
    people told me in the b past that Maxwell already knew that he would get the speed of light and only "drew the target around the arrow hitting point", but Maxwell himself denied it.
    4 years later, Maxwell used the same ether model to arrive at his famous differential equations which can be seen on t shirts students are wearing.

    Maxwell died in 1879 young, he was 48.

    Maxwell's model of electromagnetism was praised by the scientists of his time, but there were no experiments which could support his theory. In 1879, the year he died, the streets were mostly dark at night (the light bulb was invented by Edison in this year) and most of the transportation was handled by horses. So you expect people to believe some mysterious waves, unseen and untouched, moving at the speed of light? what other idea those crazy physicists will come up with, maybe that people would be able to drive their cars in America and use electromagnetic waves to talk live to their friends in India and see them too? real lunatics..

    Hermann von Helmholtz, the head of the physics department in Berlin, offered the BERLIN PRIZE to the person who will experimentally demonstrate Maxwell's predictions. The years went by and no one seemed to make good on the mission. Helmholtz was about to give up, but then, his brightest student, Heinrich Rudolf Hertz, succeeded in 1886 to transmit and receive what will be known as radio waves, and also to confirm that they are moving at the speed of light, are polarized and the rest of Maxwell's predictions.

    Of Hertz's demonstration of electric waves, Helmholtz told the Physical Society of Berlin: "Gentlemen! I have to communicate to you today the most important physical discovery of the 19th century.

    Hertz died in 1894 very young, he was 36.

    So here we are, almost at the end of the 19th century, and many believe that physics is almost complete and the only thing left is to improve the measurements to better accuracy. Thermodynamics, with its 2 powerful laws, the first and the second, was reduced to a branch of kinetics mechanics. English Newton gave us the laws of mechanics and gravity, Scottish Maxwell gave us the laws of Electromagnetism, and all we need to do now is to enjoy ourselves in the comfortable world those two British Gentlemen arranged for us and drink a cup of tea, like we do in Windsor.
    I have to admit I miss this age of innocence, when classical physics ruled, here was here, there was there, present was not past, and we had a single universe. Can you compare it with the mess we have today, with multiverse, electrons jump from one side of the universe to the other in no time and without even being in the middle, with measurements in the present affecting measurements in the past?

    In my grandfather's times, a wave was a wave and a particle was a particle.

    There was still an unresolved question: if, as Maxwell theorized and Hertz showed experimentally, electromagnetic waves propagate in the ether "sea" - then what is the rest frame of this sea? Every hydrodynamic medium has a rest frame. The oceans - the same as earth. the air - earth, or if there is a wind, the wind. The air in an airplane, the airplane.
    I just need to add that the belief in the ether at the end of the 19th century was like the belief in atoms today: almost unquestionable. that's right, no one saw or felt the ether, but no one saw or felt an atom or electron in our times too.

    So what is the rest frame of the ether?

    in 1887 Michelson and Morley performed an experiment to detect the "ether wind", and from it to deduce the rest frame of the ether relative to earth. they used an instrument called an interferometer, they used it many times in different seasons of the year, but to their dismay they found no rest frame for the ether. this was puzzling to them and most of the scientific community since as we said, almost every body believed in the ether.
    But the "Failure" of the Michelson–Morley experiment - they saw it as a failure, maybe some technical problem - and did it over and over again, along with the problem of black body radiation, led to the most creative and amazing developments in physics – Theory of relativity and quantum mechanics.

    The null result of the Michelson–Morley experiment in 1887 along with other considerations, led Albert Einstein to publish in 1905 his paper: ON THE ELECTRODYNAMICS OF MOVING BODIES.

    in which he introduced special relativity to the world.

    This concludes part 1 of the presentation, 19th century physics. If anyone has some questions or comments, this is the time before we move to the second part.

    Part 2: Relativity in a nut shell.

    I promised Julian not to over complicate the presentation too much, but I feel I must elaborate a little beat on the subject.

    Israel's first president Haim Weitzman once sailed with Einstein and Einstein explained to him the subject of general relativity. At the end of the voyage the reporters at the port asked Weizman "so now do you understand relativity?" Weizman replied "no, but I got the impression that Einstein does".
    Special relativity is actually not that complicated, that's why I would like to stay on the subject for some time, and if one person would feel he or she understands it better, I did my part. But Julian, if you think we are getting too detailed here, feel free to cut me so we can move on.

    In his theory, Einstein used 2 postulates:
    The laws of physics are the same for all observers in uniform motion relative to one another (principle of relativity).
    The speed of light in a vacuum is the same for all observers, regardless of their relative motion or of the motion of the light source.
    Einstein showed that the problem of the rest frame of the ether that Michelson and Morley failed to find, is solved if we ignore the ether assumption and replace the notion of fixed space and time in Newtonian mechanics with relative parameters while the only fixed thing is c , the speed of light.
    So let's see what this means:

    first postulate:
    The laws of physics are the same for all observers in uniform motion relative to one another (principle of relativity).

    This principle is known since Galileo's time, 300 years earlier. what it says is that if you have 2 bodies which are moving relative to each other and they are not accelerated, there is no way to tell which one is "really" at rest and which one is "really" moving. as a consequence, there is no experiment that can tell an inertial body if it is at rest or moving.

    Second postulate:
    The speed of light in a vacuum is the same for all observers, regardless of their relative motion or of the motion of the light source.
    To understand this statement, let us think of a long train standing on the railroad. if we shoot a bullet from a gun along the train and the bullet is moving at 1000 m/s, then the movement of the bullet is 1000 m/s relative to the gun which shot it, and the sound of the shot is 330 m/s s relative to the air, the medium carrying the sound. those are also the speeds of the bullet and the sound relative to the train.
    If the train will start moving after the shot in the direction of the shot, the bullet will now move slower relative to the train because the train is moving relative to the original location of the gun, and also the sound moves slower relative to the train because the train moves relative to the air, the medium carrying the sound.
    If we move at the speed of the bullet, we can catch it with our hand. If we move at the speed of sound, we'll make a supersonic boom. We can move faster than both, the bullet and the sound, and leave them behind us.

    we call the speed of the bullet an emission speed, since it is relative to the source that emitted it.
    But what about the speed of the light, the flash created by the shot? is it relative to the gun that emitted it?

    Einstein checked the so called "emission theory", but realized it cannot be real in the case of light. And as we saw, The null result of the Michelson-Morley experiment in 1887 showed that the speed of light is not relative to the ether medium.
    Einstein also imagined what would happen if he moved at the speed of light. will the light be at rest relative to him? would he be able to reach his hand and grab some light like if you can catch a bullet if you fly over it with the same speed like the bullet?
    Einstein realized the answer must be negative. It does not matter how fast you are moving, the light moves away from you at the same speed c, the speed of light. this is the second postulate of relativity.

    The subject of time and synchronization is a little complicated and I would not touch it unless there is a demand from the group. basically, how do we know that 2 clocks are moving at the same rate if they are in different places? why would they? I must tell you that ever since I started to learn the subject I check from time to time the clock in the other room to make sure it shows the same time as in my room.
    Never the less, Einstein 1st chapter in his relativity paper is dedicated to the subject of simultaneity - he suggests a way to make sure that any number of clocks in an inertial frame are synchronized, ie show the same time always regardless of location in the system .

    So let's go back to our train, and let's say that there are synchronized clocks on the train.

    The train is not moving, so it is an inertial system. for simplicity we have the railway in space, let say that the railway is the galactic railway which was built by the Vegans and cost the lives of many unlucky Andromedians slaves. it has two parallel lanes and is 100 light years long. a railway, you know, like the ones we have in la

    our train is 300,000 km long, the cars do not block light, and each car is 1 km long. and at the end there is a mirror. so if we turn a flash green light at one side of the train, it will take the light exactly 1 second to go to the other side and to hit the mirror and then to turn back, and another second to reach back to the point of origin.

    Notice that if we have a mirror on the first side too we are getting a clock, since we get a "tick" when a photon hits one side and "tack" on the other side, every second there is tick tack tick tack, just like a clock.
    So if the clocks in the train are synchronized as Einstein suggested, then at time 0 the light will leave one side of the train, will reach the other side at time 1 sec and will reach the first side at time 2 seconds. Everyone agree?
    If the train now moves on the railway, we will get the same results for any speed of the train. remember, in its own frame, the train is always stationary and the tracks are moving.
    Now let's say that on the parallel track standing an identical train, train 2 with a red light. Naturally the same will happen to train 2.

    Now what happens if one train is moving relative to the other, and at time 0 both trains ends coincide.
    At time 0 in both clocks the light leaves the end of the trains, let's say the green. We have a high resolution camera and it shows: the clock at train 1: 0, The clock at train 2: 0. Since both trains are inertial systems, it will reach the other side at time 1 second like before, and at time 2 seconds the other hand. remember, each train is a system by itself and the movement of the other has no effect on it. They are both standing, and the other train, or the railway itself, is moving relative to them.
    But when the light reaches the first side of train 2, train 2 had already advanced along train 1, lets say 1000 km, or 1000 train cars. The time in train 2 clock is 2 seconds, but what is the time in the adjacent clock from train 1 in car 1000?
    It cannot be 2 seconds because the light reaches the end of train one at time 2 seconds, and it still has 1000 km to go.. that's why the time in the adjacent clock is less than 2 seconds. If we take a picture – a snap shot – with a high resolution camera of the two clocks together at this point, from either train, the picture will show: the clock from train 2, 2 seconds, the clock from train 1 less than 2 second.
    As we remember, they both started at time 0 in each train, and we even have pictures to prove it.

    This is why we have time dilation in a moving object in a synchronized system. we can continue and show why we have length contraction, but let's skip it for now.

    Note that time dilation also happens if we do not use light at all - we used it only to demonstrate the point.
    Einstein then went to prove that nothing can move faster than light. I will spare you the proof, and say only this: any speed which fulfills the first postulate, ie is the same for all observers, regardless of their relative motion or of the motion of its source, is the upper limit for all speeds, does not matter if its the speed of light, the speed of sound or the speed of a hound.

    Later in this year in another paper, Einstein showed that mass and energy are equivalent and related by the now famous formula: E=mc^2. 10 years later in 1915 Einstein expanded his theory to include gravity, in what is known as General relativity.

    Here we end the first two parts of the presentation, The Physics before Einstein and the theory of special relativity in a nut shell. If anyone has comments or questions, this is the time before we move to part 3, CHALLENGES TO RELATIVITY.

    part 3: CHALLENGES TO RELATIVITY.

    1. Even though there are nomathematical inconsistencies within the theory, physically relativity is so strange that many physicists found it very difficult to accept. Theoretical physicists such as Abraham, Lorentz, Poincaré, and Langevin still believed in the existence of an ether.
    For example, Lorenz length contraction is especially difficult to understand and has no direct experimental support; http://math.ucr.edu/home/baez/physics/Relativity/SR/experiments.html#Length_Contraction
    Tests of length contraction
    At this time there are no direct tests of length contraction. Imagine that you move towards the star Alpha centauri (4 light years away) fast enough so that gamma factor:

    is equal to 1000,000, and you take a picture of the star from earth. Since length contraction is only in the direction of your movement and not in the perpendicular coordinates, what will your picture show? — a huge star closer than the moon?

    2. Inconsistencies with Quantum theory. Look at Sky Jason Shields' presentation, even though I missed June's lecture.
    From his presentation: Basically you can think of the division between relativity and quantum systems as "smooth" versus "chunky" or continuously interconnected versus discretely segmented.

    3. If Maxwell's Ether model is wrong - Maxwell's model is very complex and delicate - then how was he able to derive the speed of light from this wrong hydrodynamic model (equation 136 above) and his famous equations? Coincidence? Lucky guess? Magic?

    We have in Israel a popular radio fox hero by the name MAR NAHSON, which means Mr guest. He was interviewed on the radio and was asked how he succeeded in being the only one to guess all the results of the soccer games 4 weeks in a row.

    "Well", he explained, "this is very simple. What I do is count the marbles on the floor, multiply by the length of the walls, and divide by the numbers of the neighbors. By default, it must come with the correct answer.
    And he has another system for the summer: he takes the whole family to the beach when there are lots of waves at the sea and let them in, and then watch: if one comes out, he writes 1. if 2, 2. and if no one comes out – x.
    So maybe Maxwell's model is like Mr. Guester, and he was simply lucky?

    4. First postulate of relativity:
    The laws of physics are the same for all observers in uniform motion relative to one another (principle of relativity). This is inconsistent with the big bang theory and cosmic microwave background radiation.
    Look at that in the twin paradox when the young twin meets his older brother, the universe is colder at the time they meet according to Friedman's equation:
    http://hyperphysics.phy-astr.gsu.edu/hbase/Astro/expand.html#c3
    Thus, during his journey which takes shorter time than his brother, he observes a faster drop in the temperature than his brother, even though they are both in an inertial frame.
    Note that acceleration is not a solution in this case: If the older twin was moving relative to the cmbr, he was the one to observe the rapid temperature drop, and the young one would stay warm and cozy.

    5. In the EPR paradox (1935):
    https://en.wikipedia.org/wiki/EPR_paradox
    Einstein, Podolsky and Rosen describe a thought experiment to show why Quantum Mechanics theory is incomplete. According to the article in wikipedia:
    Here is the paradox summed up:
    It is one thing to say that physical measurement of the first particle's momentum affects uncertainty in its own position, but to say that measuring the first particle's momentum affects the uncertainty in the position of the other is another thing altogether. Einstein, Podolsky and Rosen asked how can the second particle "know" to have precisely defined momentum but uncertain position? Since this implies that one particle is communicating with the other instantaneously across space, ie, faster than light, this is the "paradox".

    A variation of the experiment was performed by Alain Aspect in the 1980's, and was found to be consistent with Quantum Mechanics and John Stewart Bell's theorem, and against Einstein's claim.
    Later attempts to save relativity from Aspect's experiment were based on the claim that what relativity really forbids is the transfer of information, And it's impossible to send information with a Bell experiment (ASPECT is a bell experiment) According to no-communication theorem:
    https://en.wikipedia.org/wiki/No-communication_theorem
    Well, the fact that it's impossible to send information by entanglement was known already in 1935. So why do we have the notion in EPR's wikipedia paper:
    "Since this implies that one particle is communicating with the other instantaneously across space"?
    Would Einstein embarrass himself in the face of the whole world in his most important paper since general relativity? Or maybe the maestro saw that transfer of information faster than light and thus to the past, does occur in contradiction to relativity in the case of entanglement?

    And if Einstein says that there is a contradiction between ilocality in quantum entanglement and relativity - can we tell Einstein he is wrong? Do we understand relativity better than Einstein?

    6. But what about all the unquestionable successes of relativity? GPS? Muons? and above all – E=mc^2?

    I can answer that in a question: What about all the unquestionable successes of Newton's theory? almost every technology we know is based on it and yet, Newton's theory is just a special case of relativity in low speeds.
    and E=mc^2? this is pure Einstein isn't it?

    From Wikipedia:

    Olinto De Pretto (26 April 1857 – 16 March 1921) was an Italian industrialist and geologist from Schio, Vicenza. There are claims[1] that De Pretto may have been the first person to derive the energy-mass-equivalence E=mc^{2}, generally attributed to Albert Einstein. He suggested that radioactive decay of uranium and thorium was an example of mass transforming into energy.

    From 1899 to 1903 De Pretto began to study the emerging field of nuclear physics and its relationship to astronomy. He focused on the theory of aether, a hypothetical substance that at that time was believed to fill all space.

    De Pretto used the expression mv^2 for the "vis viva" and the energy store within matter, where he identified v with the speed of light. His formula precedes by two years, and is in agreement with Albert Einstein's later formula E=mc^{2} for mass–energy equivalence, which was derived by Einstein as a consequence of special relativity.

    I raised those objections to relativity for many years, and not too many agree with me. But less than a year ago I found an article in SCIENTIFIC AMERICAN's 2009 paper: A QUANTUM THREAT TO SPECIAL RELATIVITY, which says exactly what I was saying for many years:

    "The status of special relativity, just more than a century after it was presented to the world, is suddenly a radically open and rapidly developing question. This situation has come about because physicists and philosophers have finally followed through on the loose ends of Einstein's long neglected argument with quantum mechanics".

  71. Israel
    Drop quantum theory for a moment. Ignore her completely.
    Is there now a reason not to accept the basic assumptions of the theory of relativity, and its conclusions?

    As an exercise, try to explain the story of muons without shortening length.

    Relativity is simple, both special and general. Quantum theory is terribly complicated. Why prefer the complicated one?

  72. Let's see what I'm trying to do here.

    I'm not trying to explain that the scene or the transfer of information - although I have some idea - we have already agreed that as long as you get the ability to influence from a distance in zero time there is no need to use the word information.

    What I am trying to do as part of the comments in this article is to show what is the basis for the supposed contradiction between quantum and relativity.

    I brought the Scientific American article. One can agree with what is said in it or not, but the thesis there is that non-locality in interweaving threatens the special theory of relativity, due to an influence from a distance faster than light.

    My claim does not include interweaving or an effect faster than light, but concerns a fundamental problem in the basic premise of relativity: that the photon is only at a certain point at a given moment. This hiding from what we know from quantum mechanics causes, in my opinion, all the other problems if they exist, as written in the article.

    I also brought a description of a photon that can solve the strangeness of relativity and also shed light on very strange things in quantum such as influence on the past.

    This description is not just arbitrary - it is a logical inference arising from Maxwell's ether model in an open and infinite system, and from what seems to me to be a fundamental error in the starting point of the Michelson-Morley experiment.

  73. Israel
    How does this explain weaving? How do a pair of photons transfer information between them? Do they send more photons between them? If so then a detector between them would detect these photons.

    All speeds including all directions? If not, how do you explain bypass?

  74. Without a doubt. You can emit a photon from point A and after a suitable time receive it at B.

    But if it is spread in the space before, and with equal probability, then there are two possibilities:

    1. The reality of the photon beyond B is not something physical - it is only the wave function or philosophy or zebra virology, but not the photon itself.

    (A bit strange if we consider that the square of the wave function is the probability of finding the particle at a certain point).

    2. The particle itself is also beyond B.

    In case 1, then there are no realities for a photon that moves faster than c, Einstein is right and Quantum regrets. But to my understanding, this is not the case.

    In case 2, if the photon itself is beyond B in a time less than ct, then there is no escaping the conclusion that it moves faster than c and in fact at any speed.

    Do I know what the solution is? Negative. But it seems to me that no one here knows. A doctor of physics agreed with me that the particle was also beyond B before the measurement.

    I proposed what seems reasonable to me and fits the conditions of the problem: the photon moves at all speeds like a rubber band that stretches, but our measuring devices can only pick up the part that moves relative to the measuring device at c. I also explained why - for example, the shells that pass through a tunnel in Israel and only those that move at a certain speed can be recorded. In this example, the part moving faster than c simply cannot be captured by the measuring device, just as a shell moving beyond the escape velocity will not be captured in a tunnel.

    good night and happy new year

  75. Israel
    I never said that. I say that as long as we haven't found it a billion light years away, we don't care.

    He leaves A and arrives at B. If you look for it in a straight line between A and B you will find it. If you look for him in Andromeda, you won't find him.
    This is the view as a particle. If you want to look at it as a wave applied in space, that's something else.

    Do you accept that I can emit a photon from point A and after a suitable time receive it at B?

  76. Alec philosophy..

    So you claim that only philosophically the photon can be a billion light years away after 9 seconds and not physically? that it has no physical existence a billion light years away?

    So why does this description appear in a physics book and not a philosophy book?

  77. Israel
    Does a tree falling in the forest with no one around make a noise?
    When no one is looking, can the photon be everywhere? Yes - as long as no one was looking at him. This is not the case for Einstein's photon. We will detect it at a distance of ct, always.
    Think of the two-slot experiment. We know when a particle leaves and when it arrives. On the way, the particle has properties of a wave - the wave first passes through narrow slits and therefore diffraction is created. At the edges - it is a particle for everything. In Einstein's experiment, the phenomenon will manifest itself in a certain dispersion of the photons in the collector. Even the best laser has dispersion of the beam.

  78. So do you believe like Einstein that if a photon left point A at time 0 then after time t it will be distant from it ct?

    So what is this whole story that a second ago he was a billion light years away from her?

    And what is the connection to measurement? This is a thought experiment in which no measurement is mentioned, but the calculation of the photon's position through thought.

    Or maybe the thought causes the focus of the photon at a certain point?

  79. Israel
    I do agree with that. I don't understand what bothers you about that.

    This does not contradict quantum theory. More than that - there are laboratory experiments with single photons, in which we know where individual photons are, and when.

    We know how to see many elementary particles, with the help of bubble cells for example. A Geiger counter also detects individual particles.

    True - the uncertainty principle states that certain pairs of data, such as momentum and position, cannot be known with unlimited precision. But, the accuracy is definitely high enough to confirm the special theory of relativity.

    They are... maybe because of the uncertainty principle, when we see a single photon then we don't know what its color is? (This is nonsense of course... there is another reason for this).

  80. I don't think my idea is as good as theirs or even as good. Only an experiment will determine.

    And Newton's theory does not provide accuracy. Amazing experience? And Ptolemy's theory?

    I didn't understand why you don't agree that relativity is built on the idea that a photon is at a certain moment in a certain location, which conflicts with the quanta.

  81. Israel
    There are many contradictions between them (black holes, the big bang, determinism versus randomness, alocality and so on). But - each in its field provides incredible experimental accuracy.

    There is a freshness to combine them, ideas of people who understand both. What makes you think your idea is better than theirs?

  82. Well, already.

    But this is more or less what I was told about the so-called contradiction between quantum and relativity, and then suddenly there is the article in Scientific American that says exactly what I am..

    It doesn't mean that they and I are right, but it sure means that someone understands me.

    So maybe in a few years there will be an article about what I'm saying now.

  83. Israel
    Maybe one of them will be able to understand what you are trying to say. It seems to me that you insist on not understanding something simple.

  84. Nice

    also in the eye. So what? Don't you see that this is a thought experiment?

    Well, if I haven't been able to get the point across to you by now, we'll wait for an answer from the blogs where I posed the question.

  85. Israel
    Technically - it is possible to transmit a single photon and receive it in a detector. This is done in laboratory experiments.

    I really don't understand what bothers you.

  86. Miracles

    You can read directly in Einstein's relativity paper from 1905, without interpretation.

    Let a ray of light depart from A at the time ta, let it be reflected at B at the time tb, and reach A again at the time t'a

    http://www.fourmilab.ch/etexts/einstein/specrel/www/

    The experiment is a thought experiment, and what it says is this: let the photon leave point A, reach point B and be returned to point A.

    Then he continues the development of relativity in the familiar way, but nowhere is it about measurement, only about a thought experiment and drawing logical conclusions from it.

    But this logic is based on what was clear from Elio in 1905, namely that a photon has a definite position at a given time and at a given moment and is distant ct from the source.

    And if this premise is not true, then what does that say about relativity?

    As the premise of not transferring the information in the interweaving is that "all the data is already in the system".

    But if this basic premise is violated, as in an aspect experiment where another factor is added to the system, namely the state of the polarizers that is not included in the initial data, then what does this say about the transfer of information?

    A bit of trivia: the experiment was named after Aspect even though almost identical experiments were conducted a decade earlier. The great thing about Aspect is that it managed to change the state of the polarizers after the photons have already left the source, thus avoiding the fear that the polarizers will "conspire a conspiracy" (so in the original!) and thus the data will already be included in the system.

  87. "The root of the dispute", Israel Shapira, is - in the interpretation. It is politics in science that creates controversy. Not the experiments.

  88. Israel
    No, I don't agree. The whole idea is definitely based on measurement. The lengthening of time, the shortening of length and the increase in mass are only measurement results.

    If you sit in a spaceship that accelerates at 1g for years, nothing will change for you. You don't agree with that? If someone in another system measures, he is the one who will get relative results.

    I don't understand the problem here. Einstein's experiment is performed every day by measuring muons, for example.

    Technically, a photon is a light particle...we see a single photon in the eye 🙂

  89. Miracles

    and?

    The whole idea of ​​time dilation (at least according to Einstein's article), is based on the fact that the photon is in a certain place at a certain moment, not related to measurement. agree?

    So are you claiming this to be true? Can a photon be at a given moment in a certain location? So what was he doing a billion light years away before?

    And what is related to what Einstein knew in 1905? Why if he didn't know then it didn't happen?

    And it should not be mentioned that Einstein himself fought with all his might for this description of the photon, and lost.

  90. Israel
    You created a photon in your flashlight. Conveyance concern in a certain direction. You cannot precisely determine the conditions of the formation of the photon, therefore it will have a certain dispersion.
    Think of a laser beam - the narrower the beam, the greater the dispersion (therefore, when measuring the distance to the moon, the pulse is sent through a telescope, which expands the lobe).
    Another example - radar. To improve angular discrimination, the antenna is increased. The magnification widens the lobe, but decreases the dispersion.

    Albanzo
    In this case (sending a photon from A to B), it is hard for me to understand that there is no information about the position along the way. If I put a disk with a hole (that can be blocked) in the middle, then only if the hole is open at the right time will we see the photon at B. If it is a charged particle, I can also verify that the particle passed through the hole.

  91. Try again this time.

    Does the photon have a definite position before the measurement?

    Isn't the relationship built on. So it has a distinct location?

    You read the article, didn't you? Contrary to what is said here, the name of the article is a quantum threat to the theory of relativity, and the statements there are clear and unequivocal. I'm just trying to show where the root of the dispute is, and that without interweaving and the speed of light.

  92. Albanzo

    Photons, electrons, speeds and forces. Physics.

    And not you, you didn't study, you didn't understand and you lied. psychology.

    What's more, as someone who writes there is no difference except, and after that it turns out that this exception is the main thing, I would think twice before lecturing someone on reading comprehension.

    Nissim, the national explainer.

    So how does it work out?

  93. Miracles,

    Successfully. I guess you were able to follow and understood my words. If you want to try your luck, I'll give you a hand.

    Israel,

    The answer to your question is found in what I wrote. If you do want to understand, all you need to do is spend a few minutes going back - I promise you the answer is there. If you are not going to bother to read the things that I spent quite a lot of time writing and quite a lot of energy trying to help you - then I don't see how you can honestly claim that you want to understand.

  94. Israel
    You determined its location and direction.

    You did not determine its polarization - therefore you will find that at B there is a 50% probability that the photon is polarized vertically or horizontally, depending on how you place the polarizer.
    If you put a vertical polarizer at A, now you will find that if the photon reaches B then its polarization is always vertical.
    And here's the magic - if you put a polarizer at 45 degrees to the road, then you'll find that again there's a probability that it's polarized vertically, or horizontally (again, depending on how you place the polarizer at B).

  95. Albanzion

    Thanks for the education series, but if as you write you don't want to fight, try to talk only about physics and don't get personal.

    And if Einstein didn't know about quantum mechanics does that mean it didn't exist? After all, the basic assumption of relativity is a photon that is in a specific place at a specific time, and moves at only one speed, c.

    So if this assumption is not true, what does this say about relativity?

  96. Israel
    You got answers:
    1) Yes, this is also true for photons.
    2) Once you describe a photon, you have actually measured it, so it is not spread out in space as in 1).

    Albanzo
    What I don't understand is how it works out with the two slit experiment - after all, there we shoot the particles as Israel describes, yet we witness that the particles are spread out in space. I guess the explanation is the partial knowledge - I know when the particle came out, but I don't know in which direction. To be precise - it is not my knowledge that is important, but the measurement itself.

    Shmulik
    Penrose's idea is interesting. This explains why the ordinary world seems to us ... ordinary.

  97. Israel,

    You just don't read what I write. Why should I bother anymore?

    In 1905, there was no quantum mechanics, and Einstein wrote special relativity as an extension of classical mechanics. Since then we have learned and the Torah has been reformulated in such a way that it can also be applied to quantum mechanics. Insisting on looking at a 1905 paper to understand private relativity is like trying to study evolution by focusing only on "the origin of species", or computer science only from Turing and his work in the 40s.

    You probably just don't want to understand. On the plate later.

  98. I didn't say I didn't know it, I just wanted to make sure..

    So I return to the original question that opened all the Wagers:

    Israel Shapira
    December 26, 2017 at 12:43 p.m
    Anyone volunteer to help?

    The uncertainty principle states that a quantum particle has no distinct properties prior to measurement. Is this also true for photons? If this is so, then how does it fit with the theory of relativity in which Einstein says in his original article:

    Let a ray of light depart from A at the time ta, let it be reflected at B at the time tb, and reach A again at the time t'a

    http://www.fourmilab.ch/etexts/einstein/specrel/www/

    After all, here we are talking about the fact that a light beam (it can also be one photon) is at a specified place - point A - at a specified time - t'a - and its momentum is specified - Planck's constant divided by the wavelength of the photon.

    So how does it work out?

  99. Roger Penrose has an interesting idea linking quantum mechanics and general relativity. He says that a particle can remain in superposition as long as the space-time curvature does not exceed a certain threshold. If a threshold is crossed (he assumes that the threshold is about one Planck mass) a collapse will occur. In my understanding his idea came to explain the transition from a quantum system to a classical system and not the collapse that occurs in the two slits experiment. He even proposes an experiment to test his idea.
    https://en.m.wikipedia.org/wiki/Penrose_interpretation

    Albantezo, do you know of any other experiments aimed at trying to test the limits of quantum mechanics?

  100. Albanzo
    The measurements do not mean that one knows the location, but every "classic" measurement, right?

    Example: If I perform the two-slit experiment with electrons in a closed laboratory, and record the position of the electrons using a detector, then no interference image will be formed. This condition exists, even if the detectors are not recording.

    What is not clear to me: the photographic film is also a type of detector... is it similar to polarizers? There is a simple experiment with 3 polarizers, which shows that a measurement on one axis "deletes" the information on a vertical axis. It's quite an amazing experiment 🙂

  101. Of course, when I write "a photon can move faster than light" I mean "a photon can move faster than the constant c". The photon is the light and therefore according to its definition it always moves at the speed of light :). Of course, the reference here is to a movement faster than the speed of light in classical physics, the constant denoted by c. As I said, even in quantum mechanics light moves at this speed but only between two measurement points. When it is not measured, it does not necessarily move at this speed.

  102. As I wrote in the previous comments (you can go through them again and see for yourself), a kino photon is required to move at the speed of light. The only commitment is regarding measurements, but as long as it is not measured it can move faster or slower than light. This is also true for other particles - an electron can move faster than light when it is not measured. The particles can also move in curved paths even though no force acts on them. Quantum mechanics distinguishes between the properties of the particle and the image we see. This difference is enormous in its importance and is responsible for most quantum phenomena. Most of the things we are used to from classical mechanics exist only in the picture obtained from measurement, and not in principle.

    If you studied many years ago and you don't even remember the basic principles or terms, how do you expect to understand? I recommend you do a very thorough refresh. The things I explain to you here are only basics and without them it's no wonder you don't have a clue when you ask more advanced questions about weaving, for example.

  103. I learned, but it was many years ago.

    "But in the ninth second (when you didn't measure it) it could have been a billion light years away."

    So if the photon managed to travel a billion light-years in nine seconds - doesn't that mean that it slightly exceeded the speed of light?

  104. Israel,

    If you want to try to understand and talk about quantum mechanics, you have to start applying its basic ideas. In classical physics there is no difference between "the particle is here" and "when I made a measurement I found the particle here". In quantum there is an abysmal difference.

    As I have already written several times - if you measure at a certain point the position of the photon (or produce a photon whose position is more or less well defined, up to a small uncertainty) and measure its position after 10 seconds, the difference between the positions will be 10 light seconds. I have written this explicitly many times already. But as long as you don't measure the photon you are not guaranteed where it is. That is, after 10 seconds you measured it and saw that it was 10 light seconds away, but in the ninth second (when you didn't measure it) it could have been a billion light years away. In classical physics this is not possible, in quantum it is. And this difference is huge, because physics is determined not only by what we measure but also what happens behind the scenes when we don't measure (examples of this are the two-slit experiment, the Mach-Zehnder interferometer, etc.).

    A photon has no trajectory in quantum mechanics. You know where it is at the moment of the first measurement, and if you measure it again you will know where it is at the moment of the second measurement. The difference between them will be ct, but this is only a tiny part of the physics of the problem. If you want to understand these topics you must must must study Introduction to Quantum.

  105. In classical physics, if a gun shoots a bullet at 300 meters per second, then after 10 seconds it will be 3000 meters away from the gun. It is not possible to find the ball at a distance of 2900 or 3100 meters.

    So I want to make sure: if a laser source emits a single photon at instant 0, and we divide the straight line in the direction of the photon's movement into intervals of 300,000 km, then after 10 seconds it will find light 10 seconds away from the laser? Doesn't he have a probability of being between interval 7 and 8 or between 13 and 14? And if the probability exists, is it negligible?

  106. Miracles,

    Now I think you may have asked something else. Perhaps you meant that you are measuring the momentum of a particle (which you do not know what its wavelength is - you are just an experimenter who finds an electron in the laboratory and measures its momentum). Of course, your measurement result will have a standard deviation, which represents the uncertainty in the electron's position (assume for the moment that your measuring instruments are infinitely accurate and no longer introduce error into the system). Are you asking if in such a case a large standard deviation in the momentum (or in other words, a lack of information in the momentum) would indicate a balance of information in the position?

    Yes and no. Yes, if you have many copies of the same particle and on one half you perform momentum measurements, and on the other half position measurements, indeed a lot of information on one side will translate to little information on the other side. That is, in most cases if you can describe the momentum with very few bits of half of the particles you will need a lot of bits to describe the position of the other half (there are of course situations that have great uncertainty in both momentum and position, but let's forget about them for now).

    No, because this is of course just a statistical statement about an ensemble of particles. As soon as you measured the momentum of a single particle, you changed its state and in particular you moved it (changed its position). It is simply impossible to measure momentum and position at the same time, so the question is not well defined (it depends entirely on the order of your measurements - first momentum then position, or first position then momentum).

  107. Israel,

    Photons have a location. Since photons are always relativistic, we have to talk about them in the framework of relativistic quantum mechanics - or in other words, field theory. There are ways to handle them also within the framework of "normal" quantum mechanics, but this is just a complication of things. Within field theory, a position is not an operator but a label of the field - the whole mathematical formalism becomes much more complicated. So the question "does a photon have a position operator" is very misleading, because the Torah within which we describe photons does not include position operators at all. But yes, if you insist on sticking to the classical formalism of quantum mechanics, it is possible to define a position operator for a photon (but it has to mix position and time because of relativistic covariance, i.e. because time and space are relativistically mixed). It is not clear to me why you are pursuing this question, maybe you want clarification because you have read something about it? In any case, this is a somewhat complicated technical question that I do not see the relevance of. Photons have a location that can be measured (but cannot be completely located as explained earlier). Your laser does not produce photons with a defined momentum, but photons in narrow wave packets that are strongly centered around a certain frequency, but they are nevertheless wave packets - that is, a superposition of momentums, and this is enough to allow us to "place" them and describe them as a beam. Because the wave beam is narrow, so when you measure its frequency the light you get with a very high probability the same frequency for all photons and therefore measurement shows you that the light is only at one frequency, i.e. very coherent. But we have already talked about the abysmal difference between what a measurement shows and what happens in reality - in quantum mechanics, the result of the measurement shows us only one end of physics, and a lot of things happen behind the scenes. As Nissim mentioned, this is the whole secret behind particle entanglement or the two-slit experiment which is completely destroyed as soon as it is measured.

    Miracles,

    You have to be very careful here. In any case, if we have the wavefunction of a particle (or its quantum state, it's exactly the same thing) we know everything there is to know about it. The state (or the phon') describe the particle completely. When we say that there is great uncertainty in its location, it does not mean that we lack information. This means that the particle itself is in such a distribution of locations that it is very difficult to guess where it will be measured. You know exactly what the distribution is anyway.

    There really is a connection here to information theory - distributions that have great uncertainty are distributions with high entropy in the classical sense. But I wouldn't go into it, I think it will confuse you more than teach you anything new. My short and concise answer is - von Hegel tells us everything there is to know about the particle in any case, whether she tells us that it is certain to be at a certain point or whether she tells us that it can be found at any point in the universe with equal probability (for the sake of it, let's assume that the universe is finite) . The interesting applications of information theory in quantum mechanics is when talking about particles that cannot be described using a state in the Hilbert space because they are not part of a closed system. Or in other words, thermodynamic states.

  108. Not an ideal photon - an idyllic photon.

    One that is spread out on a futon mattress in an idyll, in an unspecified state, spread over the entire space, from a weak base...

    Miracles

    In my understanding of quantum it is not that if you know the exact momentum then you do not know the position - there is simply no momentum or position until the measurement.

    Albanzo

    Many people will think that if you write "there is no difference except..." then you mean that there is no difference except marginal details, and not in essence.

    But as I wrote, if we agreed that one side in the interweaving immediately affects the other side, we will sue.

    I am not only addressing you - the reference was "to anyone who volunteers to help". You volunteered. ignore?

    So aren't the photons created by my laser spread out in space with equal probability? Are they only focused at ct distance? Does a photon have a position operator at all?

  109. Albanzo
    I have a bit of a weird question. In my understanding, the better I know the momentum then I actually have more information (the number of significant bits representing the value of the momentum). Is it related to the "lack of information" I have about the location? That is - is there any connection to information here at all, or is the explanation more fiscal.

  110. Miracles,

    Right. States that have well-defined momentum (or well-defined position) exist mathematically but are not physical, except when space is finite.

    Israel,

    1. I understand. So the root of the problem was that I explicitly and clearly said that there is a difference and that is that one is quantum and the other is classical, and you decided on your own that this difference is negligible and from that you concluded that I was actually saying that there is no difference. It's hard for me to think of a less negligible difference, but so be it.

    2. I admit and confess - I didn't even read what you wrote about Aspa's experiment. Once I realized you were taking something specific I said about a particular case (the puzzle where the shared information between two end users was predetermined when they matched their spin pairs) and applying it to something I didn't even talk about, I realized that at best I'd have to dig through past discussions again and teach you another lesson Private (Tell me, did you notice during the discussion how many things I explain to you? How much information I share with you? Now go to the discussion and see how many things you told me. Sorry, but you can't call what's happening here a discussion at all because it's one-sided, there simply isn't another word except private lesson). At worst, it's just another fight. I'll leave you to think about an assembly experiment yourself and come to whatever conclusion you like.

    3. Listen, I don't know how to explain it more clearly than I did. Every particle in quantum mechanics is described by something called a "state". This state can be a certain frequency (what you call an ideal photon), but it doesn't have to be - there are also photons that don't have a certain frequency but have a superposition of frequencies. "Ideal" photons as you say (there is nothing more ideal about them than others, but I'll adopt your language if it helps you understand) are spread all over space, no matter what their frequency is. An ideal photon with a frequency of 1 Hz is spread over the whole space, an ideal photon with a frequency of a billion terahertz is also spread over the whole space. Non-ideal photons are not spread over the entire space, and of course they do not differ from each other in frequency (after all, they do not have a well-defined frequency, so it is quite clear that this is not the distinguishing feature between them). At most it can be said that they differ in their wavelength in the frequency space (which, of course, does not contain information only about their frequency but about all their physical properties such as location).

    4. An ideal photon as you say (ie having a certain frequency) cannot be created in a flashlight. It is spread evenly over the entire space, both at the moment of creation and 13 billion years later. The only thing that can change this is if it changes its quantum state, as a result of an interaction with another system or as a result of a measurement. So it will no longer be spread over the entire space, but it will no longer be ideal either.

  111. Israel
    In my understanding, Albenzo says that once you have information about the photon, you have measured it, and therefore it is no longer "scattered throughout space". It is similar to the slit experiments - as soon as you look through which slit the particle passed, then it no longer passes through both (no interference).
    Therefore - the same photon from your flashlight will move in a classical way.

  112. Albanzo
    I think I understand what you're saying:
    1) If I have complete knowledge of the momentum of a particle then I have no knowledge of its location, so there is an equal probability of finding it anywhere in space.
    2) In practice - it is impossible for us to know the momentum (or the position) absolutely.

  113. Let's start from the end.

    "Maybe you think I just want to humiliate you and that I enjoy telling you that you don't understand and don't know, but I assure you that's not the case."

    No way? I'm glad you tell me things I didn't know and try to learn from them.

    But it's a little different when you start with the liar, idiot, crook, etc.. Do you at least in this case see the fundamental difference?

    But does that mean I agree with everything? Negative. If you make a valid argument that I understand and accept, I agree and thank you, as I did for explaining the 4 states of Bell.

    If not, I continue to ask questions and pit my understanding against yours. This is the way to hold a discussion and learn, until one of the parties is convinced (or not).

    But what to do when when I don't agree with you you immediately get angry and start cursing me? Well let's hope it gets better.

    Here's an example:

    You wrote: "There is no difference between this solution and any other solution of a pre-coordinated classical code (as some of the commenters here tried to suggest), except that this code operates according to quantum laws, and in particular it is found in superposition."

    To me it's like saying: you have to choose between the two twins Chaya and Muarta. There is no difference between them except for a few marginal details - for Haya there is a spot on her back and for Muerta a small scar on her hand, etc.

    But when you try to understand what the difference is, it turns out that they are indeed genetically identical, they look the same wearing the same clothes, everything is the same except for the negligible and marginal fact that she is alive, and Muerte is dead (Muerte in Spanish). negligible

    So what does "there is no difference between this solution and any other solution of classical pre-matched code" mean? The difference is mostly, isn't it?

    And does it matter if you met before and coordinated the spins? From the moment you introduced new data - the state of the polarizers in the Aspect experiment, which changes after the photons have left the source and is therefore not included in the previous coordination - the previous coordination is no longer relevant.

    This can be seen in the example of the numbers that are written on notes and separated. It is true that in classic coordination I look at my number and know what your number is without the transfer of information.

    But from the moment an additional factor is introduced - for example each party has to multiply its number by another number that the other party does not know about - then what does all the early coordination matter anymore?

    Does not matter. If we agreed that in weaving a measurement on one side *affects* the measurement on the other side (something that does not exist in the case of numbers or gloves that were separated) then we have no more disagreements, and I already wrote a long time ago that it doesn't matter to me if they call it information, it can also be called Moshe.

    Just a little advice for you as a teacher: try when you write "non-local correlations" and "quantum laws" to also mention that this includes the ability to influence from a distance.

    photons. It is still not clear to me: is there a difference between photons apart from the wavelength? Can there be two photons with the same wavelength and still have some kind of difference between them? Different wave packages maybe?

    And if the answer is negative. When you say "Suppose I'm looking at a specific photon that does have a well-defined momentum, i.e. its uncertainty in momentum is 0. Yes, in this case it will have an infinite uncertainty in position, i.e. it will be spread all over space", does it not require that in fact Are all photons like this? After all, the difference between that idyllic photon and the others is only their frequency, isn't it? (Now we don't go into Toria theory and what the frequency is, it's enough that there is one photon that spreads evenly in space).

    And a second question. Can the same idyllic photon be created in my flashlight?

  114. Israel,

    Let's start from the end. I'm sorry, your reading comprehension is bad. When they say "there is no difference between Moshe and Danny except that Moshe is blond and Danny is red", does that mean that Moshe and Danny are the same? When you meet them, will you be able to tell them apart or not? I wrote that there is no difference *except* that one of them refers to classical laws and the other to quantum. It's a big difference. How did you come to the conclusion that I am saying that there is no difference between local systems and non-local systems? I wrote explicitly, in black and white, that there is a difference and this difference is the difference between the quantum and classical laws. The quantum allows non-locality, the classical does not. My God, it's like you're not even trying to read what's being written to you. As usual, the problem stems from the fact that you focus on one sentence (or one paragraph) that you think will help you advance your opinion and prove that you are right, and ignore everything else. Back to the discussion. In the original discussion I tried to explain something simple to you:

    1. Classically, it is impossible to solve the puzzle without transferring information. Yes, it can be solved by pre-coordinating a code between the two experimenters.
    2. It can be solved with interlaced spins.
    3. You deduced that the entangled spins involve information transfer.
    4. This is wrong. The entangled spins considered are not analogous to the information transfer solution, but analogous to the pre-coordination solution. As evidence, this method cannot be applied without the two experimenters meeting first and adjusting their systems. It is also possible to quantify the amount of information they need to coordinate among themselves in advance and see that it is no less than the amount of information they had to convey to each other to solve the puzzle.

    That's all I'm going to say about it. I told you that this is not a story time where you can bring up any memory from the past that comes to your mind and expect me to entertain you. The only reason I answered your question is because I was shocked by your level of reading comprehension ("You're saying that the only difference between a kilo barbell and a ton barbell is the weight? So you're basically saying there's no difference between them?!")….

    Regarding the photon, I'm sorry to tell you - you still don't understand. First of all, just because you have a measuring device that shows a certain wavelength does not mean that the photon has a certain wavelength. As usual I will explain that I do not intend to insult you, but I must repeat the constant sentence - you simply do not understand the basic principles of quantum mechanics. Even a photon that is in a superposition of infinitely many different frequencies will show you one specific frequency when you measure it. You simply do not perceive that the quantum state of a particle and the results of the measurements obtained on it are different things. This is actually all of quantum mechanics on one foot - in classical mechanics a particle is uniquely and completely described by the collection of measurement results on it, while quantum is misinterpreted. There may be two different particles with completely different physics that will give exactly the same measurement results.

    It's really the simplest thing in the world, and it's exactly like I said before: it's created in the whole space. Your question "where is it made" is like asking what color a bald person's hair is or how many meters are in a kilogram. is a nonsense question that stems from classical thinking that is explicitly wrong in quantum mechanics. From the moment the photon is emitted, it is found throughout space. It is not created near the flashlight (of course I'm talking about the photon with a well-defined frequency, and not the flashlights you have in the lab that create photons that are wave packets). Once you measure its position, you collapse its wavelength. This hook ensures that if you measure its location again, meaning a second time, after one second, you will get a result that is exactly one light second from the first place you found it. If you really want to understand these things you must learn an introduction to quantum, specifically learn what a measurement is, what a quantum state is and what the difference between them is.

    Regarding the probability, you are both somewhat correct. A particle with a well-defined momentum indeed has an equal probability of being found at any point in space. Nissim is right to say that such a uniform distribution is not possible in infinite space, or rather - it cannot be normalized. This is completely true and indeed in quantum mechanics on an infinite space there will be no particles with a well-defined momentum (that is, the eigenstates of the continuous momentum operator are not in the Hilbert space, but in an auxiliary space which includes the Hilbert space and are the so-called overcomplete basis of states). That's why I made sure to say that they can only be created approximately - they are real physical objects only in a finite space. We still talk about them in quantum mechanics for several reasons: first, they are not really physical but as close as we like to real physical states. Second, any physical state can be built from them and therefore an understanding of their physics makes it possible to understand the physics of any real physical state. Third, they give us good intuition because their mathematics is simpler than that of wave packets.

    Israel, you keep repeating the same mistakes over and over again. Earlier you said that each photon has a certain momentum. I explained to you that this is not quantum true. So now you say that the only thing that differentiates photons is the momentum. Ok, you changed a few words but you still insist on completely ignoring quantum mechanics and dealing only with classical physics, where each photon has a path that uniquely defines position and momentum. In case it is still not clear - no, the difference between photons is not only in their frequency. Most photons have no frequency, as has already been explained to you several times. The difference between photons is in their quantum state (which includes their possible frequency distributions). Sorry to sound like a broken record, but if you want to really understand any of this stuff, you have to study Introduction to Quantum. You may think that I just want to humiliate you and that I enjoy telling you that you do not understand and do not know, but I assure you that is not the case. Remember that I teach as part of my profession and that I have the instincts of a teacher. I am really telling you these things to help you understand - you have a huge gap in the most basic ideas of quantum mechanics (what defines a particle in quantum mechanics? What is the difference between the description of the physical particle and the description we see when we measure it? What does a measurement do to a particle? What is the connection between momentum and position? etc. etc. etc.). Without completing these gaps you will not be able to understand the solutions to your questions, and will only sink deeper and deeper into your misunderstanding.

  115. ELBENTZO

    Please answer the question I posed to you earlier.

    You wrote "There is no difference between this solution and any other solution of a pre-coordinated classical code (as some of the commenters here have tried to suggest), except that this code operates according to quantum laws, and in particular it is found in superposition"

    Do the quantum laws and superposition include the possibility of influencing in zero time from one end of the experiment to the other end, something that does not exist in the classical code? This is the root of the problem, isn't it?

    The issue is important because there are two options:

    1. The answer is positive, that is, the quantum laws and superposition include the possibility of influencing in zero time from one end of the experiment to the other end, something that does not exist in the classical code, and then in fact there is a big and fundamental difference between this solution and any other solution of a pre-coordinated classical code (as tried Some of the respondents here suggest).

    2. The answer is negative, then there is really no difference between the two options.

    If the option is 1 - then you are basically saying that there is no difference between a system that allows non-locality and a system that does not?

    If the answer is 2, the situation is even worse. Don't you see the obvious difference between the two cases?

    And in any case, if you say there is no difference - then why do you get angry and call me a liar and a liar when I tell you to try to create such a classic code? We know that quantum works, so if there is no difference as you say then why don't you try to offer such a system?

    It seems to me that, like many cases in the past, you simply did not understand what I was saying and immediately attacked me.

  116. What's wrong with equal probability?

    Muons have nothing to do with photons. Photons are a different animal in the particle jungle, because they are the only ones (as far as I know) that obey postulate 2.

    The lifetime of the muons is lengthened or the distance they travel on the way to the ground is shortened due to the lengthening of time or the shortening of length according to relativity. This happens to any bone, especially the bones of twins.

    But that has nothing to do with the issue at hand, unless you see it as slam dunk proof of relativity.

    The issue before us is this: if a specific photon does have a well-defined momentum, i.e. its uncertainty in momentum is 0, there is infinite uncertainty in position, i.e. it will be spread throughout space, doesn't this require that all photons are like that? After all, the only thing that differentiates between photons is their wavelength, so why does the part of our photon differ from the rest of its brothers? What is special about it?

    And if each photon is actually spread with equal probability throughout space, then why do we only measure them at a distance ct from the source?

    I believe it may be because our measuring instruments cannot measure the fast or slow parts of the photon from c but only the part that moves relative to them exactly at c, but there could be another reason.

  117. Israel
    The lifetime of a muon is too short to pass through the atmosphere. From the point of view of an observer on the ground, we see that the muon's time is lengthening, meaning its life span is increasing.

    But for him, the path through the atmosphere must be shortened.

  118. Israel
    It doesn't seem to me that you understand what a uniform distribution is (I assume that's what you mean).
    Such a distribution can only exist over a finite domain.
    We learned this in high school…

  119. as written:

    "Its uncertainty in momentum is 0. Yes, in this case it will have infinite uncertainty in position, meaning it will be spread over all space."

    And what is not written but I believe that Albatezo will confirm soon:

    with equal probability.

  120. Israel
    I wrote earlier - there is a very high probability of finding it where you expect, a lower probability in the environment very close to this point, and a very very very low probability of finding it at a greater distance (I think the probability is exponential).

    How did you arrive at equal probability?

  121. Let's see what Albantezo wrote:

    Suppose I'm looking at a specific photon that does have a well-defined momentum, meaning its uncertainty in momentum is 0. Yes, in that case it would have infinite uncertainty in position, meaning it would be spread all over space.

    Him, Him I love. It's just not clear to me: where was it created? Can I create it in Santa Monica? Because if so, and I don't see why not, then despite the infinite uncertainty in space and the equal probability of being found everywhere, rest assured that only a light second away will you receive it a second after it was created.

    So where did his remains disappear scattered with equal probability in space?

    What's the problem with muons? The extension of time?

  122. Israel
    It is not true that you will receive it everywhere - there is a certain probability that you will receive it everywhere, and that is something completely different. The probability that you will receive it at a distance c*t after time t is very high, and that the probability that you will receive it in the vicinity close to this point is very small.

    By the way - how do you solve the muon problem? How do they get to the ground? They are not photons and do not move at speed c.

  123. My laser located in Santa Monica emits a monochromatic green photon with a wavelength of 507.65789654346 nanometers. This is what the measuring device shows.

    If it is created in the entire universe then at any point in the universe after a second we will absorb it at any other point. Since the only place it's been picked up is a light second away from Santa Monica, that's probably where it was created.

    And you didn't understand the rubber example. Think only about 100 stones that move at speeds from 1 to 100 meters per second. If I am an MGB soldier in the areas where the stones are moving in my direction like a barrage, can you tell me the speed of the barrage?

    Miracles are not a problem, they are the solution.

  124. Israel
    Even then, I wouldn't have put money into those photons.
    But let's assume so - what's the problem here? In everyday electronics, components based on such phenomena are used (you can buy these on Amazon).

  125. If you are in Los Angeles and turn on a light that creates a new photon, and that photon is *a photon with a well-defined momentum* (ie having a specific wavelength and not a wave packet) then it was not created in Los Angeles. It is created equally at every point in the universe, i.e. in superposition. Welcome to quantum mechanics, I recommend that you repeat the introductory course from Year XNUMX. As I have already told you several times, your intuition that the photon comes out of the flashlight and has a path (beam) that can be followed directly stems from the fact that the flashlights you know do not produce photons with a defined frequency, but produce wave beams that contain a sequence of frequencies.

    And no, even in a classical world, the rubber object can only be defined as a speed if it is a rigid body, that is, the parts that make it up move together. You can repeat it as much as you want, it doesn't make it right. Each part of the rubber moves at one single speed, and because the speeds are not identical to each other, tension is created in the rubber.

  126. Because I didn't send a single photon, I sent the whole movie Gone with the Wind and I got it in Andromeda.

    It is possible that these are not the same photons, but the same can be said about the existing flashlight photon.

    do i agree Please Eraf, how do I know? I say it's at least as likely as the alternative: multiple worlds and influence on the past. You have to do an experiment and check.

  127. Israel
    Yes - but do you agree with it?

    By the way - your photon is not in Andromeda after 10 seconds. Let's say we did the experiment. You put two detectors, one in Andromeda and one in Makrah 3 million km. After 10 seconds you did not record a photon in the nearest detector, and you did record a photon in Andromeda.
    How do you know it's the same photon?

  128. And regarding no comm.

    What I wrote is correct. In the device I described with the radios and the coins, nc refers only to the second part, namely sending information through the device which is impossible. It does not deal with the first part, namely how the communication goes between the radios themselves.

  129. of

    https://www.hayadan.org.il/is-the-universe-ramdom-0405168/comment-page-6/#comment-707429

    "Finally, regarding my comment that all the information was already in the system. So we started with 300 interlaced spin pairs, right? But wait, how did we get them? And is it enough to know that they are intertwined? Each pair should be produced by local measurement. That is, before the start of the experiment, the two experimenters (the one from KDA and the one from Mars) had to meet somewhere and produce the intertwined pairs. But it is not enough. They must know exactly which interlaced state they have (because even the smallest system has at least 4 interlaced states), otherwise the code cannot be set. So not only do they have to meet before and perform a measurement, they also have to know exactly what measurement they performed and what was the process of the collapse of the system. Only then can each of them go to their end point and perform the experiment. That is, they come with a *huge* amount of pre-matched information, and all they do during the experiment is collapse it with a local measurement, each on their own spin. There is no difference between this solution and any other solution of a pre-coordinated classical code (as some of the commenters here have tried to suggest), except that this code operates according to quantum laws, and in particular it is found in superposition (as you know, the secret of the magic of entanglement is that it is a state of superposition, and not just but a special superposition which is non-spherical to pure subsystems)'.

    When you say:

    "There is no difference between this solution and any other solution of a pre-coordinated classical code (as some of the commenters here have tried to suggest), except that this code operates according to quantum laws, and in particular it is found in superposition" Do the quantum laws and superposition include the possibility of influencing in zero finite time One of the experiment on the other end, something that does not exist in the classical code? This is the root of the problem, isn't it?

    "The article does not mention a problem with the mathematical model. That's exactly what I'm saying, why don't you understand? In the article they talk about a certain tension, but this tension is in the philosophy of physics and not in its mathematics.'

    I did not see a mention of the mathematical model in the article. Here is what is written in the article about the physical model:

    "Quantum mechanics has shattered many intuitions, but this is the most profound of them all. And this particular shattering carries in its wings a threat whose shadow has not yet been removed, a threat to special relativity, one of the cornerstones of our 21st century physics.

    The most alarming thing about non-locality, apart from the jarring strangeness inherent in it, is that this feature carries with it a threat to the special theory of relativity as we know it today. In recent years, this concern, which has finally gained entry into the hall of serious thoughts in the field of physics, has become the focus of discussions that may, in the end, tattoo, distort, reimagine, consolidate or disintegrate the very foundations of physics.

    The bad news was therefore not the lot of quantum mechanics but of the principle of locality, and anyway, it seems, of special relativity, since, at least apparently, it is built on the assumption of locality.

    It seems that the type of non-locality encountered in quantum mechanics requires absolute simultaneity, which poses a real and deadly threat to special relativity.

    The state of special relativity, only a little more than a century after it appeared on the world stage, suddenly became a wide open and rapidly developing question.'

    So you claim that phrases such as "disintegrate the very foundations of physics" and "a real and deadly threat to the special theory of relativity" are merely "a certain tension, but this tension is in the philosophy of physics and not in its mathematics"?

    "Here is an excellent example of a crazy distortion of my words and even an invention: "Are you claiming that in the subtext they actually meant the opposite and that there is no threat and that they are getting along great and that the authors of the article fabricated something imaginary to increase circulation?"

    Didn't you write a few days ago:

    "First of all, you have to remember that every article in a magazine - even a scientific one, and certainly in Scientific American, which addresses the general public and has a huge interest in selling copies - can fall into a natural tendency for sensational or bombastic headlines."

    and also:

    "In conclusion, it is very, very difficult for me to see how in the content of the article there is any risk of combining private relativity and quantum mechanics."

    Indeed there is no risk. "to crumble the very foundations of physics" "a real and fatal threat to the special theory of relativity". I just distorted, fabricated and invented, and as always I lied and lied and lied again.

    Matter of fact:

    "5. As I wrote earlier, a photon of a certain frequency will indeed have an infinite uncertainty in position. You ask if it can be found ten light years away from where it was 10 seconds ago. This question has a two-part answer: First, ten seconds ago you didn't even know where he was. It is a photon with a certain momentum, its position is unknown. You assume that ten seconds ago you knew where he was, but it's just not possible. Just as now you don't know where he is, you didn't know ten seconds ago either.'

    I'm in Los Angeles and I turn on a flashlight and create a new photon that didn't exist before. After 10 seconds, the photon is also found in Andromeda, whose clocks are synchronized with Earth.

    So what does "Ten seconds ago you didn't even know where he was" mean? He was in Israel, where he was born. And if after 10 seconds it's in Andromeda, then it's a fast and very fast photon, isn't it?

    "The rubber doesn't move at any speed, that's what I'm explaining to you. There is no such physical object as the "rubber". I mean, obviously you can define it (the connection of all the particles that make it up) but you can't assign a speed to it.'

    We are talking about a classical world, in which there is such a physical object, rubber. If I shoot 10 stones and each one moves at a different speed than its friend, then they move at 10 speeds. And if they are connected to each other with rubber, then the object stones + rubber moves at 10 speeds + all the speeds in between. The same with 1000 trillion stones, until in the end you are left with only the rubber that moves at all speeds from 0 to the highest.

    And what is important for our purposes, such a strange description for the photon solves no less strange descriptions such as influence on the past and also explains postulate 2.

    You don't have to respond if you're not interested, good night.

    Miracles

    That's what Quantum claims, isn't it?

  130. 1. I prefer not to go back to old fights and I definitely don't want to start talking about past conversations. All that happened here is that you put a link to a certain article and I gave my opinion on it. Then you wrote something about no-comm sentences and I saw that what you wrote was incorrect and could also mislead others, so I wrote a clarification. At no point did I intend and still do not intend to open any discussion or fight with you that we have ever had. But there are no absolutes in life and you may want to open an old conversation because it is actually an interesting topic of conversation that is relevant to something I wrote here. In this case it's quite possible that I'll be remiss and I'd be happy to clarify what I meant, as I did when you asked me about the uncertainty of a photon (even though I've already explained it to you before). In conclusion - and this is as clear as I can be - I am not interested in opening old discussions, but I do not guarantee 100% that if you ask me to clarify a previous claim I will refuse. If the claim is interesting and seems relevant to me or you deserve clarification, I will clarify. In this case you must give me a full link to what I said and the context in which it was said because that's the only way I can explain what I meant.

    2. The article does not mention a problem with the mathematical model. That's exactly what I'm saying, why don't you understand? The article talks about a certain tension, but this tension is in the philosophy of physics and not in its mathematics. That's all I claimed in the original comment I wrote on the article, and that's what I'm claiming now. To me there is a *huge* difference between a problem in the theoretical model and a problem in the philosophical interpretation. If you prefer not to make a difference, leave. your right I thought it was very important to comment on that.

    Here is an excellent example of a crazy distortion of my words and even an invention: "Are you claiming that in the subtext they actually meant the opposite and that there is no threat and that they are getting along great and that the authors of the article fabricated something imaginary to increase circulation?" You know very well that I never said anything like that. I did not write that they fabricated anything. I did not write that they meant the opposite. I'm just repeating - unfortunately you can't understand, but that's between you and yourself - that the problem they present is a problem in the philosophy of physics, that both the problem itself and its solution are found in the interpretation of how we understand the model and reality, and not a problem in the mathematical model itself.

    3. What does it mean "why does it not have an absolute wavelength in a given reference system"? Why is 2+2 not equal to 9? You will learn quantum mechanics. A photon has a state (ie, described by a vector in Hilbert space). This state can be an eigenstate of the momentum operator (in this case any measurement of the photon's momentum in a certain frame of reference will always give some result, and we call this result the photon's momentum) but the photon's state can also be a state that is not an eigenstate of the momentum operator. Since the Hermitian momentum operator and its eigenstates are a complete set that spans the Hilbert space, then it will always be possible to present this state as a superposition of momentum eigenstates - that is, a photon will not have a particular momentum, but each measurement will give one momentum with such probability, a second momentum with another probability and so on . You simply insist on ignoring quantum mechanics and looking at a classical picture where every particle has a well-defined momentum.

    Regarding the green photon - let's start with the fact that you have probably never seen a green photon. When we see a "green" photon in reality, we actually see a photon in a variety of wavelengths, but the green one has the highest amplitude among them. That is, what you call a green photon and think intuitively that it has a precise wavelength that corresponds to the color green is usually simply a relatively narrow wave packet centered around the green frequency. A photon with a green frequency is exactly that, as I said earlier, something that can be roughly created in a laboratory and is not actually found in nature. But again - as I said before - I am not claiming that there is no such thing as a green photon, that it cannot exist. I'm just saying it's not true in general. Saying "a photon has a certain wavelength" is like saying "according to Newtonian mechanics, a massive body exerts a gravitational force of 3 newtons around it". In general this is not true (the force depends on the mass of the body and the distance from it) but it is clear that there may be situations where the force of gravity is really 3 newtons... there are green photons. But to say in general that a photon has a certain wavelength is simply not understanding the foundation of the foundation of the beginning of the introduction to quantum mechanics. Sorry.

    5. As I wrote earlier, a photon with a certain frequency will indeed have an infinite uncertainty in position. You ask if it can be found ten light years away from where it was 10 seconds ago. This question has a two-part answer: First, ten seconds ago you didn't even know where he was. It is a photon with a certain momentum, its position is unknown. You assume that ten seconds ago you knew where he was, but it's just not possible. Just like you don't know where he is now, you didn't know ten seconds ago either. You suppose you could tell it was ejected from the lantern and therefore was there, in the lantern. But quantumly this is not true. If you have a flashlight that emits photons with a certain momentum, it can emit them at any point in space - even very far away from itself. Again, this is of course related to the fact that the flashlights you know from everyday life simply don't work like that and don't emit photons with a specific wavelength (and therefore it is possible to more or less follow the position of their light beam).

    The second part of the answer is related to the fact that quantum mechanics is a statistical theory. A quantum state (or phonon wave) is not subject to all the laws you are used to, and unlike a photon moving at the speed of light. What is subject to these laws is *measurements*. There is of course a mathematical proof that if we measure the position of a photon at a certain moment, and measure its position again after a second, the difference between the positions will be one light second. But it is related to the measurement process in which the Hegel wave collapses. This is not true for the quantum state of the photon, which can be in a superposition of all kinds of states. When it is not measured, the particle is, for that matter, virtual and is not obliged to comply with the equations of motion (I have also explained this to you before). In quantum mechanics the physical conditions hold for the things we measure.

    6. The rubber does not move at any speed, this is what I am explaining to you. There is no such physical object as the "rubber". I mean, obviously you can define it (the connection of all the particles that make it up) but you can't assign a speed to it. Just like you can define a "Moshe and Danny" object, but you cannot associate a speed with this object. If Danny is standing still and Moses is in a spaceship that moves close to the speed of light, this does not mean that the object "Moses and Danny" is moving at the speed of light and standing still at the same time. It is possible to assign velocity to an object only when all the particles that make it up move together, and this is what is called a rigid body. We know that in reality there is no such thing as a rigid body, but in classical physics it exists and is a good approximation (for example, you can ask how fast I am moving even though I am also made up of many different particles. Treating a person as a rigid body is not accurate but is a very good approximation on the everyday scales that are not relativistic or quantum).

    7. If you cut the rubber nothing will come close. Simply at a certain point there will be no more internal forces in the rubber, i.e. tension, and as a result half of the rubber will stop accelerating (again, there is a chain reaction here because there is still tension in the cut part as well). I have no idea what you are after.

    Israel, you still ignore what I asked you, and you repeatedly treat me as a private tutor that you can just bombard with questions and expect him to teach you quantum mechanics or relativity. With all the good will, I don't have time for that. If I write something (in response to you or someone else) and you want clarification or want to argue that I'm wrong or something, that's your right. But I think this story time where you just sit and pour questions on me about photons and uncertainty and frequencies and superpositions and rigid bodies and rubbers and what is the difference between physics and the philosophy behind physics, has come to an end.

    Good night.

  131. Do not understand.

    Do you want me to bring links to what you said in the past as implied by "please give me an exact link to what I said and the context" or you don't want me to bring past links as implied by "I don't know why you think I have time to go with you to look for fights from the past and wallow in them again".

    Knew me clearly, mind reading and texting have not been working so well lately.

    Where is it mentioned in the article "a problem in the mathematical model that tries to combine quantum mechanics with special relativity, which of the three possibilities exists - is the field theory not relativistic as I claim, is it not quantum as I claim, or is it simply an inconsistent theory?" I claim that there is no contradiction at the level of the model and I even gave an example of a model that contains both theories. Now you have to help me understand that I'm wrong and the model doesn't really combine the two.'

    I brought 4-5 quotes from the article including the title: Quantum threat to relativity. Are you claiming that in the subtext they actually meant the opposite and that there is no threat and that they are getting along fine and that the authors of the article fabricated something imaginary to increase circulation? Goodbye, good health.

    But there is no doubt that at least on the face of it, in simple terms, they say exactly what I say, don't they? So explain all the quotes I gave.

    "A photon does not necessarily have an absolute momentum" Why does it not have an absolute wavelength in a given frame of reference? If so, what is the wavelength of a certain green photon? And isn't the momentum of the photon equal to dividing the absolute Planck constant by the wavelength of the photon?

    "But let's move forward. Suppose I'm looking at a specific photon that does have a well-defined momentum, meaning its uncertainty in momentum is 0. Yes, in that case it would have infinite uncertainty in position, meaning it would be spread all over space. So what? It's not clear to me what you're aiming for, or why you even chose a photon because this is true for everything within the framework of quantum mechanics.'

    Beauty. So let's work with this particular photon. Does he have complete uncertainty in place? It could also be a light hour away from the source that emitted it 10 seconds ago? So if speed is defined as distance divided by time, didn't we get a much higher speed than the speed of light?

    "In general, if you look at the last posts you will see that there is a strong trend here where you are just trying to look for fights" How did you come up with this? I haven't even turned to you in over a year, but you are the one who turned and is turning to me.

    But let's continue in the present way. Actually starting to be productive.

    "The fact that each part of the rubber moves at a different speed does not mean that "the rubber moves at all speeds".

    So at what speed does the rubber move? What if we mark each ion from it and measure the speed of that ion, won't we get approximately 100 different speeds that gradually increase from 0 to the speed of the arrow?

    And since this is just an example of the way I see the photon, if we cut the rubber at some point during the arrow's flight ("measurement"), won't the rubber "collapse" at once when the speed of the collapse is of the order of the speed of the rubber? Remember something?

    Good night, 3 in the morning.

  132. The fact that each part of the rubber moves at a different speed does not mean "the rubber moves at all speeds". I mean, you can say that if you want, but it has no meaning in physics because the rubber is not a rigid body and therefore no velocity can be attributed to it. You can assign an average speed to it, of course, or say that every other part of it moves at a slightly different speed (ie, stretching). If I run at a certain speed, Nissim runs a little faster, Moshe runs even faster and so on, does this mean that the person moves at all speeds? Physically this is a meaningless statement, although it can be understood literally.

    Clarification: If the rubber is made of a series of particles, each of them has one defined speed (of course, at the moment we are talking classically). There is no particle that does not move at one particular speed. The idea of ​​"moving at all speeds" is just a result of you taking many different particles and deciding to treat them as one object. But they are not, this is exactly what is called a rigid body and we know that it is not physically possible - neither in rubber nor in any other complex body.

  133. 1. As usual, you talk in headlines. In the article, the writers don't just say the words "there is a problem" as you keep quoting, they also specify what they think the problem is. Stop, take a moment to read about the two problematic aspects they present, and check what is problematic about them. Stop blindly quoting the words "there is a problem" just because it suits your agenda, learn what exactly the problem is. I spent some time on the subject (not only reading the article itself but also researching the topics presented in it in professional magazines and on the Internet) and I claim that the problem is completely philosophical and is not reflected in the mathematical model. The authors also tell about a possible solution by adopting an interpretation (of Hilbert space as the space in which we live instead of space-time). Instead of constantly demanding that I answer you and explain to you, maybe for once you will be the one to explain: explain to me if there is a problem with the mathematical model that tries to combine quantum mechanics with special relativity, which of the three possibilities exist - is field theory not relativistic as I claim, is it Is it not quantum as I claim, or is it simply an inconsistent theory? I claim that there is no contradiction at the level of the model and I even gave an example of a model that contains both theories. Now you must help me understand that I am wrong and the model does not really combine the two.

    2. I have no idea what you are talking about. I don't think I ever philosophized here, I only talk about physics. Again, a whole section with no content, you're just looking for a fight.

    3. I can't understand what you don't understand. Now you added the words "you always said". Ok. I'll say it again, maybe a little more slowly - if you want me to explain a certain claim that I supposedly made, please give me an exact link to what I said and the context. It's not about physics at all, it's just basic decency. When a person is asked to explain a claim, he must be given the opportunity to understand exactly what the claim is, when it was made and in what context. If you give me an exact link to an old discussion, I'll try to make time to read it and explain again. Although as I hope you understand, fighting with you is pretty low on my list of priorities. I don't know why you think I have time to go with you to look for past fights and wallow in them again.

    4. A photon does not necessarily have absolute momentum. Every particle (including a photon, but also an electron, etc.) in quantum mechanics is in a certain quantum state. This state can represent a particle that has a well-defined momentum (at the mathematical level - the state is an eigenstate of the momentum operator), but it doesn't have to be. A particle can be in a superposition of two momentums, or in any infinite wave packet containing a spectrum of different momentums. This is also true for photons. In fact, photons with precise momentum is something that is only approximately found in the laboratory, and all the photons you encounter in everyday life are actually wave packets that have no definite momentum.

    But let's move on. Suppose I'm looking at a specific photon that does have a well-defined momentum, meaning its uncertainty in momentum is 0. Yes, in that case it would have infinite uncertainty in position, meaning it would be spread all over space. So what? It is not clear to me what you are aiming for, or why you even chose a photon because this is true for everything within the framework of quantum mechanics.

    In general, if you look at the last posts, you will see that there is a strong trend here where you just try to look for fights, ask me mysterious questions without explaining what you want (a well-known tactic of trying to knock someone down) and it is not clear where they suddenly appeared, and nover in past fights. At the same time, you ignore what I write, in particular about the existence of a complete and consistent mathematical model that is both quantum and relativistic. It's getting really boring.

  134. Miracles

    Photons are unique in that they move at c relative to everything. For now let's talk about photons because relativity is about them.

    How fast is the arrow moving? And the part about the tree? And the center between them is a buffer?

    Each part of the rubber moves at a different speed (doesn't it?) and therefore the rubber moves at all speeds.

  135. Israel
    Entanglement is not just between photons. The particles can be electrons, crowded molecules and even small diamonds. Does your idea cover these as well?

    Regarding rubber: there is not a single particle there that has more than one speed.
    If a particle has any velocity, then it has infinite momentum - how does that work out? And also infinite energy…

  136. Miracles

    Think of a long rubber that is tied to a tree on one side and pressure-connected on the other.

    You shoot the arrow. How fast is the rubber moving? Not from 0 to linear arrow speed?

    I'm not saying that's what's happening, but such an explanation for the photon can explain many strange phenomena, including non-locality, postulate 2 and the paradox of the effect on the past.

  137. 1. "The article presents a certain tension at the philosophical level between private relativity and quantum mechanics. It is clarified that this tension does not necessarily lead to a contradiction, and it is explained that it can be resolved at the level of interpretation - that is, in philosophy. There is no mention of a physical contradiction (that is, a contradiction in the models)'.

    From the article:

    "The most alarming thing about non-locality, apart from the jarring strangeness inherent in it, is that this feature carries a huge threat to special relativity as we know it today. In recent years, this concern, which has finally gained entry into the hall of serious thoughts in the field of physics, has become the focus of discussions that may, in the end, tattoo, distort, reimagine, consolidate or disintegrate the very foundations of physics.'

    No philosophy, just physics.

    "The state of special relativity, only a little more than a century after it appeared on the world stage, suddenly became a wide open and rapidly developing question. This situation arose because the physicists and philosophers finally followed the blunt ends of that forgotten debate of Einstein's with quantum mechanics.'

    True, philosophers (so what?), but also physicists.

    So what is the difference between what the article says and what I say?

    2. "Obviously, measuring on one side affects the other side as well."

    So what's all the fuss about the gloves and the numbers that were written and sent to different ends? Is there any way that a measurement or some other operation done on a glove or on a number written on a note will somehow affect the other side?

    And what's the fuss about me lying by twisting your words? Didn't you say again and say that there is no difference between classical code and quantum code except that in classical code you can do different things? Do these different things include remote influence in zero time? Isn't this the root of the controversy?

    I have said many times that if you agree that in the interweaving one side affects the other side in zero time there is no need to call it information, you can call it Moshe.

    3. You always claimed that no information is passed because all the data is already in the system.

    So that you don't claim that I'm trying to fail you, in the Aspect experiment not all the data is in the system, the state of the polarizers is determined after the interlacing.

    4. Must run (poker!) and the subject is important and we will expand on it later, but here is the gist:

    After all, a photon has a clear and absolute momentum. Doesn't this require a complete lack of location?

  138. 1. The article presents a certain tension at the philosophical level between special relativity and quantum mechanics. It is clarified that this tension does not necessarily lead to a contradiction, and it is explained that it can be resolved at the level of interpretation - that is, in philosophy. There is no mention of a physical contradiction (that is, a contradiction in the models). It is my understanding that you are misrepresenting yourself by selectively quoting things like the title and ignoring the body of the article. But the truth is that I don't quite understand what you want from me because I don't think I have accused you of anything (apart from the conclusion that the article points to a contradiction between private relativity and quantum mechanics). The claims I made were about the article and not about your words. If there is something specific I said that relates to your words that you want me to explain or that you claim is wrong, specify what it is and I will address it. At the moment it seems to me that you are just looking for another fight by force or a general request that I forcefully find something we disagree about.

    2. Why? Measurement of an entangled state causes the state to collapse on all parts of the system. It is clear that measuring on one side also affects the other side. I'm just trying to explain to you that this effect is not an expression of information transfer. It expresses correlation, but these are two different things. I have no idea where you even got the idea that I deny one of the most well-known and studied phenomena of quantum mechanics of the twentieth century. You could equally ask me if I deny that a particle can be found in two different places at the same time.

    3. As I already wrote to you, I have no idea what you mean. If you want me to address a claim that I made before, it is only fair that you bring the claim to me including the entire context in which it was made, and I will be happy to try to clarify it. Giving me a four-word quote and telling me it's "about an assembly experiment", and then expecting me to be able to understand what it's about and explain it to you is either a bad joke or a deliberate attempt to fail me.

    4. Of course the uncertainty principle applies to photons. Actually, you already asked me this before and I already explained it to you. But I would like to explain again.

    Quantum mechanics as written in the twenties and thirties of the last century (its roots are earlier of course but these are the years of essential growth) is not a relativistic theory. In this theory, velocities are connected by a Galilean transformation, a change in potential causes an immediate change in the accelerations of all bodies in space, etc. That is, she contradicted her private relationship.

    Similarly, private relativity as written in 1905 was an extension of classical mechanics. In particular, the positions and momentums of particles are finite and can be known (you don't need to look at photons, you can look at just a massive particle of matter). Particles cannot be found in two places at the same time, cannot perform entanglement, etc. This theory is not quantum and it contradicts quantum mechanics.

    But already a long time ago - close to a century - people realized that these problems are problems in the formulations of the theories. It is possible to apply the principles of private relativity to their moorings in a theory in which position and momentum are not posits but operators with uncertainty. All the principles of quantum mechanics can be applied without exception in a theory in which the connection of velocities is not performed by a direct scheme. Already a few years after the publication of the Torah of Relativity, people realized that it was not a single Torah, but a set of principles and that any Torah that upholds it is "relativistic". Also, quantum mechanics is not a single theory but a set of principles and any theory that upholds them is called "quantum". Now the question could be asked, are there teachings that are both quantum and relativistic? The answer was already given about 80 years ago (I think Dirac was the first to ask it and made very significant progress in finding the answer). Of course the answer is yes - there is no problem writing a Torah that is both relativistic and quantum. In 1965, Feynman, Schwinger and Tomonga received a Nobel Prize for writing a complete theory, mathematically consistent and with high observational accuracy, which was both quantum and relativistic (quantum electrodynamics).

    In particular, if you take this theory and look at the mathematical limit where the speed of light goes to infinity (that is, all speeds are negligible compared to the speed of light and in this limit the Lorentz transformation is identified with the Galilean transformation), you will get exactly the quantum description of electromagnetism as it was known in the earliest version of mechanics The quants, before they knew how to combine it with relativity. If you take the same theory but instead of the limit of the high speed of light you look at the limit where Planck's constant goes to zero (that is, the quantum characteristic size is negligibly small compared to the size of any system), you will get exactly the description of electromagnetism in the framework of special relativity, as published in 1905 and before they realized that another formulation of it is consistent with quanta. These limits are not just an arbitrary invention: the limit of the speed of light going to infinity is exactly what links special relativity to Newtonian mechanics and the limit of Planck's constant going to zero is exactly what links quantum mechanics to Newtonian mechanics.

    In conclusion - if you look at the way Einstein wrote special relativity in 1905, you will see a picture that does not fit with quantum mechanics. But what to do, in 1905 nobody really knew anything about quanta yet, so it's no wonder Einstein didn't write the theory compatibly. Once quantum mechanics was established, people found perfectly reasonable ways to formulate special relativity without having to simultaneously know the position of a particle as well as its momentum, etc., and there was no problem writing relativistic and quantum theory.

  139. Come on, you can't say we didn't try.

    "You have lied many times, because most of your arguments are empty of content, because of your strong tendency to talk about things you have no idea about"

    So you claim that I consciously lied.. Nice. I always thought that physics deals with facts and logic, but here is a physicist with powers who deals with the occult knows things about me that even I don't know..

    Some questions for you:

    1. Can you explain to me and the forum what the difference is between what is said in the article and what I am saying?

    2. Are you claiming that measuring one entangled particle does not affect the state of the other particle?

    3. In the aspect experiment, is "all the data already in the system" as you have claimed several times to explain why information does not pass between the particles?

    4. And the question I presented (I really want to know): Does the uncertainty principle apply to photons? It seems to me that this is required, otherwise the Compton effect could be used to know the exact position and momentum of an electron (not sure but it seems logical).

    And if so - then how does this work out with the relativity on the basis of which position and momentum are defined for a photon?

  140. Israel,

    :). You know exactly what I'm thinking, so why are you wasting your time? Again technicality and no shoes, you have lied many times, because most of the time your arguments are empty of content, because of your strong tendency to talk about things you have no idea about. Just as is happening now, when you write more and more things about information when it is clear to everyone - including you - that you simply know nothing about information theory, including the most basic definitions of what information is or what is beyond information.

    Again, I will continue to address comments on physics. This is my right. you can answer me. That's your right. you can ignore This is also your right. You can try to drag me into fights. Do whatever you want.

  141. mmm…

    So you're claiming I lied, so I must fail a lie detector test, right? After all, that's what he does, discovers lies..

    Isn't it better to just apologize and close the matter? You know now that you're just trying to come off as entitled on a technicality and you have no way of knowing if I lied or if I believe what I'm saying (even if I might be wrong).

  142. 1. It's not the subtext you're missing, it's the text. You look at the title (by the way, I don't know if you know or not, but the title and subtitle are written by the editor and not the author of the article) and the entire summary line. Look at everything in between. Yes, when the authors of the article write that von Neumann gave a guarantee to the physicists that there is no contradiction, and from this moment on every division on this claim comes from philosophical arguments (first from a number from 94 and then from one of the authors), and the only mention of a more recent scientific reference is from a mathematician who chose Taking a different view than von Neumann and reaching the same conclusion - I see this as confirmation by the authors of the article that there is no division on coexistence among physicists. Not that I need such permission. All that is needed is to open any modern article on field theory.

    2. Yes, you definitely lied in the many discussions we had. It's funny of you to expect me to now know how to quote you exactly where, how much, why and how considering that these are discussions that some of them were 3 or 4 years ago, but I have a few examples in mind. One example is that for a long period of time you would go back and write a quote of mine over and over again, which miraculously would always get cut off in the middle of the sentence. I have explained to you many times that you are citing half a quote and thus completely changing what I said, but it didn't bother you. The first part of the sentence concerned the fact that there are people who do not understand quantum mechanics and therefore claim that it is wrong - and you have the same problem. The part you would have omitted made it clear that I am not accusing you of denying quantum mechanics but of the same kind of mistake - you do not understand something and therefore conclude that it is probably wrong. Another example is that during a long discussion about solving one or another coin puzzle with the help of interlacing, you claimed that I said it had a classical solution even though I explicitly wrote many times that it did not have a classical solution. I was just trying to explain to you that the quantum solution does not include information transfer because it requires the pre-synchronization of a huge number of entangled pairs - which is the quantum equivalent of pre-coordination of code, cheating according to the rules of the game. And yet you repeatedly accused me of claiming that there is a classical solution.

    You will of course reject these examples, although there are many more. But I really don't care. If you still haven't figured it out, I have no intention of starting a fight with you. Not going to apologize either, and not going to sign any contract with you. We both know exactly why you are trying so hard to push the discussion there - because you have nothing to say. Want to talk physics? it is possible Do not want? Your right, promise not to hold your refusal against you as people sometimes do. Want to write a hundred more times that you are a victim and that I am persecuting you? Probably so.

    3. Ok, everyone can make mistakes sometimes. Not bad. But I hope you realize now that even in Sakurai there is not a shred of support for your claims, let alone the relevant professional literature. And this is in contrast to what you wrote in the previous comment, which supposedly can be understood from Sakurai that there is a point in your words.

    4. "But how can we know if information has passed, useful or not, if a reviewer does not define what information is and you only learn this after a reviewer"? Very easy. We will go to a relevant book on information theory instead of an introductory book on quantum mechanics, we will study it, its definitions and the proofs of theorems that people have discovered. So it seems that according to information theory no information has passed. And as usual, I will remind you that if you feel like inventing a new Torah with a new definition for useful and non-useful information and all that, that's your right. At the moment the best we have is information theory and according to the understanding it provides us (which seems to be an excellent understanding according to its many successes in physics, computer science and engineering), information does not pass.

    "And in general, how do biologists talk with such confidence about "genetic information" that is inherited and without definition?" That's easy too. Biologists do not deal with information theory and they use the word information or information in the same way that you use it, in the everyday sense. Just as the historian can say about the Roman Empire that it was the strongest power in the world in its time, but that does not mean that it lived up to Newton's equation. The comparison between information as defined in information theory and information in a certain code (eg computer code, or analogously genetic code) can certainly be made, but it is not trivial. Again, the solution is by studying the Torah and its definitions *before* deciding that it is incorrect or trying to interpret the sentences contained in it.

    Is this related to what you claimed several times that "all the data is already in the system"? I have no idea what you are talking about. Perhaps you are talking about what I mentioned before, about how solving the coin puzzle using many pairs of intertwined spins requires that before the puzzle begins, the two participants sit down and prepare all the pairs of spins together, and even verify exactly in which intertwined state each pair is (because an intertwined state is not unique , there is always a lot of it that differentiating between them is not trivial at all). I don't know, maybe in this context I once told you that they don't pass information between them but that they already coordinated the information in advance when coding the intertwined pairs. But if you want a more specific reference I will need a reference to exactly what I said and the context in which I said it.

  143. Miracles

    Very simple: it is not possible to send information through interweaving.

    ELBENTZO

    "You received quite detailed explanations of why the article in Scientific does not say what you claim it says"

    The title of the article is: A quantum threat to the theory of relativity. The central thesis: "It seems that the type of non-locality encountered in quantum mechanics requires absolute simultaneity, which poses a real and deadly threat to special relativity."

    And that's the trouble.'

    The sentence that opens his summary: "The state of special relativity, only a little more than a century after it appeared on the world stage, suddenly became a wide open and rapidly developing question."

    Is there some subtext here that I couldn't understand?

    And are you referring to the following sentence:

    "For decades, the entire community of physicists saw von Neumann's proof as a sort of insurance certificate that the non-locality of quantum mechanics and special relativity can exist peacefully side by side."

    As a reference for the consensus in the academic community on the lack of contradiction between relativity and quanta? Did you read the sequel?

    I'm not calling them shameless liars… well, ok. By the way, they didn't lie as far as I know, so I don't know why I would call them that.

    So I guess I did shamelessly lie..

    Can you point to one and only one place where I lied - that is, that I failed the lie detector test?

    "You also said a lot of unkind things to me."

    Can you point to the one and only time I attacked you first? Because I can point to dozens of times you've done it.

    And if you or someone is going to come up with the kindergartener's argument, that is, it doesn't matter who attacked first and who responded - then is that intellectual willing to be attacked himself and shut up?

    "Are you talking about another place in the book? Or did you get confused between "not passing useful information" and "not passing useful information"?"

    I haven't looked at the book in a long time and I'm at work, but I guess you're right and the meaning is "not passing useful information".

    But how can we know if information has been passed, useful or not, if Scourai does not define what information is and we learn this only after Scourai?

    And in general, how do biologists talk with such confidence about "genetic information" that is inherited and without definition?

    Is this related to what you claimed several times that "all the data is already in the system"?

    We - soon we will return to the distant galaxy where we can discuss freely.

    Do you also encounter a problem with typing the texts? I thought it was only on Mac, but I see it is also on PC.

  144. Albanzo
    Thanks!
    According to my understanding - what the theory of relativity does not "allow" is a contradiction. Mario Livio describes a thought experiment in which a landing plane collides with a vehicle crossing the runway (watching at the end of a runway). If the speed of light were the same as the speed of the plane, then from the observer's point of view they would not collide, which is of course a contradiction.
    But - there is no restriction, as long as there is no contradiction. In particular - interweaving does not create a contradiction.

    And in the context of what I said, because it is a random process, it makes no sense to say that "information passed from one side to the other". On the other hand - we (at least I) don't really understand what's going on there.

  145. One more thing: I wanted to read in Sakurai about what you said ("transfer of useless information") to try to understand what happened there - does he use the word information simply in an everyday way (he does not define information as a mathematical concept and even approaches the subject in the whole book), or maybe not You got something right. I re-read all the relevant parts of the book and even ran a word search. I didn't find anything like that. All I found was a place where it says that no useful information was passed (which is *very* different than "unuseful information was passed"), and a few lines later it says that no information was passed at all. I think that this careless wording was done simply because the book does not distinguish between correlation and information. This is a second edition, page 245.

    Are you talking about another place in the book? Or did you get confused between "does not pass useful information" and "passes useless information"?

  146. Miracles,

    1. I don't think so. In principle, information theory defines information and talks about information in the context of random variables (classical theory, of course. In quantum theory, everything is a little different). Every random variable is basically a lottery which after its execution only one result is obtained, therefore the phenomenon you mention is common to all information and not only to interweaving. In fact, the information is precisely related to this "collapse" - in a certain sense, it describes how expected it was that out of all the possible results, the result obtained was the correct one.

    2. I don't think I understood exactly what you mean. Can you try to clarify? What is special about interweaving is that each part of the system is not well defined on its own, so every measurement is actually done simultaneously on both parts. What actually causes there to be no contradiction to general relativity is that quantum mechanics is a statistical theory and can be "superimposed" on one of the parts of the system. This averaging shows that someone at one end of the system is not actually receiving or giving information to someone at the other end. I'm not sure if that simplified or complicated the matter…. In any case, I, like you, claim that there is no contradiction between private relativity and quantum mechanics, and so does the whole world of physics. As you probably know, for the last seventy years (more or less) the dominant theory has been quantum field theory, which has been repeatedly found to be an unusually accurate and phenomenal description of reality (although we know for sure that it is not a complete description because it cannot describe gravity, and therefore is at most a holography of reality). This Torah is explicitly quantum and relative (private only) and no contradiction has ever been found in it in this context. Therefore, even in the article brought by Israel, which challenges the ability of the theories to live in peace with each other, it is explicitly written that among physicists there is a consensus that there is no problem, and the claims that are made there (which, contrary to what Israel said, I am not laughing at them or belittling them) are philosophical claims, and as which the writers say explicitly, they can be solved by accepting a different interpretation than the accepted one regarding the reality in which we live.

  147. And by the way, at no point did I "snicker" at the article in Scientific. I said that he was indeed more philosophical and not so scientific, but I didn't make fun of him and even wrote that he was interesting. If you read it, you will see that the authors also agree with me - all their arguments regarding the possibility of a certain tension between the theories are philosophical arguments. They themselves do not pretend to find any mathematical contradiction, and even talk about the fact that in recent years mathematicians are finding more rigorous ways to show that there is no contradiction (more rigorous than the trivial ways proposed in the XNUMXs and XNUMXs, such as von Neumann's).

  148. Israel,

    I'm not going to apologize to you. Contrary to what you are saying now, the things I said to you I did not say to you because "you dared to express an opinion different from mine". I told you you were a fool when I thought you were acting like a fool, etc. You also said a lot of bad things to me. I'm not going to apologize and I'm not going to get into personal arguments again. I thought you weren't interested either. If I was wrong - enjoy, you can write scrolls about how much you are a victim and that just one day I came and abused you because I didn't like your name or some other nonsense. I don't think I will respond to these things.

    Regarding physics - 1. You received quite detailed explanations of why the article in Scientific does not say what you claim it says, and what it does say. All you have to say is that I don't call them shameless liars… well, ok. By the way, they didn't lie as far as I know, so I don't know why I would call them that.

    2. You are just repeating the same mistake. Brings a quote from Wikipedia but does not understand at all what he says. You don't know what observer is or what it is to communicate (of course you know what the words mean, but don't know what they define in the mathematical theory of information theory). So you take a sentence and interpret it however you like according to your everyday understanding of those words. If I am wrong and you do know what these concepts mean, please explain to me at the mathematical level what my mistake is.

    3. Of course, Sakurai is just a straw man whose goal - and I believe probably unconsciously - is not to deal with your lack of knowledge in information theory. As has been explained to you many times before, Sakurai does not deal with information at all. He is not "mythological" and the only reason you know this name at all is because, in a completely unrelated way, I tried to make you understand that you have knowledge gaps in quantum mechanics that do not allow you to understand the subject of entanglement and information at all , like a person trying to understand composite analysis without knowing anything. That's why I kept referring you to Sakurai, and tried to make you understand that this book, which reflects the knowledge of a student at the end of the first degree, is still a long way ahead of you. And I didn't say it then to insult and neither now, just to make you understand. In other words, all your perceptions of one or another word (note, theoretical physics is a well-defined mathematical system - words do not have much meaning in it) in a book that does not pretend to say anything or half a thing about quantum information, is just an escape from reality. Go to the texts that deal with the subject, see what information is, see what information transition is, see what it predicts. So you can see that what you call "unknown information" is not necessarily information according to information theory, and therefore its "transition" is not necessarily information transfer according to information theory. Your whole argument is based on the fact that you are starting from an intuitive-everyday assumption where any correlation between two situations indicates a transfer of information between them, but information theory explicitly defines information and correlation, according to which a correlation that does not include a transfer is possible.

    As I have told you many times before - if you have an idea for a better information theory, with other definitions according to which there is an unknown information transition (and which reproduces the known successes of the old Torah), you are welcome. I will have fun learning something new. But to take the old Torah, with the existing sentences in it according to certain definitions, and then try to apply them while changing the definitions and inventing new concepts that are not well defined at all - this is a joke.

  149. ELBENTZO

    I asked you to apologize for all the insulting nicknames you called me for daring to express an opinion different from yours. It's interesting by the way that Scientific American claims what I claim almost verbatim, and even though you try to deride their position as "philosophical" and not scientific, at least you don't call them shameless liars, dumb trolls and seniles spreading supposedly "scientific" garbage who can't deal with the fact that they don't understand What twenty-year-old children understand easily.

    And as a matter of fact, from Wikipedia:

    In physics, the no-communication theorem is a no-go theorem from quantum information theory which states that, during measurement of an entangled quantum state, it is not possible for one observer, by making a measurement of a subsystem of the total state, to communicate information to another observer

    If anyone has read the example of coins and radios that I brought, you can clearly see what I am claiming: the no-communication theorem refers only to the second part - that is, to the sending of the information using the device that I proposed with the radios that reproduces the scene, and not to the first part, how the radios communicate with each other no -communication theorem does not refer to the subject at all.

    Regarding unknown information: the mythological reviewer, although it is claimed that it is only learned before studying information theory, talks about the transmission of useless information by interweaving (and this without defining information or useless information).

    I believe Skorai's "useless information" is the same as my "unknown information". I won't be able to provide a rigorous mathematical proof, but Skorai doesn't do this either and uses the concept without a definition, so maybe I'm fine..

  150. Albanzo
    I've got two questions.
    Suppose my wife lost my keys at home. As long as I haven't found them - there is a certain probability that they are in all kinds of places in the house. I know what the probabilities are: from experience, the keys are usually in a drawer, sometimes in the pants pocket, and sometimes between the cushions on the couch.
    As soon as I found them - all these probabilities are reset. Does this example intuit the state of interlacing?

    The second question is related to what I said to Israel - is the important point the randomness of the measurements? It is true that there is interweaving between the states, but each state in itself is random (say, polarization of photons).
    I look at it as if it were actually a single photon found in two places - I don't see it as a contradiction to the theory of relativity (causality, etc.).

  151. Israel
    How do you understand the following sentence (from Wikipedia)
    "In these experiments, the no-communication theorem shows that failure of local realism does not lead to what could be referred to as "spooky communication at a distance" (in analogy with Einstein's labeling of quantum entanglement as "spooky action at a distance" )”

  152. Israel,

    Or maybe I should actually direct the response to Nisim and the other people who may be reading the discussion and want to understand, because Israel does not want to talk to me and that is of course his full right. My goal is not to drag him into a debate but to provide information to those who are interested in quantum information - and those who are not interested (or who are interested but do not want to hear from me personally) can ignore.

    No-comm sentences are mathematical sentences. They prove certain claims about information theory. Of course these claims, like the conditions for the existence of sentences, are formulated in the language of a logical system on certain definitions. That is, there is a precise mathematical definition of what the "information" is in the context of the sentence, what is "transfer of information", etc. It's just ridiculous to try to apply these theorems, but replace the definitions with other definitions (especially since those other definitions are not mathematically well-defined). Those who are really interested in the subject can learn the basics of information theory and prove to themselves that under its definitions there is no "known information" or "unknown information". Information is a quantitative quantity that is defined without any dependence on the "observer" and when the no-comm sentence proves that information cannot be transmitted, this is also true for a conversation between two friends who decide what to say to each other, and also for a radio signal that has no control over a system but can communicate to an end user Track the state of his coin and activate accordingly a machine.

    Again, in conclusion - the no-comm sentences really do not say what Israel claims. His claim is based on a misunderstanding of what information is or passing information under the definitions of the mathematical Torah, and replacing the correct concepts with the intuitive concepts that fit the way of thinking of most people in everyday life.

  153. Vicky

    Unresolved paradoxes in relationships, including Ehrenfest paradox, are not known. But it is without a doubt strange and if you read the article, maybe it conflicts with the quants, and if you read the link to the presentation in Santa Monica, with some other theories.

  154. A round rail and it goes up.

    intuition? We are not dealing with quanta and relativity, so what does intuition have to do with it?

    I'm just trying to show how strange the shortening of the length is (see how complicated we got and finally came to a wrong conclusion, see how complicated 4 professors were in the twin paradox paradox and only one found a logical answer, all the others gave contradictory answers) to show that a solution of a photon moving at all speeds solves A lot of problems despite its strangeness.

    "The very fact that it is impossible to transfer information in this way shows that something is happening here that is not a classic "transfer of information".
    It is true that the two coins always fall on the same side, but we have no way of influencing which side..."

    Here is an explanation that I have given before.

    Consider coins in different rooms without communication between them. Is there a way to make the coins always fall on the same side? Negative.

    Now connect the coins to the radio, and when one of the coins falls on a tree or field, this information goes to the other room and a mechanical mechanism immediately arranges the other one in the same situation.

    Now of course you can make the coins always fall on the same side. Was information passed between them? Positive. The radios are the ones that transmitted it.

    Is it possible to send information using this device? Negative. To be able to send information we must be able to influence which side the coin that fell first will fall and determine the state of the system. Since we cannot do this, the side on which the coin fell is unknown information to us, so we cannot send information in this way.

    Only the second part, sending information, is talked about no-communication theorems. What I'm talking about is the first part, with the radio.

  155. Israel
    I agree with you that it is not intuitive.
    But - the very fact that it is impossible to transfer information in this way, shows that something is happening here that is not a classic "transfer of information".
    It is true that the two coins always fall on the same side, but we have no way of influencing which side...

  156. The train travels on a circular track like a toy train and fills the entire track and of course a radial force and radial acceleration act on it.

    If the radius is small and the speed is high, it will fly off the rails. But the radius is so large that for her it travels almost in a straight line.

    What will happen when it reaches the gamma factor 2? This is the same case as the wagon question we talked about earlier.

    Previously we thought that between two clocks on the track at a given moment there would now be twice as many cars due to the shortening of the length, but this is impossible because it means that certain cars will exceed the speed of light.

    But that's not what will happen. There will be exactly the same number of crores (we talked about a billion) but there will be spaces between them or they will tear, see the Bell Paradox.

  157. I think I understand what's going on with the wagons.

    No (happens).

    The solution is the relativity of simultaneity.

    To see this, let's think of a huge circular track on which there is an equally huge circular train. For each length unit on the track, x cars fit.

    At a certain moment the train starts to accelerate and reaches a gamma factor equal to 2. Because of the huge radius the radial acceleration is negligible.

    The logic says that because of the shortening of the length, when the train reaches a constant speed then 2x cars will now fit each unit of length on the track, but this is impossible because we will get double the cars before and this is impossible.

    What will happen is that because of the instantaneous relativity the carriages will be shortened. If they are separate and self-propelled, they will separate from each other and spaces will be created between them, but still x cars will fit per unit of length on the track.

    If they are connected to each other like in a normal train, then they will just snap and come apart, like the rope in Bell's Spaceship Paradox.

  158. It seems to me that perhaps the explanation is aversion, that is, because of the shortening of the length, more particles with a positive charge affect and more strongly. I'm trying to see if that will solve the carriage problem as well, but for now aloha

  159. I recommend you drink a lot of water.

    And I don't see why the number of electrons should increase, as in a closed system of pump and pipe the amount of water does not increase.

    What's more, the effect you mentioned is not relative.

    In the video, by the way, you can literally count the amount of charges (12 of each type when the cat is at rest, 16 positive and 8 negative when it is moving).

    And since you asked what my explanation was (I don't have one), but I can guess what Lesage would have said.. (where would Yoda really be?).

  160. Israel
    Give recommendations - I'll be there in a month.
    I first want to understand what Albenzo wrote... on the face of it - it seems to me that it is not worth discussing superconductivity because it is a quantum phenomenon.
    In the classical case the number of electrons must increase because otherwise there is no current (Ohm's law). I'm not sure I'm right, because in general it is possible to create a current in the wire by another magnetic field (say - 2 distant rings, when you move a magnet near one ring, then a current is induced in the other ring as well, and here you really don't add electrons.

    There is a lot to understand here 🙂

  161. Hawaii baby, Hawaii.

    Miracles

    You can use a negligible KM, but for you special - a ring around absolute zero, the current will not stop for several days even without a KM.

    And I didn't understand the density thing. After all, the whole point is that the density per unit volume (or length) increases due to the contraction of the length and there are more positive charges than negative per unit volume and therefore a force is created. You see this clearly in the video around the second minute, where it is extreme and the number of positive charges per unit length is double the number of negative charges, while before they were balanced.

    So if this is the case throughout the closed circle, where did so many additional positive charges suddenly appear?

  162. Albanzo
    Thanks for the explanation, even though I only understood the beginning. I understand this with the following analogy: if I am near a train track and feel gravity from the train, then this gravity will not increase if the train moves at a relativistic speed.
    I understand it right?

  163. The number of electrons is not their density. The number of electrons is the integral of the density over the volume (or the area, or the length. It depends on which density it is, of course). One can see in general - without specifically looking at a ring or disk or cube or wire - that the Lorentz transformation on the density and the Lorentz transformation on space offset each other, so that the integral is invariant to the transformation. On a mathematical level, this is due to the fact that not only the integrand undergoes a transformation, but also the degree of integration must have a Jacobian and it can be seen in general that the Jacobian matrix is ​​the inverse of the transformation matrix. That's why it really doesn't matter if a certain transformation that turns a circle into an ellipse or a square into a rectangle or a watbar looks strange when you look at its density. If it is consistent with the Lorentz transformation, the number of particles will not change. Of course, all of this is only true in a private relationship where the space is flat and the viewers are all connected to each other at a constant speed. If the space is not flat or if some observers are accelerating, you have to look at general relativity and there the story is completely different (the number of particles is definitely not constant, and you even see it experimentally).

  164. Israel
    To cause a current in a wire you need a voltage along the wire. This means that there is an excess of electrons in the wire (this is no different from blowing air through a pneumatic tube).

  165. On the plane, on the way to Hawaii, 5 more minutes before take off.

    So there is no way that a car that is a light hour away will reach the photo within 10 seconds.

    And regarding the crowded electrons: if you think about a ring in which an electric current flows, then how can the electron density increase at some point on the ring without decreasing at another point? After all, the number of electrons is fixed on the ring..

    As I said, extremely strange.

  166. Leave the photons for a moment, let's finish the issue of the wagons.

    If when you started you had cars in front of points 1 and 2 that are separated by a billion kroner, and when you finished you got cars in front of the same points that are separated by 2 billion kroner, then where did the cars come from that before the acceleration were at least a light hour away from the two points. do you get that It has nothing to do with relationships.

    If not - explain to me where they were before.

  167. Israel
    Let's consider a simple case. There are 3 points on the track, a to my left a light year away, b in front of me, and c a year away to my right. At moment 0, a photon exits from M and also from M, a photon exits towards the right. After a year, the photon Ma will reach me and the photon Mb will reach L. That is, the distance between the photons remains a light year.

  168. But where was kron x plus billion times 2 before the acceleration? Not a light hour away from the clock he reached within 10 seconds of the start of the acceleration?

  169. Israel
    I wrote you take:
    "Sharel
    Let's ignore acceleration first. In the first step, in front of point 1 is kron 0 and in front of point 2 is kron billion.
    Now - let's assume that a second train moves at a constant speed of gamma=2. In this situation, if in front of point 1 there is kron x then in front of point 2 we will see kron x plus a billion times 2."

    The problem is that I didn't write "the car number is x plus a billion times 2"? are you in kindergarten

  170. You are called to order miracles.

    What is so difficult about answering something like "the car tax is x + two billion?"

    Never mind, I guess that's your answer, but here's the problem: Car tax x + two billion was away from camera 2 about a light hour before the train started accelerating.

    Since the whole process only takes 10 seconds (from time 0 to time 10) he has no possibility of reaching camera 2 in time.

    And it doesn't matter how you play with the data, if you reached a situation where within 10 seconds of the start of the acceleration there are cars in front of the 2 cameras at the same moment in both of them with 2 billion cars separating them, then one of the parties reached within 10 seconds a car that was a light hour away from it.

    Do you see another option?

  171. Israel
    I already answered you. If the speed of the Gamma train = 2, and there were a billion krones at speed 0, then there would be 2 billion krones.

  172. compatible, compatible

    But not related to my question. No acceleration either. The question is: which car will be photographed by camera 2 at time 10? She will take a picture of some trailer, won't she?

    The train has long since reached equilibrium and is already an inertial system moving at a constant speed of 0.87c, less than your muons and also took much longer to accelerate than them.

    So what is the car number? You don't have to be precise but give a reasonable estimate because after all, what are fifty-sixty million kroner between friends?

  173. Israel
    Is what I wrote true or not? Acceleration is a complicated matter, so let's stick to constant speed. Is what I said consistent with special relativity?

  174. Let's make it simple:

    At moment 10 at point 1 the sharp resolution camera at point 1 takes a picture of car x on the train opposite.

    What is the number of the car that the camera will take at point 2 at moment 10?

    And to add insult to injury, don't forget that according to simultaneity the train actually lengthens, not shortens (remember Bell's paradox of ships? But lengthening doesn't solve the problem at hand either).

    Whatever the solution, and I can't find one at the moment, it's extremely strange.

    And this is the point I made regarding the shortening of the length, it is very strange.

    So try to say what the car number will be approximately in front of camera 2 at moment 10, and that's without discussing whether non-locality does not collapse relativity as the article in Scientific American claims, not in alternative physics.

  175. Israel
    Let's ignore acceleration first. In the first step, in front of point 1 is kron 0 and in front of point 2 is kron billion.
    Now - let's assume that a second train moves at a constant speed of gamma=2. In this situation, if in front of point 1 there is kron x then in front of point 2 we will see kron x plus a billion times 2.

    I agree with that.

  176. Oh miracles, really.

    1. At moment 0 on the track clocks, when the train is at rest, there is car number 1 in front of point number 1 on the track and car number 2 billion in front of point number 1 on the track which is one light hour away from point number XNUMX.

    2. The train accelerates in five seconds according to the train clocks so that gamma is equal to 2.

    3. And now you tell me: if at the instant of 10 seconds according to the clock point 1 on the track there is a car number x in front of it, what will be the number of the car standing at the 10th moment on the track in front of point 2? Not x + 2 billion?

  177. Israel
    Ok, you say the train started at a standstill, and accelerated within seconds to gamma=2. That is, from 0 to 260 million meters per second. From the point of view of an observer from the side, the 2 ends of the trains move at different speeds, the last cars are much faster than the front ones.

    Again - I really don't see a contradiction here.

  178. Miracles

    You miss the point.

    If within a few seconds on the track clocks which are also a few seconds on the train clocks the train is shortened by half, then if you take two points on the track and take a certain time on the track clocks (say 0) then between those two points a certain number of cars (say a billion) are trapped at the beginning and after a few seconds In the rail clocks (say 10 seconds) 2 billion.

    You can see this clearly if the cars are numbered: at moment 0 there is opposite point 1 on the track car 1 and at moment 0 at point 2 on the track billion car.

    At moment 10 at point 1 on the track there will be car x in front of it and at moment 10 at point 2 on the track there will be car x + two billion.

    Where did all the cars come from in such a short time? Just that.

    Elbentzo.

    I have no intention of arguing with you before you apologize for all the vitriol you have poured out on me in recent years (remember? Liar, idiot, moron, ignorant, senile, etc.). And this is only the first condition.

    The sad history proves that you immediately get angry when your opinion is disagreed with, so what's the point of discussion? Get into another fight?

    And no, Einstein was not wrong in the EPR paper, but I have no intention of getting into a fight again. Tomorrow we are going to Hawaii, sorry for the nerves.

  179. I now took a few minutes to read the article from Scientific and also do a little research on the sources it refers to.

    First of all, it should be remembered that every article in a magazine - even a scientific one, and certainly in Scientific American, which addresses the general public and has a huge interest in selling copies - can fall into a natural tendency for sensational or bombastic headlines. This also happens in scientific articles. So there is no point in looking only at the title, and you need to turn to the body of the article.

    The first thing that jumped out at me is that the two authors are a philosopher (admittedly a philosopher of science, but in any case not a mathematician or a physicist) and a writer. Right from the start it is clear that this is not necessarily a good source to learn physics from. I will preempt the later by saying that I am not claiming that they do not know what they are talking about, because most of the article concerns philosophy and not physics, so there is no problem. But if we are talking about physics, this article should be no more than an incentive to read real articles and research by physicists.

    Now to the article itself. The first part of the article (about 75%, I didn't count pages) just does a historical review of the background of entanglement and special relativity and ends with the full consensus of physicists that there is no contradiction between special relativity and quantum mechanics. After this part, comes the second part, which has three parts:

    1. Claim that a philosopher named Modlin wrote a book in 94' in which he pointed out a problem (not a contradiction as a fine point) in the combination of special relativity and quantum mechanics. Apart from the fact that this is a non-scientific and peer-reviewed book, the problem is not exactly explained. There is a brief explanation of what the problem is not, and then the problem is formulated with the requirement of "absolute simultaneity". I personally don't know how it comes up, but I haven't read the book.

    2. Reference to the work of a mathematician (the quote that Nissim brought earlier) who, for free theories, shows more rigorously than von Neumann that there is no problem in combining entanglement with special relativity.

    3. Reference to the work of one of the authors. The work is not explained in detail, but from the little that is explained, it is clear that it walks a very thin and dangerous tightrope, which is a reference to the trivial geometry of private relativity. Private relativity is a geometric theory from which any deviation, even the smallest of the smallest, requires the use of general relativity. General relativity is a theory that is explicitly not quantum (unlike special relativity). Therefore a combination of quantum mechanics and special relativity can only be performed under conditions of fixed geometry that does not interact or fluctuate at all. I mean, there are very subtle things here that even many physicists who don't study gravity can get wrong. But even if everything is done correctly, the problem it presents is purely a philosophical problem that can be solved by accepting the configuration space as a physical space. In other words, to understand that our history is in the Hilbert space and not in the spacetime space.

    In conclusion, it is very, very difficult for me to see how in the content of the article there is any risk of combining special relativity and quantum mechanics. A combination made explicitly in quantum field theories, which have been mathematically consistent (in this context) for about 60 years and whose experimental accuracy sometimes reaches millions of percent. I wouldn't take the title too seriously.

  180. Israel
    Let's assume that all the train cars start accelerating at time t, according to the train clock (there is no problem synchronizing the clocks this way). From the point of view of an observer on the ground - the cars do not start accelerating at the same time. In particular - an observer on the ground will see the last car accelerating before the first car accelerated.
    Therefore - strange, but there is no contradiction here.

    I will try to simplify the case to make it clear. Let's imagine that there are only two distant cars connected by a rigid rod, 2 light seconds long, and the train is moving at speed v. In the center of the pole there is a flash - and every car accelerates as soon as it sees the flash. The flash is activated at time t. As far as the cars are concerned - the light reaches the cars at time t+1, they start moving together and the distance between them remains 2 light seconds.

    From the point of view of an observer on the ground. Let's assume that the viewer sees the flash also at time t (at about....). For him, the light reaches the rear car at time (c+v)/1 and the front car at time (cv)/1. Therefore, the rear car accelerated first and therefore the train appears shorter.

  181. Israel,

    For your last question you already got an answer from me before, but it was a long time ago so maybe you don't remember.

    Although you don't want to think about it, Einstein was wrong. There are many non-local things in the world, and many things that move faster than light. Not everyone contradicts special relativity. For example, energy of a system is a non-local quantity. An example that is always given in bachelor's degrees is a flashlight that shines on a screen very far away from it. Then the flashlight is shifted to the left by twenty degrees, and this shift takes, say, half a second. If the screen is far enough away then the spot of light will move a distance of more than half a light second, meaning it will move faster than light.

    Einstein thought that a change of quantum state over a distance is probably mediated by a particle and is therefore from the family of things that do contradict relativity. What to do, even Einstein is wrong. And even a lot. You may know that Einstein would publish an article every week or two, and for half of them he would later publish an apology and a correction because he found out he was wrong. Today we know that quantum mechanics is actually not exact, and the fundamental objects in the quantum world are not particles but fields. Fields allow correlation that is not mediated by particles and can be non-local without contradicting anything in special relativity.

    I don't think it will convince you, but if you really want to get into the thick of it - you shouldn't ignore the complete consensus that exists today (after more than 80 years of smart people like Einstein who discovered a lot of things he had no idea about) regarding the subject.

  182. hahaha.. so you took care of a marginal article and skipped the main thing?

    Einstein says that if we perform a certain experiment we will get results consistent with relativity and therefore the quantum theory is wrong.

    The experiment is performed. Quantum is correct. So what about the relationship?

    And let's get over the fact that he didn't understand the meaning of the experiment. He is Einstein and he invented it.

  183. I thought a bit about the relativistic explanation for the attraction between the conductive cables. Apparently this is a slam dunk, but if you think a little deeper, the constants of electricity and magnetism are already embodied in the speed of light and the relativistic mathematical treatment only extracted them from there.

    And this is in contrast to Maxwell's model which took 3 seemingly unrelated constants - the constants of electricity, magnetism and the speed of light - and linked them in an elegant formula. It's a slam dunk! (Not slam junk!).

    But the relative explanation illustrates what I said about the shortening of the length: it is very strange.

    To see this, think about the explanation: due to the movement of electrons, the density of charges increases.

    So let's think about a very long train that is on an even longer track. At a certain moment in the train clocks all the cars accelerate within a few seconds according to the track clocks so that the gamma factor is equal to 2.

    According to the relativistic explanation, the train is shortened and the density of the cars is doubled.

    But it implies that if we take two points on the track a light hour apart from the one between which there were a certain number of cars, then within a few seconds there will be twice as many cars, right?

    Where did so many wagons come from in such a short time??

    As I said, very strange.

  184. Israel
    If you didn't understand, I'm one of those people who try not to come up with alternative theories for things I don't understand.

    This bland technical stuff is exactly what brings down many alternative theories - and your article on the GPS is just an example. The friction in Pushing Gravity is another bland technical detail, which invalidates the whole idea.

    I don't understand enough physics, or psychology, to explain why someone gave a certain title to an article, especially when the content of the article does not correspond 100% to the title.

    I'm not ruling out discussion on these issues, and I'm not saying you shouldn't come up with a crazy idea. But this is good for academic exercise only and should not be mixed with real science.

    On the one hand, you bring all kinds of reasons to invalidate a theory, and on the other hand, every time you try to explain why the reason is wrong - it's just a minor technical detail. The article about the GPS is a good example of this - "don't confuse me with facts".

  185. Miracles

    I was just Googling.. The article I linked to earlier is from Scientific American and is titled: A quantum threat to special relativity.

    So what's your theory as to why you don't refer to the title of the article but deal with bland technical details?

  186. Israel
    The article you linked to is interesting. He is also wrong. There's a reason it says alternate physics there...

    This highlights what the problem is with all the alternative theories - yours, Yehuda's and even my friend Raphael's: lack of knowledge. When someone who understands says something you don't understand (Albanzo for example) then you can come up with an alternative theory, or you can open a book and learn.

    If it's interesting - I can explain the mistake in the article you linked to...

  187. Raphael,

    1. The fact that we know with a very high degree of certainty that something cannot be true because it contradicts quantum mechanics (which, as far as we know, is a very good description of reality), does not mean that we know what is true. If you want insights into the question of what might have happened in the very early universe according to quantum gravity theories, you can go back to the video you yourself brought here a few years ago where very talented physicists explained what their ideas were and how they could be tested. As of today we do not have an unequivocal answer to the question of what happened in the early universe considering quantum mechanics.

    2. I gave an example of a singular point because I explained to Nissim about singularity as a general topic in mathematics and not in the context of the bang. I did not say that this phoenix describes the scale of the Big Bang or that it is related to the model at all. I didn't even say that the Big Bang scale is singular. I said that the conformal copy is singular at the point a=0 because it is degenerate and copies all the points of the space to one point. I don't want to sound condescending or condescending, but if you want to understand the technical side of the Big Bang model (or any other model of modern physics), you have to study. There are many things that can be more or less explained in the language of popular science, but a technical understanding requires knowledge of the mathematics behind the model.

  188. Albanzo

    Sorry to go back a bit but I still don't understand this:

    1. You wrote "a universe of size 0 is not necessary for the model and it is likely to be assumed (in fact, quite certain) that it will not be accepted as a solution of quantum theory for reasons of localization. "

    What does that actually mean? That there was always something there, only that it began to expand at a certain time?

    2. Regarding this "a trivial example of a phoenix that is truly singular is one of the parts of z. It has a singularity at the point z=0.”

    Singularity should give scale zero. 1/0 does not give zero. How is there a singularity here? (I omitted the topic of infinity because earlier it diverted all attention from the main point)

  189. Israel
    I didn't understand what you were saying about the muons. The slowing down of time was tested in both directions - with GPS satellites for example.

    Again, in your article it says that there is a possible explanation for EPR that does not contradict relativity.

    I am not saying that this or that Torah is correct. I do say that there is evidence of the shortening of the distance.

  190. Miracles

    Did you read the last paragraph in the article?

    The status of special relativity, just more than
    a century after it was presented to the world, is
    suddenly a radically open and rapidly developing
    question. This situation has come about because
    physicists and philosophers have finally followed
    through on the loose ends of Einstein's long neglected
    argument with quantum mechanics

    I can begin to try and explain every question you ask - for example the muons you mentioned were accelerated and moving against the background radiation, while according to relativity they are also in the opposite situation, i.e. if they were stationary relative to the background radiation even then their internal clock would be ticking more slowly relative to a synchronized system that is in relative motion to radiation and this has never been demonstrated in an experiment.

    I can also point out that Newton's theory also works and has been studied and proven to be correct and accurate for hundreds of years, and despite that it is not true but a special case of relativity at low speeds.

    But the truth is that how do I or anyone know. We're not supposed to know either. I believe we both agree on one thing: Einstein writes a paper - EPR - in which he tries to show that quantum mechanics is wrong or incomplete because (according to Wiki at least) it contradicts relativity. He also offers a thought experiment to prove his point.

    The experiment was conducted in reality (Shimaoni, Aspect), and the results are consistent with quantum mechanics.

    So what does this say about the relationship?

  191. Israel
    Quote from the article you linked to:
    The first result appeared in an astonishing
    2006 paper by Roderich Tumulka, a young German
    mathematician now at Rutgers. Tumulka
    showed how all the empirical predictions of
    quantum mechanics for entangled pairs of particles
    could be reproduced by a clever modification
    of the GRW theory (recall that this theory
    proposes a philosophically realistic way to get the
    predictions of quantum mechanics under many
    circumstances). The modification is nonlocal,
    and yet it is fully compatible with the spacetime
    geometry of special relativity.

    Read the last sentence again….

    But let's go - please explain to me how a muon travels 6000 meters in 2 micros, without shortening the distance.

  192. Israel
    So let's get started
    1) In connection with shortening. Let's assume that the size of the star is such that there can be an exact eclipse of the star by the moon. Thus a point on the circumference of the moon and the corresponding point on the distant star. These points are on a straight line from us, say at a conic angle a. At any velocity v of the observer (toward the center of the moon/star), the velocity to these two points is v times cosine a. This means that the distance to the two points will be shortened by the same factor, so they will remain on the same line. To illustrate this - draw the situation on a plastic surface, and stretch the plastic along one axis. The stretch is an affine transformation, one of whose properties is that points on a straight line will stay on a straight line.

    2) What's new? If we have two conflicting teachings, one simple and one horribly complicated, then it is more likely that the complicated one is wrong. Hopefully, we can find a theory that combines the observations from both fields.

    3) Maxwell's calculation for the speed of light is based on two constants (epsilon-0 and mu-0) for which he has no explanation. According to Maxwell, the speed of light must depend on the "absolute" speed of the light source, and this has no observational basis.
    Newton's equations also work under certain conditions. …

    4) We know there is a preferred frame of reference for the universe, and that's exactly the temperature you're talking about. This does not contradict the theory of relativity.

    5) It is very mature of you to quote as proof a statement of a person whose opinion you disagree with 🙂 Einstein believed in locality, and observations show that he was wrong.

    Israel - do you have another explanation for why conductive wires are attracted to each other? And let me give you another example.

    Muons are created by cosmic ray collisions in the upper atmosphere (about 20,000 feet), and their average lifetime is about 2 microseconds. Their speed is about gamma = 10. For us - they live 20 microns, and can move 6000 meters, which means we can discover some of them on the ground.
    But - as far as they are concerned, they only live 2 micros! How do they travel a distance of 6000 meters?? Hmmmmm…. What will happen if we divide the distance by 10? We will get 600 meters 🙂 And here - about half of them will indeed reach the Holy Land.

    If you do not agree with the explanation - what is your explanation?

  193. Israel
    And it is possible that each electron reaches out to the electron in the other wire, and of course it can only do that when both electrons are moving in the same direction.

    The relativistic prediction for the shortening of length in special relativity is a direct result of a very successful theory, which does not contradict any existing observation, and is even easy to understand. And there are also observations that confirm the prediction.

    Why not get it? The shortening of time and the curvature of space yes, and precisely the shortening of length no?

    We all know that relativity and quantum theory don't play nice with each other, but why exactly does the shortening of the length bother you?

  194. QED is a relativistic theory and includes within it all of private relativity, including various abbreviations/lengthenings. Maybe I'm missing the point.

    And just for general knowledge - miracles, the proof of the natural column scheme for minus one divided by twelve with the help of manipulations of columns that do not converge strictly (like for example 1-1+1-1+...) is heuristic and nice but is considered a bit of a toy proof in mathematics. By modern standards it would not be called a proof, because it assumes that the algebra of the series (which are not strictly convergent) is well defined and that is simply not true. There are of course rigorous proofs that lead to the same answer that can be reached as simply as you suggested. For example, by analytically continuing the column and performing regularization using Riemann's zeta function.

  195. You might mean this:

    http://galileo.phys.virginia.edu/classes/252/rel_el_mag.html

    Beautiful and impressive. But our question was: Is this the only explanation next in mind?

    Here is from Wiki:

    Quantum electrodynamics[edit]
    See also: Standard Model and quantum electrodynamics
    In modern physics, the electromagnetic field is understood to be not a classical field, but rather a quantum field; it is represented not as a vector of three numbers at each point, but as a vector of three quantum operators at each point. The most accurate modern description of the electromagnetic interaction (and much else) is quantum electrodynamics (QED),[37] which is incorporated into a more complete theory known as the Standard Model of particle physics.
    In QED, the magnitude of the electromagnetic interactions between charged particles (and their antiparticles) is computed using perturbation theory. These rather complex formulas produce a remarkable pictorial representation as Feynman diagrams in which virtual photons are exchanged.

  196. The formulas are the heart of the matter.

    We are moving to receive Shabbat, in the meantime try to calculate the gamma factor when v is less than a meter per second (the speed of electrons in a conductor).

    If you're missing pages for the zeros after the decimal point, you can find some unused forests in nearby Canada.

  197. Israel
    Consider the subject of magnetism. Can you explain attraction between parallel conducting wires in another way?
    It is explained nicely in Berkeley's book - and here is a short video that explains, without the formulas...

  198. Israel
    Muki:
    the behavior of colliding heavy ions can only be explained if their increased density due to Lorentz contraction is considered. Contraction also leads to an increase of the intensity of the Coulomb field perpendicular to the direction of motion, whose effects have already been observed. Consequently, both time dilation and length contraction must be considered when conducting experiments in particle accelerators.

    Longitudinal shortening also explains why two parallel conductors are attracted to each other.

  199. Miracles

    According to the link to the Senyak effect

    https://en.wikipedia.org/wiki/Sagnac_effect#Relativistic_derivation_of_Sagnac_formula

    As always, it can be explained both as the shortening of the length and as the lengthening of the times.

    Where is the only explanation for the trajectories of ions after a collision? I didn't see the link you provided.

    If you bring a mathematical concept with a logical explanation, I will gladly accept. But to say that 1+2+3+4+5+….. = 1/12-, and then bring a link where nothing is written, is ridiculous. Maybe even crushed.

  200. Israel
    Note the word direct. We have no technical possibility to check the change in length of a fast body. But this is the only explanation for the trajectories of ions after a collision. That's what Wikipedia says. In general, if you think about it, length is very difficult to measure from another movement system.

    Airplanes (at least fighter planes) have a system called RLG. This system relies on the Saniac phenomenon, which is the result of a change in the length of a laser's path due to circular motion. In the formulas that explain the phenomenon, the length change due to the movement is entered.

    I don't understand why you underestimate a mathematical concept. The root of minus 1 is also truncated, but it is used a lot in physics, and even in electricity.

  201. Miracles

    In the link about the gathering there is no explanation or proof, only a claim and many counter claims that it is just bullshit.

    Also in the link about the shortening of the length I did not find anything about pigeons or your return and no evidence of the shortening of the length. On the other hand, in the following link:

    http://math.ucr.edu/home/baez/physics/Relativity/SR/experiments.html#Length_Contraction

    It is written explicitly:

    At this time there are no direct tests of length contraction

  202. Decide, 1/12- or 1/12?

    Neither.

    An idea that I'm considering, Cesaro has scattered, if you bet money on the column coming together in half, you'll lose. Even if you put the turtle in a running competition with Achilles or Roseanne Beer in a beauty contest with Sandy Beer.

    Where is the link from the wiki about the pigeons?

  203. Israel
    No need to add anything... The column is infinite and its value is minus 1/12... The proof is simple, but it relies on the convergence I mentioned (1-1+1-1+1-1+1-1+1 = 0.5). Cesaro's idea is to look at the limit of the average value.

    And yes - there are experiments that show the shortening of the length. If you look at collisions of large ions then the only way to explain the angles of the particles is by the shortening of the ions. See Wikipedia….

  204. 1+2+3+4+5+….. = 1/12- only if we add a positive term on the left side of the equation, or a negative term on the right side where the dots are.

    Relativity - do you know any unequivocal confirmation of the shortening of the length like the experiment of the airplanes from the ends of the earth that demonstrated the lengthening of time?

  205. Israel
    Albenzo can explain to you why physically 1+2+3+4+5+….. = 1/12-
    Not intuitive but it is written in a physics book (by Polchinsky) (I thought that in Hebrew the appropriate word is "reason", but you are right that this is what I meant).

    I can explain to you mathematically why this is true ….

    And regarding the train - as far as the passengers are concerned, the train was not shortened. This is only true for an outside observer.

  206. Miracles

    This is what Leibniz said and perhaps philosophically it is true. Physically, you will always get one or two (as in quanta).

    Regarding division by 0: when approaching the speed of light, the mass of a body increases and aspires to infinity according to the Lorentz transformation. When you reach the speed of light and v is equal to c, you get division by 0, but in this case the quotient is equal to infinity and not to minus infinity and is therefore defined.

    And this is in contrast to the function one divided by x which also aims at minus infinity. It is possible to distinguish between mathematics and physics (as Nisimiino does in Zenon's paradox), but if our concern is physics, then physically Zenon's paradox is resolved (infinity), and the mass of a body at the speed of light is defined (infinity).

    And since we are back to physics (infinitely!), then as I mentioned relativity is a beautiful and perfect theory in itself, with the marginal problem that it is not consistent with what we know from the quantum theories, the bang, the strings, Maxwell...

    And not that it doesn't make sense (Nissim meant intuitive, I believe), but nevertheless, a kilometer-long train that is compressed to a centimeter without the passengers' knowledge?

  207. Israel
    The column sum I said can also be 1.5, if you define a schema in the appropriate way. This scheme is called the Cesàro scheme, this scheme is well defined and even useful in physics.

  208. Raphael
    well defined infinity. And as I wrote, there are several types of infinity, and there are also different definitions for the same "type" of infinity.
    Let's look at the natural numbers for a moment. One way of looking at them is called ordinal numbers. The meaning is that the natural numbers constitute a series, and there is a first, second, third and so on. The series is of groups - 0 is the empty group, 1 is the group containing the empty group (it has member 1). 2 is the group that contains 0 and 1 (ie it contains two members - the empty group and a group that contains the empty group). In this method there is a definition for infinity - and it is called omega. Omega is a number for everything. And you can add numbers to omega, multiply it and even increase in power. Any such action is well defined.

    The second way is through share numbers (also called Otzma). The number 3, for example, is defined by all groups that have 3 members (or - whose strength is 3). This definition sounds circular, but that is not the case: group 'A' and group 'B' have the same number of members if and only if it is possible to find a one-one-valued function and from each group to the other group.
    To illustrate, here is a cute example - 'X' is the set of natural numbers, and 'Y' is the set of even numbers. The power of 'X' is A-0, but what is the power of the two? For each x of X, the corresponding y can be defined - its value is 2x. For each y in Y the corresponding x can be defined as y divided by 2 (remember that y is necessarily even). So we proved that the power of even numbers is also A-0.

    In the same way it is possible to show that the power of rational numbers (fractions where the numerator and denominator are natural) is a-0, and also the power of algebraic numbers (equations like 7x^4 + 3x – 19 = 0) – also a-0.

    And if you want, there is also a definition for B numbers, and we don't know if the B numbers are different from the A numbers.

  209. I made a mess in the penultimate paragraph. I wrote that I chose to put x squared in the denominator and Israel x in the fourth, but then when I checked what happens when you multiply by x squared, I reversed the generator and treated the problem as if I chose x in the fourth and Israel x in the square. Hope this isn't too confusing.

  210. I'll try to shed some light on the subject of division by zero if it's of interest to people, although I hope it's clear that we've completely strayed from the subject of the Big Bang singularity.

    Division by zero is not arithmetically defined for the reason Nissim mentioned. Mathematically, division is simply defined as multiplication by the inverse - for example, division by two is to multiply by a number that is the inverse of two (the number that if you multiply it by 2 you get 1, which is of course half). Zero has no opposite (there is no number that if you multiply it by 0 you get 1) and therefore the arithmetic of the operation is undefined.

    You can look at a functional approach. Instead of dividing by zero, we will divide by some number (for example of the variable x) and take the limit that the number goes to zero. A simple example is to take the linear function x itself. Israel says (and rightly so) that the border is not well defined because it depends on whether one is striving for it from the right or the left. If we take the limit on the right, we see that as we get closer to 0, the quotient increases without an upper limit, that is, there is no finite limit and the expression diverges - and this is the reason for the intuition that 1 divided by 0 gives infinity. If we aspire to the limit on the left, we get minus infinity. But it is not true that is why the dose is not well defined. After all, if we want, we can simply look at a different point - for example, an X in a square, and it doesn't matter if we take the border from the left or the right. The problem was the discontinuity of the sign, it could have been easily solved.

    So what is the problem with the functional approach? The problem is, of course, the sensitivity to the phone selection. Israel wants to investigate 1 divided by 0, so he chooses to put the expression x in the denominator and check what happens when he strives for 0. I chose to put x in the square. Neither of us is more right than the other, and we will both aim for the expression 1 divided by 0, but we will get different results. For example, Israel will encounter the sign problem and I will not. This is only the tip of the iceberg.

    Suppose I choose to put x in the square in the denominator and Israel chooses x in the fourth. Neither of us runs into a sign problem, and we both get divergence as x tends to 0. On the face of it, we seem to agree that the portion is infinite. But entertainment is not a binary thing (there is entertainment or there is no entertainment), but a sequence of behaviors (what kind of entertainment, how fast the entertainment is). For example, you can ask what happens if we multiply this infinity we got by 0. Israel can say, "Ok, in the numerator instead of putting 1 I will put the letter x in the square (to avoid sign problems) and take the limit. The answer comes out 1". On the other hand, if I do the same thing and multiply by x squared, I will get that the answer is still infinity, because the entertainment I got is stronger than that of Israel.

    Arithmetic, the operation is simply undefined. The functionality of the operations is well defined, but requires much more information and can give completely different results depending on how fast the numerator in the denominator tends to 0. In fact, there is an entire and developed field of mathematics that deals almost exclusively with this question within the framework of composite analysis.

  211. Raphael

    You can find the definition of infinity in the wiki.

    What's the problem with infinity? What is unclear about infinity?

    And from Wiki, division by 0:

    "Division by zero is the mathematical operation of dividing a number by the number 0, and its result is often undefined."

  212. Israel

    Infinity is not a number. If it were possible to define it then they would define what it is and what it is not.

    For our sake, are you saying that singularity is not defined?

  213. 1 or 2 (the Satan series, Elek..).

    Raphael

    Why is infinity undefined? And where is the sophistication? If you divide by 0 you get both a very large number (infinity) and the minus of that number, not just the first one. That's why the division by 0 is not defined.

  214. Albanzo
    I understood what you are saying. If I take, for example, a field of temperatures and do a linear transformation on them (let's say multiply by x), then there is no way that x will not be 0. For example, x is the sine of time.

  215. Israel
    I mean logical in the sense - common sense. To the average person it does not make sense that the pace of time changes, or that light moves forward. Neither is "logical" in which temporality is not a definite thing.

    Translate "doesn't make sense" to it doesn't make sense.

    Raphael
    You said that 1/0 equals infinity. It is a matter of definition, but it is not the accepted definition in mathematics. The definition of division is the reciprocal of multiplication. That is, a/b = c if and only if a = b*c. And it does not exist in the definition you gave. You can decide that 0 multiplied by infinity equals 1, but then other problems immediately arise: multiply both sides of the equation by 7, then you get 1=7.

    You said infinity is undefined. This is clearly not true. As strange as it sounds, several types of infinity are defined. A-0 (yes, in Hebrew) is the power of the natural numbers (the number of members in the set of natural numbers). A-1 (which is usually called c, or the strength of the sequence), is the number of real numbers between 0.0 and 1.0 - which is greater than a-0 (of course you need to define what "greater" is, and there is a definition for that).

    One should be careful with the word infinity, and not get confused as to whether the idea of ​​very large or very small. I have a "conundrum":

    כמה זה 1+1-1+1-1+1-1+1-1+1-1…… עד אינסוף?

  216. Israel

    This is cleverness. It is known that infinity is not defined but it is not zero. Whereas in the singularity it was explained that the scale is literally zero.

  217. Raphael

    "Until now I knew that 1 divided by zero is infinity."

    Why not minus infinity? It doesn't depend on which side you aim for division by zero?

    See for example…….000000000000000000000000000000-/1

    The smaller the negative number, the quotient tends to minus infinity, while if the number is positive, it tends to infinity.

    So how much is it when the division is exactly 0? Infinity or minus infinity?

    This means: not defined.

  218. Albanzo

    1. You wrote "a universe of size 0 is not necessary for the model and it is likely to be assumed (in fact, quite certain) that it will not be accepted as a solution of quantum theory for reasons of localization. "

    What does that actually mean? That there was always something there, only that it began to expand at a certain time?

    2. Regarding this "a trivial example of a phoenix that is truly singular is one of the parts of z. It has a singularity at the point z=0.”

    Until now I knew that 1 divided by zero is infinity. So maybe after all there is a connection in physics or mathematics between absolute zero and infinity?

  219. In fact, phoenixes to which you can do what you wrote in your response (assign a value to them at the singular point by taking a border on the right and left) are usually not considered truly singular phoenixes, but only phoenixes that have not been defined correctly. A trivial example of a phon that is truly singular is one of parts z. It has a singularity at the point z=0.

  220. Yes. It's not that there is a difference between physics and mathematics, it's that even in mathematics what you wrote is not accurate. First of all, you are talking about a clearing discontinuity point, which is a very particular and specific case of a singular point of Pon', and really not the general case. Second, you talk in particular about the singularity of Pon'. Singularity can appear in phon but also in other mathematical objects, in particular in operators, or copies.

    In the case of the Big Bang, the singularity is in the conformal copy. A nice way to think about it is that you took a flat space (that is, whose scale is the constant 1 and is independent of time), and you applied some copy to it that stretched it and gave you back a different space. Specifically for a=0 a degeneration is obtained because the space changes from a d-dimensional space to a 0-dimensional space, and the copying actually copies all the points in the space to the same point.

    In principle, a gravitational singularity can also be interpreted as a singular point in a phoenix (for example, if the scalar curvature is written as a phoenix of a certain parameter, the gravitational singularity can be a divergence in this phoenix). But as I said earlier, even in this case it will not be a clearance discontinuity (that is, it can be redefined as a boundary of the phon in the environment around the point). Besides, sometimes these points have singular points which are not attributed to a gravitational singularity (for example, those which arise simply from a bad understanding of a coordinate system).

  221. Albanzo
    Usually in mathematics the meaning of a function value at a singular point is defined as the limit of a continuous function. For example sin(x)/x is 1 when x=0, in the sense that we can take x as close to 0 as we want. Is the meaning in physics, in particular in general relativity, different?

  222. A universe of size 0 is not necessary for the model and it is likely (in fact, quite certain) that it will not be accepted as a solution of quantum theory for reasons of localization. The very change of scale in space is a simple and accurate explanation to the best of our knowledge for observations, and is therefore also included in quantum models. But the extrapolation up to the point where a=0 results from the solution of classical equations that will not exist in quantum theory.

  223. waiting
    I guess that amount of swearing, swearing and links to XXXX and Bitcoin sites that I loaded in my previous response causes a block

  224. albentezo,
    I will try to refine. You wrote that - "According to the big bang model of the theory of relativity, the universe started from a singular point, that is, all infinite space had a scale of 0"
    Since this was probably not the case, does the explanation you gave survive? Is the initial condition: a universe of size zero a critical detail for the model or not but the model will survive the change with the requested corrections?
    In light of your last answer I understand that yes and yes I hope that now I am a little more understandable

  225. Shmulik,

    I'm not sure what you mean by "explaining the scales". I quickly went through my previous comments and it seems to me that everything I wrote is common to both classical cosmology and what we think exists quantumly, except of course the physics near the point of the bang. But if you want a more precise or decisive answer, you'll need to clarify exactly what you mean.

    June,

    First of all, Scalar is something else entirely. Despite the similarity in word, scalar has nothing to do with scala. This scalar is a term from the world of mathematics that means a quantity that remains constant after one transformation or another, when the most popular use of the term is towards a rotation transformation (therefore a vector that has a direction and therefore will change under a rotation transformation is not a scalar, but some number - such as the energy of a particle - will not change under rotation and therefore a scalar size).

    A. An article that talks about what exactly? About how an infinite universe expands? The articles concerning this were written and published decades ago. Today it is easier to find these things in any introductory book to the theory of relativity, in the chapter on cosmology. For example, Sean Carroll's book is a fairly clear book that covers the subject in a very simple way.

    Spacetime and Geometry / Sean Carroll

    B. The explicit form of the phon of the scale (note, this is the phon of time: that's the whole point, it's not a constant size but something that changes in time) is determined by solving Einstein's equations, or actually Friedman's equations, and therefore depends on what's in the space. The whole idea of ​​general relativity is that gravity is the interaction between space itself and what it contains. Therefore the solution of the scalar function depends on the exact model and the contents of its universe. What we usually look at is a universe that is more or less homogeneous and has a low density (an excellent approximation to our universe), then the scale factor can be solved and found. Note that it is a non-trivial function of time, and behaves differently whether the dominant factor in the universe is matter or radiation, for example. Forgive me for not writing phon' or equations here, but as for the previous section - the explicit form can be found in any introductory book to general relativity and cosmology, and in particular in the book I mentioned earlier.

    third. By four billion years everything was smaller. Regarding the sun in particular, I don't know how to answer clearly because I'm not an expert in stellar evolution (it's astrophysics and I study string theory). I have no idea what the sun looked like four billion years ago, maybe it was bigger due to the nature of the development processes of stars. If you want to know what the scale factor is at a certain time, you are welcome to place numbers yourself in the formulas found in the books I mentioned.

    d. I have no idea what you mean or what you are talking about.

  226. I understood what you wrote about the scalar.
    A. I would appreciate it if you could direct me to an article that talks about it.
    B. I would appreciate it if you could give me the value of the scalar and its rate of change, since this is how the time since the big bang is measured.
    third. According to you, about four billion years ago, was the sun smaller than today because the scalar was smaller, and if so by how much? (If possible please provide a reference to a scientific source).
    d. According to the updated knowledge the rate of expansion is increasing, and despite this the most distant stars that we observe are not moving away at a speed of half the speed of light, what was once taught that the rate of change of the scale was lower, and if so how despite the passage of about 13 billion years did the stars manage to move away from us a greater distance than that?

  227. Albanzo
    We ignore the time extension in the pool ball exercise, because the time extension has no meaning in this context. Not so the situation at the singular point.

    I'm afraid we're talking about different things, and I probably didn't really explain myself well. I have no reason to argue with you about the physics itself 🙂

    I'm not debating what the theory of relativity says. I'm just saying that it's weird to me to talk about what relativity says where we know it's invalid.

    I understand (now) that you are saying that Raphael's claims are wrong, also in the context of relativity. Thanks for the explanation!

  228. 1. What does "how can one ignore" mean? When you solve a billiard ball problem in classical mechanics, do you consider time dilation? Considering the Pauli force between the balls? In physics we build a model and then look at what solutions it provides to problems. In the model of general relativity there is no uncertainty and there is no quantum phenomenon, therefore when solving a problem with this model you get results that contradict these principles.

    2. No, Raphael was wrong when he said that and you are also wrong now. It is possible to write models of a (0-dimensional) point that expands into a d-dimensional space. There is no geometric problem with this as Raphael thought (and as I think you think). Are there problems with this that arise from quantum localization? Yes, in principle. Therefore, in a theory that does not include quantum principles, such a thing would be possible, and in a theory of quantum gravity, it probably isn't, at least not in the same way (although we can't say for sure that it won't be possible at all simply because we don't know for sure yet).

    3. Our discussion certainly touched on both what the theory of relativity says, and the question of whether or not it fits reality. The big bang model is a model in the framework of relativity, not string theory or quantum loop gravity. You can go back and see that throughout my comments I distinguish between what the big bang model says and what is really true, and try to clarify when relativity is reliable and when it is not. When you write that the Big Bang model does not start from a point, you are simply wrong, because it does. Maybe it's not a good model because it's not quantum, but it starts from a point.

    4. The number of times I have written that the theory of relativity does not give a full explanation of the cosmological problem is too great for me to have the strength to count it and quote it here.

    5. Again, read my comments and see how many times I write that the singularity is a predication of relativity and that modern theories contradict it, so this predication should not be taken as truth. I don't understand how it didn't come out of my mouth. For example, quote

    "As I commented, the fact that according to general relativity it is true does not mean that it is true in principle because this period of time (where the scale was very, very small) is precisely the weak point of the theory of relativity where it is not reliable. That's why modern researchers look at other teachings and come to other conclusions."

    6. Raphael had questions regarding the theoretical model, I tried to clarify them. We know that classical mechanics is wrong, but if someone comes to me and claims that classical mechanics does not make sense or that Newton's equations create a situation where relative motion cannot exist, I will prove him wrong and try to explain the issue to him. This is not to say that I claim that the universe is described by classical mechanics. Just as it is nice to understand classical mechanics (even though it is wrong) and there is a point in understanding it because you can draw from it knowledge and intuition for more advanced and accurate theories, so there is a lot of beauty and a lot of point in understanding classical models of cosmology. Therefore, when Raphael asked questions, I answered them while clarifying unambiguously that I was explaining to him how a model works even though the model is not accurate and is not really what physicists think happened in the early universe.

    I wrote this several times during the discussion, you can look for yourself.

  229. Avi,
    What did I already write that the response is waiting for confirmation?
    Additionally, I agree with the request of others to bring back the recent comments page. A big part of the fun of this site is being able to easily get to the latest comments.

  230. Will the explanation of the scales be correct even if there was no singular point due to quantum effects?
    In other words, will the explanation from Hal be true also under quantum gravity theory?

  231. Albanzo
    I understand what you're saying.
    But, I don't understand how you can ignore quantum theory at the singular point. Raphael makes an argument that is logical - no inflation of magnitude 0 will increase the point, and certainly not to something infinite. Our discussion is not about what the theory of relativity says at this point, but about what really happens there (or then..).

    All I'm trying to say is that I don't think it's necessary to claim that the theory of relativity gives a full explanation of what happened, and to my understanding, that's what you write implies. If the singularity is a result of relativity, in a place/time that you yourself say is not valid, then why are you explaining how the universe expanded from that point? It seems to me that this is not necessary.

  232. Miracles,

    What I explain over and over again is that the whole theory of relativity contradicts quantum theory. It is classical theory, not quantum. There is no need to look for singular points in the big bang, look at the movement of a free particle in empty space in the theory of relativity and you will see that the uncertainty principle is broken.

    What I wrote in several of my last comments is that the theory of relativity does contain a singular point of size 0, but which is a classical theory. That's why we know it's not accurate. Up to a certain point it is a good approximation to reality, but starting from a certain point it is no longer and then quantum gravity is needed. In any case, you cannot use arguments of uncertainty in the framework of general relativity because it does not respect them, just as you cannot take a solution to a problem in Newtonian physics and say that it is good or bad depending on the uncertainty principle. The entire Torah contradicts quantum mechanics, so these arguments are irrelevant.

  233. Raphael
    you surprised me I thought you started believing because it made sense to you. I have heard many "logical" arguments from you for the existence of a creator, such as for example - the existence of natural laws.

    I must have been mistaken.

  234. Raphael
    What we call "logic" does not deal with the problems of the very small or the very large. Logic says that the earth is flat and the sky is above us. Logic is what guides children in kindergarten (ask a small child to draw - where does he put the sky? After that we are convinced that it does not work beyond our daily lives.

    Logic is something completely different, and it took many, many years until we learned the laws of logic (Aristotle, Boll, Frege, Russell, Wittgenstein and so on).

    Raphael - I think you also agree that religious belief comes from logic, don't you?

  235. Albanzo
    Doesn't this contradict the quantum theory saying that all matter is static, at one point? I don't know if it contradicts the uncertainty principle, or Pauli's principle, but it should work, right?

  236. Miracles,

    This is not true. First of all, uncertainty is irrelevant because relativity is not quantum. So in the interpretation there is no uncertainty in it. Second, there is definitely a singular point where the scale factor goes to zero. Just as there is a singular point inside a classical black hole, which quantum will probably spread and not be singular.

  237. Rafael/Albanzo
    I don't think all Big Bang models start at an absolute point. This contradicts the uncertainty principle, and I also don't think it is necessary that the beginning was a point. The "singularity" is the point where the theory of relativity does not hold, precisely that x/x is not defined at 0. The math allows us to set the value at 0 as a limit, but that doesn't mean the value at 0 is 1!
    It reminds me a little of Permendes' Torah, and Zeno's paradoxes. Mathematically - the paradoxes are solved, but philosophically - there is still no solution 🙂

    Raphael - Regarding time, there is a concept called "proper time" this is the time that an observer at that point will measure, and this time is well defined.

    Don't try to use logic to understand the big bang. Relativity defies logic (I mean common sense, not philosophical logic), quantum theory defies logic. So don't expect that if you bring both to the limit then you will get something reasonable 🙂

  238. Raphael,

    I don't know what to tell you, it's just not true. We "occupy the whole space" as you say if and only if the scale factor is 0. For any other number, we don't. Look at the example I gave, place any number except a=0, you will see that you and I are moving away from each other. It's pretty simple math, if you want you can try to look at it - if you understand what a metric space is then all you need to look at is the FRW matrix. All this talk is a bit pointless because you can see that you are wrong by calculating one second (if you know what a metric is).

  239. Albanzo

    If you and I do not grow relative to the scale, then a distance is created between us that allows movement in space, but if you and I grow together with the scale, then just as in the beginning we occupied all the space (which was then zero) so we will always occupy all the space, no matter how much the scale grows, and in that case no There will be room to move.

  240. The distance was 0 only at one moment in time. Motion is always defined over a segment of time. Let's leave the scale as vanishing, let's call it a.

    Let's assume that the distance between me and you is one bar, that is, a distance of 1*a (as in the previous example - if the scale is 1 then it is a distance of one meter, if it is half then half a meter, etc.). But this is a snapshot of a single moment. Think of the picture in which at the starting moment the distance between us is a and a second later it is 2a, two seconds later it is 3a etc. You say that if we set a=0 then we get that there is no movement, which is technically true, but of course meaningless because the whole point is that the scale changes over time. At the moment of the bang it is equal to 0 but a fraction of a second later it is no longer 0. So there is no problem with relative motion as long as you don't look only at the moment of the bang itself (and as I said, without anything to do with relativity when talking about motion you can't just look at a moment Single).

  241. albenza,

    1. Thanks.

    2. If the distance between me and you was initially zero and everything expands in the same ratio, then the distance between us will always remain zero because you and I also grow in the same ratio and then no movement will be possible...

    3. Thank you for clarifying that 13.7 billion years is related to the reference frame of the theoretical gauge (which at the moment of the bang settled in place in empty space and has not moved or passed through a significant gravitational field since then). The problem is that not everyone is aware of this and it creates a lot of friction and difficulties that actually have no place.

  242. 1. According to the big bang model of the theory of relativity, the universe started from a singular point, that is, all infinite space had scale 0 and therefore topologically it is equal to one point. You ask "how" but this is exactly what I explained to you. If the explanation is not comprehensive enough for you, you are welcome to read more professional literature, but this will require the development of non-trivial mathematical abilities. As I commented, the fact that according to general relativity it is true does not mean that it is true in principle because this time period (where the scale was very, very small) is exactly the weak point of relativity where it is not reliable. That is why modern researchers look at other teachings and reach other conclusions.

    2. The fact that everything expands together does not come at the expense of relative motion. If two ants are on a balloon, one stands still and the other moves away from it, and the balloon inflates, then both effects happen simultaneously - the ants both move apart as a result of the inflation and there is relative movement between them. I don't understand what the problem is. And that a change in scale is not a change because everything changes is not true, because there is time. At any point in time if you freeze-frame and look at the universe you will indeed not be able to tell what the scale is. But there are methods and measurements that can tell what the scale is today in relation to what it was yesterday.

    3. The flow of time is indeed relative and can be played with. The numbers that are usually given for the time that has passed since a certain event refer to a certain frame of reference (an observer moving with the universe). For that matter, you can say that this is the time that would have been measured by a person who at the moment of the explosion settled in place in empty space and since then has not moved or passed through a significant gravitational field. Other viewers may see a shorter time depending on their frame of reference.

  243. albenza,

    We have indeed talked a lot in the past around this topic. But now I want to clarify a matter of principle. If the universe started from a singular point that is literally zero (and not as explained by Nissim) then we still need a simple explanation of how the universe can be zero and infinite at the same time. And if so - what is the difference between zero and infinity?

    Another thing I want to ask due to your parable about the distance between you and me that has shrunk by half. You say that in that case everything shrunk by half. That is to say that actually there was no change because you and I also shrunk. But in the universe you don't see that everything is expanding together because otherwise we wouldn't be able to notice any movement, for example - we wouldn't be able to notice the galaxies moving away from each other because the galaxies themselves are also growing in the same ratio.

    One last thing - please refer to the matter of 13.7 billion years. Would the creatures living on another planet with a much greater gravity than ours agree that 13.7 billion years have passed since the universe began to expand?

  244. Raphael,

    I'm sure we've had this conversation before and I've already explained all of this to you.

    First you need to understand what Scala is. If today you and I are standing next to each other and we are separated by a ruler that says "1 meter", then the distance between us is a meter. 100 such rulers can be inserted along a football field, so it is 100 meters long. About 80000 such rulers fit between Haifa and Tel Aviv, so the distance between them is 80 km.

    Yesterday was a different scale. As on a computer you can sometimes stretch an image or shrink it without changing proportions (grab the corner and drag), so is the universe. So if yesterday the scale was 1/2 compared to today, then you and I - even though we are still separated by the same bar - would be half a meter apart. The bar also shrank, everything shrank because space itself shrank. On the soccer field, 100 rulers go in, but it is 50 meters long, and between Haifa and Tel Aviv, 80000 rulers go in, but the distance is 40 km. Yesterday the situation was even worse, and the scale was 0.1. That is, the distance between Haifa and Tel Aviv was 8 km, although 80000 rulers still went between them, etc. Note that on each day, although the scale is different, the universe is still infinite. You can still move forward in any direction you choose at any distance you choose. If Moshe is suspected of bribery and wants to flee 3000 km from the State of Israel, then today he can flee to Switzerland. Yesterday we had to run to the east coast of America. Yesterday he had to flee into outer space to get 3000 km away from the law enforcement authorities, but in any case it is possible. The universe has no end in either case.

    You can ask what happens if you go back in time to the point where the scale is literally 0. So the distance between me and you is 0, even though we are separated by a bar. The length of the lot is 0 and the distance between Haifa and Tel-Aviv is 0. Since the distance between any two points in the space is 0, the space is actually a point (it is a 0-dimensional space equivalent to a point). This is what is meant by the singular point - the time when the scale was exactly 0 and therefore every two places in the universe were separated from each other by a distance of 0 and were in fact the same place. The term singular point is derived from the mathematical expression which means that the conformal transformation (which changes the scale) is singular - not defined in units. It maps all points in the universe to the same place.

    Contrary to what you wrote, according to the theory of relativity the universe did start from this point. It is possible to go back after the inflation of our universe to a time when the scale was literally zero. But, as I wrote to you before, before reaching this time, we reach a time when quantum effects become equal in importance to gravitational effects in the evolution of the universe, and therefore quantum mechanics cannot be neglected and the process must be described with the help of a theory that combines both gravity and quanta. General relativity is not like that and therefore its predication (that the universe was literally a point) is not necessarily reliable. What is reliable? It's a bit long and we've talked about it a lot in the past (I remember you bringing up a video of researchers from the Perimeter Institute talking about what was really in the bang and what preceded it, by looking at quantum theories of gravity instead of general relativity while close to the bang).

  245. Miracles

    In other words, you are saying that the universe, as physicists see it today, did not start from a point, because a point does not take up space at all, but rather from a very small three-dimensional initial body that swelled up. So 13.7 billion years is not the time that has passed since the universe was created but it is the time when it started to inflate for some reason.

    I also remind you that it is still not understood how to calculate 13.7 billion years since time is not a global matter but a very personal matter and differs from body to body according to the speed at which the body moves and according to the gravitational force exerted on it.

  246. Raphael
    Think of it like this: you are at the singular point and at time 0 you start moving. The point "inflates" much faster than you are able to move. Therefore, for you, there is no "end" to this universe.

    The point is not of size 0, but of some minimal size, very small.

  247. Albanzo

    Can you explain to simple science-loving people how an infinite universe starts from a singular point that is definitely finite?

  248. June,

    I read your comments again and again and cannot understand where you see a contradiction. Can you explicitly write two clauses and show that they lead to opposite conclusions, meaning that there is a contradiction between them?

    And by the way, it doesn't matter if you write "see Wikipedia", it won't make anything you say more or less true. The universe has always been infinite and the whole point is only in changing the scale of space. The universe has and never had a size (according to cosmological theories accepted today).

  249. This is exactly the contradiction, on the one hand according to the big bang theory everything started at a singularity see Wikipedia, then there was inflation that increased the universe to the size of a medium galaxy see Wikipedia, and since then everything has been according to the physical sizes we know today. On the other hand, they claim that there are stars that are even more than 13 billion light years away from us, or as the "observable universe" defines it.
    I also agree that when the light left the star it was closer but because space itself stretched, it took longer to arrive. But we know that the space between us and the most distant quasars stretches at about a fifth of the speed of light, and this is what causes the redshift, so we can calculate with a rough estimate that if it took light 13 billion years to reach us, then this quasar was at a distance of about a fifth of its distance today, which is about a distance of 10 billion light years, it is implied that it managed to move away since the big bang in 690 million years, about 10 billion light years.

  250. June,

    What you write is wrong. The universe (as we know today and according to the accepted cosmological theories) has always been infinite. What changes is only the relative scale of distance. The universe was never the size of a galaxy, or any other size. Maybe you're getting confused with the visible universe, which is a whole other thing? Either way, the relevant question is how long it takes for two objects to interact (for that matter, exchange light between them). Before inflation, the scale was so small that more or less any two objects could interact almost instantaneously and the sky was homogeneous. After her it is no longer true.

    The universe is expanding, therefore light that traveled for 13 billion years and reaches us today, did not start at a distance of 13 billion light years from us but closer, and the object that emitted it is today not 13 billion light years from us but more. This is exactly what the change in scale means. But I don't understand what all this bothers you about.

  251. יוני
    You look at the Big Bang as an explosion in a three-dimensional world. In such a world, it would really take a relatively short time for light to arrive.
    But the universe is 4-dimensional, and what is happening is not like a normal explosion that we know. Such an explosion has a center, a point in space where the explosion happened.
    In the Big Bang, the situation is different. Space itself expands. The result is that light takes much longer to arrive. There is even a range from which light will never reach us and we will never know what happened there (or then....).

    And by the way - to my understanding, the black hole described in the article is much further away, something like 45 billion light years, or much less than 13 billion years old...

  252. It is true that according to the big bang theory the universe was created before 13.7/690, but it appears on the site that they saw a black hole from a time when the universe was 13 million years old, that is, XNUMX billion years ago.
    To the point: according to the theory of inflation, the universe grew in a few fractions of a second to the size of a medium-sized galaxy, which means that the light should have reached us in a short time (at most a hundred thousand years), even if it continued to move away at a very high rate, even for that matter at a speed of one-fifth of the speed Light (which is absolutely enormous speed) for 13 billion years, light would take a fifth of that time to reach us.

  253. יוני
    The big bang was 13.7 billion years ago, not 13. As Albenzo explained to you, there was a phase of inflation in which the universe expanded much faster than the speed of light, but the time it took (if the concept of time can be used at all) is a small fraction of a second.

    Your last paragraph is not what Albenzo wrote at all. read it again

  254. So if I understand correctly, you are claiming that it was 13 billion light years away from us in less than a few seconds. I read about the big bang quite a lot and I understood that they came to the conclusion that everything was at one point because we see that the stars and galaxies are moving away from us, so if we rewind time we arrive at the fact that 13 billion years ago everything was at the same point, but according to your claim 13 billion years ago there were already stars that were far away from us A distance of 13 billion light years.

  255. lover,

    Its size is more or less the same. The amount of radiation released by a black hole is inversely proportional to its mass. That is, large holes almost do not radiate. Even a black hole the size of the sun's mass the radiation is really tiny. The orders of magnitude of the time it would take for such a large black hole to accumulate enough energy for us to notice the decrease in mass is much larger than the lifetime of the universe.

    June,

    It is true that when the universe was young enough, everything was awfully close and more or less any two objects could interact in a short time, but this period is pre-inflation. Cosmic inflation inflated the universe by many orders of magnitude faster than the speed of light, after which there was no longer such homogeneity. The black hole in question is indeed very far away and very old, but these are not relevant time constants for inflation. In other words, when it was created the universe was already quite large and in a phase of slow expansion, similar to the picture we see in the day when we look at the universe.

  256. Apparently there is a contradiction: on the one hand they claim that the light came out more than 13 billion years ago before it reached us, which means that by the time it came out it was already very far away from us (I don't know how much, because it is clear that we moved away while the light "traveled" towards us), on the other hand according to the big bang theories The big one, the whole universe is very small in the beginning so the light should reach us very quickly. I would appreciate it if someone could sort me out with the calculation.

  257. I wonder what the size of this black hole is today in light of the Hawking radiation emitted from it in the last 13 billion years.

Leave a Reply

Email will not be published. Required fields are marked *

This site uses Akismat to prevent spam messages. Click here to learn how your response data is processed.